Simple Time-Dependent Model of the Atmospheric Greenhouse Effect

August 30th, 2016 by Roy W. Spencer, Ph. D.

I thought it would be a useful followup to post a simple time-dependent energy balance model (spreadsheet attached) to demonstrate how infrared radiative flows affect the Earth’s surface temperature and atmospheric temperature. (I might have done this before…it sounds familiar).

The model is the simplest I could come up with to demonstrate how an atmosphere that absorbs and emits IR radiation ends up warming the surface, and itself as well, while maintaining an atmospheric temperature below that of the surface.

Here are the basic energy fluxes included in the model. The illustration is just schematic.

Fig. 1 Cartoon representation of the energy flows in the simple time-dependent energy balance model of the climate system

Fig. 1 Cartoon representation of the energy flows in the simple time-dependent energy balance model of the climate system

The energy input from the sun is fixed at an assumed 240 Watts per sq. meter. The radiative fluxes use the Stefan-Boltzmann equation (sigma T^^4), where T is either the surface or atmospheric temperature. Surface emissivity is assumed to be 1 (changing it to 0.95 or less make no difference to the conclusions, only the details).

You can adjust the IR absorptivity of the model in the spreadsheet, which is just a multiplier on the radiative flux coming from the atmosphere, and the radiative flux coming up from the surface and being absorbed by the atmosphere.

The model is initialized at absolute zero Kelvin, and heat capacities are prescribed so you can see the temperature changing over time as the model goes toward energy equilibrium. The heat capacity of the surface and atmosphere are assumed to be the same, equivalent to 1 meter of water for simplicity (the atmosphere is really more like 2 m of water effective heat capacity).

Using “0” for the atmospheric absorptivity leads to a surface temperature of 255 K, and zero atmospheric temperature (the model is radiative only, no convection, no conduction, so without any atmospheric absorption of radiation, the atmosphere cannot warm):

simple-radiative-EBM-of-sfc-and-atm-abs0.0

Then, to see how this “no-atmosphere” earth changes with an atmosphere that absorbs and emits IR, an IR absorptivity of 0.8 gives a surface temperature close to 290 K, and an atmospheric temperature of about 244 K.

simple-radiative-EBM-of-sfc-and-atm-abs0.8

If the model had dozens of atmospheric layers all interacting, it would produce much higher surface temperatures, and much lower temperatures in the upper atmosphere, producing a strongly super-adiabatic temperature profile (Manabe and Strickler, 1964). This is what causes atmospheric convection, which provides a net transport of heat from the surface to the middle and upper troposphere (not contained in this radiation-only model).

Again, this is an EXTREMELY simplified model of the effect of radiative flows on the global climate system. It is only meant to demonstrate the most basic components of the atmospheric “greenhouse effect”, which act to:

1) make the Earth’s surface warmer than it would otherwise be, and

2) keep the atmosphere cooler than the surface (since the atmosphere cools radiatively to deep space, but partially “blocks” the surface from cooling to space).

UPDATE: Based upon a few comments, it might be useful to point out:
1) the final equilibrium temperature does not depend upon the initial temperature assumed at the beginning of the model integration, it can be 0 K, 100 K, or 1,000 K.
2) the final equilibrium temperature does not depend upon the assumed heat capacities of the Earth’s surface and atmosphere…those just change how much time it takes for equilibrium to be reached.


1,458 Responses to “Simple Time-Dependent Model of the Atmospheric Greenhouse Effect”

Toggle Trackbacks

  1. Ed Bo says:

    Oh, now you’ve gone and done it! Introduced transient as well as steady-state conditions. Heads are going to explode around here!

    • I’m waiting for the claims that the quantitative, simple model is SOOOO unrealistic that it can’t be useful. Yet, their qualitative hand waving with words is supposed to be more accurate.

      • gbaikie says:

        We can find a lunar surface and an Earth surface which is 70 C.
        70 C on average over 100,000 square km area for both Moon and Earth. So something like desert on Earth which is 316 km square.
        Or time of day at lunar equator or a higher latitude at noon.

        So you are in middle of this 316 km square and going up in elevation
        of both locations and measuring the amount energy looking down which going pass you and going upward towards the space.
        So measure at 1000 meter elevation and then 2000 meters, and so on.
        Is there difference between Earth and the Moon at 1000 meter elevation?
        I imagine there is difference. Earth surface is losing energy due to convection and Moon isn’t as one example. And there are other factors which could make difference.
        But assuming a baseline at 1000 meter, it what manner does it alter
        as you increase in elevation?
        Or 2 km, 3km, 4 km, etc anything exciting happen? Is the growing trend of something exciting happening?

        • I have no idea what you are talking about.

          • gbaikie says:

            “I have no idea what you are talking about.”

            Well the ocean and land surface is suppose to cool slower
            due to radiant effects of greenhouse gases.
            So anything warmer than air temperature should cool quicker
            as compared to cooling in a vacuum.
            So if had large rock which was 50 C and air temperature cooler than 50 C, then the rock apparently should cool slower on Earth because of the GHE.
            And, so, how much slower?
            So say Earth sky at night is say +200 K and Moon sky is 2 K.
            Obviously if object temperature is close to air temperature there is not much cooling, so in comparison of difference between the Earth and the Moon the object needs to be warmer than the air temperature- and how much is the difference suppose to be.

            Or take a sealed 55 gallon barrel of water, put on the lunar surface, how long before 50 C water cools to 30 C.
            Does it cool twice as fast, 3 times as fast, 1/2 as fast, or what?

            Or do we agree that if the air temperature is -30 C, then a 50 C barrel will cool a lot faster compared to 30 C air temperature.
            So what would air temperature on Earth would it need to be, to equal the cooling rate of 50 C barrel of water on the Moon?

      • coturnix says:

        The model is somewhat much oversimplified,since the temperature that everyone refers to as ‘the surface temperature’ and goes all nuclear about when discussion touches climate change topics, is not the actual surface temperature of the earth, but rather the temperature of the boundary layer of air (something like 2 metres above surface, i am sure know it). And we all kinow while those are certainly correlated they are not identical. I myself observed many tomes during the sunny days how ground surface temperature can be much higher or lower than the air temperature. While the model certainly useful to demonstrate what greenhouse effect is all about, that very variable that everyone goes coocoo about is MISSING from the model! It needs to be put back in, and then because the model creates some temperature *discontinuity* at the surface, you must put in some non-radiative heat transfer as well, even if convection is not included.

    • Rosco says:

      How do you add metreKelvin to Kelvin ?

      These aren’t even equations !

      • Ed Bo says:

        Roy’s adding Kelvin to Kelvin in the proper way.

        The only thing screwy is that the unit labels did not show up correctly, showing as “W/m” instead of “W/m^2”. It’s tough to get superscripts to show up properly in Excel.

        It should have been totally obvious from context.

        • Rosco says:

          Ed Bo says – mistakenly by the way.

          “Roys adding Kelvin to Kelvin in the proper way.

          The only thing screwy is that the unit labels did not show up correctly, showing as W/m instead of W/m^2. Its tough to get superscripts to show up properly in Excel.

          It should have been totally obvious from context.”

          OK Ed – let’s see.

          From both the current spreadsheet and one sent to me years ago we have – both of which are ostensibly the same only some slight changes in terms to protect the innocent:-

          Temperature of surface :-

          B12 = B11 + ($C$1 – C12 + $C$4*0.0000000567*E12*E12*E12*E12)$C$2/$C$3

          Temperature of atmosphere :-

          E12 = E11 + (D11 – F11 – G11)$C$2/$C$3

          Let’s evaluate the B12 expression – simple inspection shows the atmosphere temperature calculation will evaluate the same way.

          Here goes :-

          B11 is Kelvin.

          $C$1 is constant 240 W/m2

          C12 is variable with units of W/m2 emitted flux by surface.

          *$C$4*0.0000000567*E12*E12*E12*E12 is also variable with units of W/m2.

          Hence the expression in brackets evaluates to W/m2 !

          $C$2 is seconds as stated explicitly

          $C$3 is – direct quote – J per deg. per cu. metre of water (or 50 % odf atmosphere depth)

          So lets see how we go – again direct quote from the spreadsheet

          B12 = B11 + ($C$1 – C12 + $C$4*0.0000000567*E12*E12*E12*E12)$C$2/$C$3

          K = K + (W/m2)*seconds*1/J/(m3.K)

          K = K + (W/m2)*seconds*K*m3/ J

          K = K + (W/m2)*seconds*K*m3/W*seconds

          W cancels, seconds cancel and we are left with

          K = K + K*m

          Exactly as I said – no mistaking anything about context.

          As 2 “equations” are dimensionally inconsistent AND their evaluated values are input to other “equations” NONE of the “equations” are correct.

          What do you mean “The only thing screwy is that the unit labels did not show up correctly, showing as W/m instead of W/m^2. Its tough to get superscripts to show up properly in Excel.”

          Firstly it is child’s play to properly format Excel spreadsheets – the 2013 version sort of has this right – still the same dimension problem though !

          “It should have been totally obvious from context.”

          Ed – the ONLY thing that is OBVIOUS is that YOU didn’t actually examine the spreadsheet !

          Din’t Kelvinmetre used to make refrigerators ?

          • Rosco says:

            Sorry slight mistake in
            B12 = B11 + ($C$1 C12 + $C$4*0.0000000567*E12*E12*E12*E12)$C$2/$C$3

            Should obviously be E11 not E12 – doesn’t matter though – the analysis is the same as E11 and E12 are temperatures in K.

          • Ed Bo says:

            If you had bothered to read Roy’s description of what he was calculating, you would have seen that he was using the top 1 meter of water.

            So you’re upset that he didn’t put a “divide by 1” in every cell?

            If he had wanted to use the top 10 meters, he would have needed to divide by 10, but here there was no point. Excel calculations don’t care about units.

            Have you ever done this kind of calculation before????

          • Rosco says:

            I can’t seem to reply to Ed Bo’s last comment so I’ll do it here.

            No Ed I couldn’t give a tinkers cuss about”a divide by 1 in every cell?”

            That is simply one of the stupidest arguments I have ever heard !

            I’ll say it again – the equations as written are dimensionally inconsistent!!!!

            B12 = B11 + ($C$1 – C11 + $C$4*0.0000000567*E11*E11*E11*E11)$C$2/$C$3

            I fixed the little subscript problem here but it makes no difference.

            This expression evaluates to Kelvin (B11) + Kelvin.Metre.

            The temperature calculation for the atmosphere is:-

            E12 = E11 + (D11 – F11 – G11)$C$2/$C$3.

            Again, this expression evaluates to Kelvin (B11) + Kelvin.Metre.

            Let’s see it again

            E11 has units of Kelvin.

            D11, F11 and G11 all have units of W/m2.

            $C$2 has units of seconds.

            $c$3 has units of Joule/(Kelvin.m3) = Watt.second/Kelvin.m3

            Evaluate E12 = E11 + (D11 – F11 – G11)$C$2/$C$3

            and we have Kelvin + (W/m2)*seconds*Kelvin*m3/Watt.seconds

            The seconds cancel, the Watts cancel and the expression evaluates to E12 = Kelvin + Kelvin.metre

            It is even worse if one uses the standard definition for heat capacity units of J/K.

            Then it becomes

            E12 = Kelvin + Kelvin/m2

            Ed – What does the “top 1 metre of water” and “divide by 1 in every cell” have to do with anything ?

            Roy wrote the equations and they are INCORRECT because they fail to maintain consistent units !

            Ed smugly asks “Have you ever done this kind of calculation before????”

            (He seems unable (unwilling) to admit the problem.)

            NO – I have obviously never performed this type of calculation because it isn’t a valid calculation !

            Let me explain it to you with a mechanics example showing how wrong this “type of calculation” is-

            Force and bending moment are obviously “linked”. Force has units of newtons, N, and bending moment has units of newton metres N.m.

            There isn’t an engineer on Earth that would add these two quantities.

            N + N.m = gibberish

            K + K.m = gibberish

            Get it ?

            If this happened in engineering no multi story building would stand.

            I saw clearly several years ago – August 2 2013 if you want the actual date was when it was sent to me – how these equations were crafted to provide the desired result. I saw immediately the dimensional problem.

            I have simply analyzed the equations as presented in the spreadsheets and applied all the units from the equations as written – I haven’t changed them in any way !!!!

            Young and Freedman states in chapter 1.4

            “An equation must always be dimensionally consistent. You cant add apples and automobiles; two terms may be added or equated only if they have the same units.”

            Ed Bo says – “Excel calculations dont care about units.”

            Obviously neither does he !

            I have programmed many equations into computer programs and it was always MY responsibility to ensure I COMPLIED with “An equation must always be dimensionally consistent.”

            The computer or Excel doesn’t care – or even “know” about units – it will generate any sort of gibberish you tell it to !

            BUT if units ARE NOT consistent THEN the expressions written ARE NOT equations.

            1. The 2 temperature calculation expressions as written are NOT “equations”.

            2. The results of these calculations are used as input variables into all the rest of the equations and because of this NONE of them are “equations” either !

            This is indisputable for these equations AS WRITTEN and no amount of “qualitative hand waving with words” on your part changes this simple reality !

          • Ed Bo says:

            Rosco:

            Add a cell “C5” to the spreadsheet with a label in “A5” that states “Depth of water (m)”. Put a value of 1.0 in cell C5.

            Now add a “/$c$5” to the end of the equation in each cell of Columns B and E in the main table, starting with the 2nd row (Row 11 of the spreadsheet).

            Now your nitpicking concern is gone, and guess what — the result is exactly the same!!!

            If you increase the depth value to 2.0 meters, the time of the transitory response doubles, but the steady-state solution is the same.

            Dr. Spencer is obviously used to students who are capable enough to understand that a “divide by 1” operation is not necessary to do explicitly.

            The fact that you could not understand this when it was explicitly pointed out to you does not argue well for you.

          • Rosco says:

            Thanks Ed – I just wanted to get you to say that explicitly.

            So we have a cube of water – 1 metre x 1 metre x 1 metre with a heat capacity of 4184000 J per deg. per cu. metre.

            Somehow we multiply this whatever it is ? by one of its sides and transform the units to 4184000 J per deg. per square metre.

            I think this is simply contrived to give the result you want.

            But let’s leave that to one side for the moment and consider this.

            On August 2 2013 I was sent the almost identical spreadsheet explaining the Steel Greenhouse Effect – the one with a steel sphere and shell radiating against each other in deep zero K space.

            There is no metre depth of water in this case, the sphere and shell capacity are the same 4180000 J m-3 K-1 and both spreadsheets produce the same results. Dimensional analysis of the Steel Greenhouse spreadsheet shows the result of Kelvin + Kelvin.Metre.

            I find the results contrived in both cases.

            As usual I accept we will never agree so that is my final word on this topic.

          • Ed Bo says:

            Rosco:

            Huh? You no longer have the excuse of a plausible misunderstanding, missing one step in Dr. Spencer’s logic.

            I showed explicitly how to make the units come out right, and you still can’t get it.

            The 4814000 J / m^3 / K is right out of science and engineering tables for water (although usually given as 4.184 kJ / kg / K) — if you had any actual experience in working with these values you would have seen that immediately.

            What on earth is contrived about this? This is very standard introductory undergraduate material?

  2. Wim Rost says:

    I suppose anything that delays transmittance of energy direction space would have had an effect of rising temperatures in the atmosphere and at the surface. Blankets delay loosing energy and so we ourselves (or the Earth’surface) warm below the blanket. A blanket is enough to get the effect of real warming.

    • jerry l krause says:

      Hi Wim,

      “Blankets delay loosing energy and so we ourselves (or the Earthsurface) warm below the blanket.”

      You are the heat source and your surface temperature might be a little below 98.6F and with a blanket you, not the blanket will warm your surface temperature up to 98.6 and if you put on too heavy a blanket will begin to overheat as your internals continue produce waste heat involved in your body consuming fuel to drive the body functions which keep you alive.

      The Earth’s surface has no significant internal source of energy which heats its surface as it, the surface, begins to cool even before sunset.

      Have a good day, Jerry

      • Wim Rost says:

        Good morning Jerry, thank you!
        “The Earths surface has no significant internal source of energy which heats its surface as it, the surface, begins to cool even before sunset.”

        Energy (from the sun, radiation) makes molecules move more: their temperature rises. During nighttime soils are loosing part of the daytime heat. 70% Of the Earth’ surface is ocean. The heat content of water is enormous if compared to atmosphere and soils. So seventy percent of the surface can easily continue emitting during the night and in the same time advect energy to other places 24/7, by wind and currents. Al that [land and ocean] heat content is the surface’ significant internal source of energy used for emission. Also during the night.

    • David Appell says:

      Wim Rost says:
      “I suppose anything that delays transmittance of energy direction space would have had an effect of rising temperatures in the atmosphere and at the surface.”

      Greenhouse gases don’t delay the transmittance of outgoing energy, they redirect some of it.

      • Nabil Swedan says:

        But this is what is observed, reduction trend of the outgoing radiation to outer space, Loeb et al. (2016). If radiation to outer space decreases, then surface warms up.

      • Wim Rost says:

        Greenhouse gases dont delay the transmittance of outgoing energy, they redirect some of it.

        David, when a cloud receives radiation, its temperature rises. Water molecules which are brought in a more energetic state – even when it is only during a split second – will collide with air molecules that dont loose energy that quickly. The air molecules are heated up by the (temporary) more energetic water molecules. Clouds water molecules themselves will heat up as well, the temperature of the whole cloud goes up. Loosing that heat takes time. That is the delay the proces of loosing heat to space gets.

        This process happens everywhere where there is water vapour and water (drops, ice) in the air. Where there are more water molecules, the air column is heated up more than elsewhere. This will result in (extra) convection.

      • David Appell says:

        Nabil: Yes, there’s a reduction of radiation to space, but not because it’s “delayed.” It’s because some of it gets redirected downward.

      • coturnix says:

        On a microscopic level it does redirect it, but macroscopically what is observed as a result of it is the effective slowing-down of the energy flow to the outer space. It is a different way to word things. It redirects radiation, it slow down/screens net energy flow.

    • Ross Handsaker says:

      If you wish to cool off under the blanket, just put your foot out to the side of the bed and let the warm air escape through convection.

  3. Tim Folkerts says:

    I looked through the spreadsheet — it all looks good.

    When I first saw that “wiggle” in the surface temperature graph I thought there must be something wrong, but no, that is the right solution. As the atmosphere warms, it gives a brief boost to the rate the surface warms.

    It can be fun to try other weird things, like set the initial temperature to 400 K and then watch things cool back toward the same final results.

    (I might nitpick the units listed for the heat capacity, but that doesn’t affect the value of the spreadsheet).

  4. don penman says:

    I would think of it like this for what it is worth first there is the earths surface which if there was no atmosphere would cool to space and if we took temperature measurements at regular intervals during the night we would see an exponential cooling curve as the surface cooled less as it got cooler if we did the same with the atmosphere we would get the same cooling curve during the night but if we add them together then we will have the same cooling curve but it will be shifted to the right because there is more energy. The surface cooling curve may be less steep than the atmosphere cooling curve for reasons stated by the greenhouse effect but where are we on the cooling curve on the steep part or the flat part. How warm are we relative to space?

  5. Kristian says:

    Spencer,

    True: If you somehow managed to turn the atmosphere into a completely non-participating medium (you wouldn’t, though), then there would be no apparent atmospheric “DWLWIR” to the surface, no matter how hot the atmosphere itself happened to be.

    However, you would get the EXACT same result if the atmosphere had an effective absorp tivity/emissivity equal to 1, but a temperature no higher than space (2.7K). The surface would then still receive a ‘flux’ from its surroundings worth of 0.000003 W/m^2, effectively zero.

    IOW, the effect of the “DWLWIR” on the surface radiant heat loss is indisputable, but its final temperature effect on the surface as a result of this isn’t. As long as the magnitude of the “DWLWIR” is itself a direct radiative effect of the final temperature.

    It appears pretty circular arguing that without the apparent 345 W/m^2 down from the atmosphere to the surface, the surface would cool considerably, because then it couldn’t radiate 398 W/m^2, as if the 345 W/m^2 worth of “DWLWIR” somehow directly and all by itself CAUSED the surface to warm by 57K, from 232K (from a radiative equilibrium with the Sun alone) to 289K. After all, in purely radiative terms, it makes no difference to the surface whether the atmosphere radiates at 2.7K or at 279.3K. The surface radiative heat loss could easily be 53 W/m^2 in both cases, only at a T_avg of 175K in the former case (53W/m^2 – 0W/m^2) and 289K in the latter (398W/m^2 – 345W/m^2). So what is it, then, that enables the temperature to rise from 175 to 289K? Or even above 0 K? Naturally, it’s the ability of the surface and of the atmosphere to absorb energy transferred to them as heat, to ‘thermalise’ it before it escapes back out again, and to thus store it up within in the form of molecular kinetic energy. That’s how matter acquires ‘temperature’ …

    IOW, the 345 W/m^2 apparent DWLWIR from the atmosphere is merely a radiative EFFECT of what happens to be the steady-state atmospheric temperature. It’s not what you would call a (partial) CAUSE of the surface temperature. It reduces the surface radiant heat loss, yes, but we cannot tell how this alone affects the surface temperature. Because there are other (NON-radiative) heat losses at play as well … How is the TOTAL surface heat loss affected?

    You need BOTH mass AND radiative properties to enable the atmosphere to create a thermal effect on the solar-heated surface. It is just as plainly obvious that you need the atmosphere to have a TEMPERATURE as you need it to RADIATE in order for it to be able to make a difference to the surface heat budget (and thus potentially its temperature).

    However, you DON’T need radiative properties for the atmosphere to warm. You only need radiative properties for the atmosphere to cool. But you’re right, you do need radiative properties to maintain stable circulation within the bulk atmosphere. Once you have stable atmospheric circulation established, though, then you cannot change sfc T_avg by simply changing the “degree” of atmospheric IR activity.

    Empirical observations from the real Earth system verify this point, that making the atmosphere *more* radiatively active (that is, from one positive (non-zero) level to a more positive one, NOT from the zero level to a non-zero one) does not thereby make the surface T_avg any higher.

    All the same, most people simply seem to assume it will, almost by default. Based on simplified radiative theory and models alone. Like MODTRAN. Which is strange indeed, considering how easily available the empirical evidence actually is.

    Here’s the “Net Total Flux” (net SW in (radiant heat gain/Q_in) minus net LW out (radiant heat loss/Q_out)) for two tropical/subtropical regions in Africa, one semi-arid and one humid (CERES EBAF Ed2.8 sfc).

    Firstly, the Sahara-Sahel (semi-arid, 20-14N, 15W-36E):
    https://okulaer.files.wordpress.com/2016/08/net-flux-sahara-sahel-sfc.png

    Secondly, the Congo (humid, 5N-6S, 10-27E):
    https://okulaer.files.wordpress.com/2016/08/net-flux-the-congo-sfc.png

    It is evident from these plots that the surface net radiant heat (Q_rad in – Q_rad out) is *much* more positive in the humid Congo than in the semi-arid Sahara-Sahel. In fact, the Congo has a net radiant heat surplus over that of the Sahara-Sahel of about 50 W/m^2 (or +65%) at the surface.

    • Kristian says:

      Furthermore, we know from CERES EBAF Ed2.8 ToA data that the radiative imbalance (SW in minus LW out) at the top of the atmosphere is vastly different for the two regions in question: While the Sahara-Sahel imbalance is negative by 12.3 W/m^2 (more radiant heat OUT than IN), the Congo imbalance is strongly positive, by 62.7 W/m^2 (more radiant heat IN than OUT), a total difference in favour of the Congo of [12.3+62.7=] 75 W/m^2!

      We also know that the Congo on average absorbs just as much solar heat (‘net SW’) at the surface as the Sahara-Sahel:
      https://okulaer.files.wordpress.com/2016/08/congo-net-sw-sfc.png
      https://okulaer.files.wordpress.com/2016/08/sahara-sahel-net-sw-sfc.png

      So how come the surface T_avg in the Congo is STILL lower than in the Sahara-Sahel by several degrees (~2.5K, adjusted for altitude) …!?

      This SO obviously contradicts any general assumption (the “MODTRAN logic”) that if you only increase the atmospheric “DWLWIR” to the surface, thus reducing the surface (and the ToA) radiant heat loss (‘net LW’), then the surface T_avg has to go up, as if by physical necessity.

      It is NOT a physical necessity! Real-world observations tell us it isn’t.

      What part of the total system is it, then, once the atmosphere has become radiatively active, that will routinely baffle (as in ‘negate’) any further RADIATIVE attempt at producing extra warming? Yup: ‘Atmospheric circulation.’

    • still twisting yourself into a pretzel I see, Kristian.

      At least you now recognize that the atmosphere must be able to cool radiatively.

      So, now go try to appreciate that anything that emits IR must also absorb IR (Kirchoffs Law). It’s a consequence of energy conservation.

      • Massimo PORZIO says:

        Hi Dr. Spencer,

        I use to agee with Kristian here.

        Why don’t you consider the continuous exchange of energy at the surface interface due to the kinetic energy needed to keep the whole atmosphere up there?

        Isn’t that energy continuously transferred to the atmospheric molecules by conduction and wholly returned back by gravity in the same way?

        What is the difference between the energy-work-energy conversion needed for keeping the whole atmosphere up and the energy-work-energy conversion needed to bend the CO2 molecules?

        Except for the fact that in the first case the whole energy is returned back to the ground and in the second one only half is returned back, I don’t see any difference between them.

        The buoyancy of air parcels is only a formalism needed to easily analyze the atmospheric dynamics. There are empty spaces between molecules instead. Empty spaces that must be maintained against gravity to avoid the atmospheric collapse to the ground, the only “driver” I see for that work is the surface temperature.

        If back radiation warms the ground (which I would agree), why shouldn’t the continuous molecular fall down due to gravity don’t warm the ground?

        Note that I’m not telling that the gravity force is warming the ground, is the molecular vertical run up and down that does it. Exactly as it happen when we throw up a rock and it fall back to the ground, its KE is finally returned to the ground as heat (let me ignore any deformation to the ground that could involve structural tensions).

        Where am I wrong?

        Have a great day.

        Massimo

        • Ed Bo says:

          Massimo:

          The earth does not need to expend any energy to “keep the atmosphere up” any more than your chair needs to expend any energy to “keep your body up”. Does your chair require a power source to hold you up?

          • Massimo PORZIO says:

            Hi Ed.

            You missed one detail:
            my chair is a solid not a gas!

            Have a great day.

            Massimo

          • Ed Bo says:

            Massimo:

            The earth’s surface is a solid (or liquid), not a gas! I maintain my point!

            And a transparent atmosphere (no GHGs) could not lose any energy to space, so the surface would not need to expend any energy to maintain the atmosphere’s temperature.

          • Massimo PORZIO says:

            Hi Ed.

            Really?!?

            So gases are not affected by gravity?

            Do you really believe that the molecules stand there up in the sky against the gravity force without any energy flow?
            No matter if the net flow is zero, but the flow must be there otherwise all the sky falls down, and it’s not Chicken Little’s physics, it’s Newton’s one.

            I can’t remember where I found it, but a long time ago I read that Maxwell himself computed the statistical maximum height a molecule of gas can reach for a known ground temperature.
            The value for 0K was zero meters of course (atmosphere collapsed).
            This means that the temperature at ground is the source of energy that keeps the molecule up there indeed.

            I repeat, the buoyancy of parcels is a good formalism for the analyses of the slow motion dynamics of the atmosphere, and I’m not here arguing that it doesn’t work in that context, but the air parcels don’t exist indeed. There are just myriads of molecules that would fall down every moment that are pushed up again by other myriads of molecules below them that are fired up by the ground temperature.

            Have a nice day.

            Massimo

          • Ed Bo says:

            Massimo:

            What is your point? You admit the net energy exchange between ground and the bottom of the atmosphere is zero.

            The net radiative energy exchange between surface and atmosphere is generally not zero.

          • Massimo PORZIO says:

            Hi Ed.

            My point is that even if the net exchange is zero, it doesn’t mean that there is no energy (heat) fluxes at the ground interface (in my opinion there is indeed).

            So, if the 50% back radiation is eligible for adding temperature by GHGe to the simple SB calculated one, why shouldn’t that 100% “back-heat” eligible too? (allow me the neologism).

            That’s my fundamental question, do you get it?

            Take care that I’m just an electronic engineer. So, I could be plain wrong, but I would like someone explain me why I should be wrong.

            Have a great day.

            Massimo

          • Ed Bo says:

            Massimo:

            You must distinguish between momentum transfer across a boundary, which if the boundary is inside the atmosphere means there is wind, and [kinetic] energy transfer across a boundary, which creates heat transfer.

            Which are you asking about?

          • Massimo PORZIO says:

            Hi Ed,
            I’m a little confused by your last message.

            I was writing about the KE exchange at the surface/atmosphere interface.

            My point is that the net exchange is zero at steady state, but there is a continuous energy flow in both the directions (in and out from the surface), that keeps the atmosphere up as a function of the current ground temperature.

            So, why is not accounted that 100% back energy for compute the about +30K offset respect to the simply SB computed surface temperature, while the 50% back radiation is accounted for it?

            Where is the difference?
            The heat flow is simply completely returned back, while the radiative flow is not, IMHO it’s anyways energy in movement.
            Don’t you agree?

            I apologize, I’m Italian and I know that I’ve lot of problem with my English.

            Have a great day.

            Massimo

          • No, the KE exchange at the surface isn’t zero. There is a net wind-induced torque on the Earth at the surface, much larger in mountainous regions, including friction-induced dissipation of the kinetic energy of the wind at the surface.

          • Massimo PORZIO says:

            Hi Dr.Spencer,

            I’m still arguing about a theoretical underlying process in a idealized GHGs free atmosphere.

            I’m just not able to realize why the GHGe is accounted for the abt +30K ground temperature offset respect to the SB computed temperature, because returns 50% of the ground radiation.
            While the zero net (or almost zero, it doesn’t matter in this context), heat exchange at ground interface that keeps up the atmosphere seems to be not accounted in that offset.

            As far you know, someone ever accounted for this?

            Thank you for considering this message.

            Have a great day.

            Massimo

        • Rosco says:

          Ed Bo says –

          “And a transparent atmosphere (no GHGs) could not lose any energy to space, so the surface would not need to expend any energy to maintain the atmospheres temperature.”

          If this is true – it must be as Ed says so – then exactly how does the 99% of the atmosphere that isn’t GHGs ever lose energy to space ?

          Imagine the atmospheric temperature above the Lut desert where the surface temperature has been recorded at over 70.7 degrees C during the day despite an almost total lack of water vapour !

          Unless you believe that 99% of the atmosphere NEVER absorbs thermal energy and hence never changes temperature THEN Ed Bo is CATEGORICALLY STATING greenhouse gases COOL the otherwise basically IR and shortwave transparent atmosphere !

          Who in their right mind actually believes that non greenhouse gases DO NOT absorb thermal energy ?

          EVEN IF, despite the fact it is basically impossible, only IR emitted by the surface transfers ALL heated surface energy to GHGs which then transfer it by collisions with the vastly greater numbers of non GHG molecules there is still energy transfer to non GHGs !

          Ed Bo is advocating that O2, N2 and Ar are heat trapping gases – “a transparent atmosphere (no GHGs) could not lose any energy to space” – the very definition of GHGs.

          • Rosco says:

            Meant to add – IF 99% of the atmosphere has to rely on an average 2 – 3 % of water vapour, 0.04% C)2 and tiny traces of other IR active gases to emit energy to space then no wonder the atmosphere retains thermodynamic energy !

            98+% of O2 and N2 at temperatures up to 45 degrees C over a substantial proportion of the globe completely unable to ever cool down !

            Less than a few percent of molecular collisions will be with GHGs – no wonder it was the hottest year evah !

            You are kidding aren’t you.

            GHGs trap “heat” yet the rest of the transparent atmosphere – which only a fool would claim does not absorb thermal energy as -remember – a transparent atmosphere (no GHGs) could not lose any energy to space .

          • Tim Folkerts says:

            “If this is true it must be as Ed says so then exactly how does the 99% of the atmosphere that isnt GHGs ever lose energy to space ?”

            1) The GHG moloecules lose energy to space and cool off.
            2) The nearby non-GHG molecules collide with the GHG moleclues, causing the cool GHG molecules to warm a little and causing the warm non-GHG molecules to cool a little.
            3) Go back to step 1.

          • Ed Bo says:

            (Having trouble posting, so am splitting this up.)

            Rosco:

            You ask: “exactly how does the 99% of the atmosphere that isnt GHGs ever lose energy to space ?”

            It cools by transferring KE to the 1% through collisions (conduction) and the 1% can transfer energy to space by radiation (but keep in mind that the 1% is just as likely to transfer energy downward).

            You ask: “Who in their right mind actually believes that non greenhouse gases DO NOT absorb thermal energy ? EVEN IF, despite the fact it is basically impossible, only IR emitted by the surface transfers ALL heated surface energy to GHGs which then transfer it by collisions with the vastly greater numbers of non GHG molecules there is still energy transfer to non GHGs !”

            The non-GHGs do not absorb energy from IR directly, but they do absorb energy from the GHGs that have absorbed this energy through collisions (conduction). A GHG excited by absorbing an IR photon is a million times more like to transfer this energy to another molecule through a collision than it is to re-radiate a photon before a collision.

          • Ed Bo says:

            (Won’t let me post 2nd half — will have to try later)

      • Ross Handsaker says:

        “…anything that emits IR must also absorb IR…” Does this apply to oxygen and nitrogen which comprise 99% of the atmosphere?

        • Ed Bo says:

          Both O2 and N2 barely absorb any IR, and barely emit any IR.

          • Wim Rost says:

            Ed Bo says:
            August 30, 2016 at 11:47 PM
            Both O2 and N2 barely absorb any IR, and barely emit any IR.

            WR: Ed Bo, do you know HOW MUCH O2 and N2 absorb in real Earth conditions if compared to H2O and CO2?

          • Mike Flynn says:

            Ed Bo,

            CO2 absorbs about 1000 times more IR than O2/N2. However, CO2 is only about 4 parts per 10000.

            So 4 parts of CO2 absorb about 4000 units of IR.

            The other 9960 parts of O2/N2 absorb about 10,000 units.

            CO2 absorbs about 40% of the IR in a sample of dry air.

            Please let me know if you have any facts to the contrary.

            Cheers.

          • MikeB says:

            Oxygen and nitrogen do not absorb nor emit in the mid-infrared or far-infrared. Nothing at all, zero, zilch. They do not emit 1000 times less that CO2 (where did that silly claim come from?) They emit absolutely nothing, zero. And this is the region where the Earth is emitting its infrared radiation. So oxygen and nitrogen play no part in the greenhouse effect.

          • MikeB says:

            See
            http://vplapps.astro.washington.edu/vplselectmicro.php

            ….this is an interactive tool to display the spectra of various gases at various wavelengths

          • David Appell says:

            N2 and O2 do not absorb IR, except for a brief time when their molecules collide.

            IR absorbing molecules all have three or more atoms — it’s the vibrational and rotational energy states whose transitions are in the IR part of the spectrum.

          • Rosco says:

            Ed Bo says –

            “Both O2 and N2 barely absorb any IR, and barely emit any IR.”

            OK so what do both O2 and N2 emit when their molecules are at 35 degrees C ?

          • Ball4 says:

            MikeB: your link is showing plots of line intensity, this is not irradiance from the ideal Planck curve for O2. Nor is it measured extinction coefficient of each species.

            Plug into the Planck law a wavelength you know like 10micron, and a temperature you know like 288K and find an ideal non-zero irradiance of O2 (or any object). Do it across the spectrum and find the Planck curve at a T.

            Then use the measured mass extinction coefficient for each O2,N2, the change in total pressure, the mass mixing ratio of the species to find the change in optical depth of an atm. over that pressure change in hydrostatic equilibrium. This will take finding a good ref., probably a visit to the library, but you sound up to it.

          • Ed Bo says:

            Rosco:

            You ask: “OK so what do both O2 and N2 emit when their molecules are at 35 degrees C ?”

            Virtually nothing! They can transfer energy to and from molecules that can emit and absorb significantly at this temperature.

            This is well-understood spectroscopy, demonstrated in repeatable laboratory experiments for the last century or so.

        • yes, it does…they both emit a very tiny amount of IR, and so they must absorb a tiny amount of IR.
          Just read up on Kirchoffs Law.

          • alphagruis says:

            No Mike Flynn is wrong, Quadrupolar N2 emission is not 1000 times smaller than dipolar CO2 emission but 1000 ^2 = 1000 000 times smaller. The factor is (wavelength/Bohr radius)^2. The square is because we’re talking about electromagnetic energy not field amplitude.

          • alphagruis says:

            MikeB

            Oxygen and nitrogen do not absorb nor emit in the mid-infrared or far-infrared. Nothing at all, zero, zilch. They do not emit 1000 times less that CO2 (where did that silly claim come from?) They emit absolutely nothing, zero.

            Wrong and one may ask where did your silly claim come from, instead. Which University taught you that kind of physics ?

            Of course N2 emits or absorbs IR at its rotation vibration frequencies !

            Its just that for symmetry reasons the always much stronger electric dipole interaction with radiation field is forbidden. What’s left is then the higher order quadrupolar (and magnetic dipole for O2) interaction term that is typically a factor ( Bohr radius / wavelength) = 10^4 smaller.

            From a simple physical point of view vibrating CO2 (except symmetric stretching mode) behaves as a tiny oscillating dipole antenna because of the differently polarized atoms whereas symmetric N2 in vibration can only behave like a tiny oscillating quadrupole i.e. two opposite oscillating dipoles formed on both sides of the symmetry plane of the molecule by an N nucleus and its distorted electron cloud towards the molecular bond direction.
            If both dipoles were located exactly at the same place there would be strictly zero emission as their respective fields would cancel. Yet they are not and so there is a very small but non zero residual emission.

          • MikeB says:

            OK, lphagruis, it is necessary to read what I said. I said that O2 and N2 do not emit in the mid-infrared or far-infrared; not that they didnt emit anywhere in the spectrum..

            I also gave a reference t an interactive online tool.

            http://vplapps.astro.washington.edu/vplselectmicro.php

            Try it out. Neither gas, oxygen or nitrogen, emits (or absorbs – Kirchoffs Law) at wavelengths greater than 8 microns. Oxygen will of course emit in the visible and in the microwave region, but that is irrelevant to the greenhouse effect.

            Notice also, oxygen emission in the near-infrared (7 microns) is millions of times weaker that CO2.

            Use the tool, try it out and learn something.

          • Wim Rost says:

            I read the discussion, but because I am not skilled in these things, I cannot count what I have to count. So I want to repeat my question from before:
            HOW MUCH [energy] O2 and N2 absorb in real Earth conditions if compared to H2O and CO2?

            Anyone who can give the exact answer?

          • the total absorption by O2 and N2 over all wavelengths is so small that it can be ignored…it probably falls in the error bounds on estimates of absorption by water vapor.

            In physics computations it is useful to know what things can be safely ignored, even when they are non-zero.

            Such as relativistic effects when driving a car.

          • alphagruis says:

            Well MikeB the N2 vibration mode is at about 4 micrometers as the (non dipolar allowed) symmetric stretching mode of C02, actually.

            Not sure what you meant by mid-infrared, as IR begins at 0.8 micrometers !

            Admittedly soil or ocean don’t emit much at these shorter wavelengths at 300 K

          • alphagruis says:

            Well MikeB the N2 vibration mode is at about 4 micrometers as the (non dipolar allowed) symmetric stretching mode of C02, actually.

            In fact there was an error and should have read;:

            Well MikeB the N2 vibration mode is at about 4 micrometers as is the strongest dipole allowed asymmetric stretching mode of C02, actually !!!

            Moreover O2 also emits at similar levels (10^10 times smaller than above strongest CO2 mode) in the 6-7 micrometer range !!!

            So your statement that N2 and O2 emit “absolutely nothing all all, zero zilch” in the very range of CO2 emission is utterly wrong, as I pointed out initially.

      • Kristian says:

        Roy W. Spencer, Ph. D. says, August 30, 2016 at 3:48 PM:

        still twisting yourself into a pretzel I see, Kristian.

        No, just trying to make you appreciate how your way of viewing this subject is way too simplified and WAAAY too stuck on the radiation. You don’t wanna. I get that. Is that why you go out of your way here to ‘prove’ just ONE part of a thermal effect? You’re missing a bigger picture, Spencer …

        Mars, Venus, Titan. Humid tropic vs. arid tropics. OLR over the last 30+ years.

        The atmosphere functions as an insulating layer on the solar-heated surface because it is warmer (and heavier) than space.

        The radiation is but a tool in all this. Not itself the cause. The “DWLWIR” is an apparent temperature EFFECT, not a temperature CAUSE. The atmosphere’s MASS is the cause. The hammer vs. the man wielding it.

        At least you now recognize that the atmosphere must be able to cool radiatively.

        Say what!? I’ve specifically been pointing this out all along. I think you might have me confused with someone else.

        So, now go try to appreciate that anything that emits IR must also absorb IR (Kirchoffs Law).

        I do. You seem not to have read at all what I write.

        • if you admit that DWIR exists, then we don’t have any fundamental disagreement. I agree DWIR can be thought of as an “effect”, because the energy it contains is originally from the sun.

          But it DOES exist.

          And it DOES increase the surface temperature of the Earth, as the model demonstrates.

          And atmospheric “MASS” does not create temperature. There has to be energy input from something to give mass a temperature. Statements like your’s about “mass” causing things suggest to me you have never taken a basic physics course.

          Where is your quantitative time-dependent model that produces, based upon well-established physics equations, terrestrial temperatures when initialized from any temperature you like, as I have done here?

          • Kristian says:

            Roy W. Spencer, Ph. D. says, August 31, 2016 at 8:24 AM:

            if you admit that DWIR exists, then we dont have any fundamental disagreement.

            Our fundamental disagreement, Spencer, is that you think the atmospheric thermal effect is a radiative effect, that the atmosphere simply warms the surface some more by radiating down on it, while I say the atmospheric thermal effect on the solar-heated surface ultimately arises as a result of the atmosphere’s mass. The physical mass of the atmosphere allows it to warm beyond the background ‘temperature’ of space, and it allows it to weigh down on the surface, something the vacuum of space could never do. Strongly limiting the total surface heat loss at any given temperature. That’s how it insulates the surface.

            You seem unwilling to admit, even to consider, this simple circumstance.

            I agree DWIR can be thought of as an effect, because the energy it contains is originally from the sun.

            No. It’s a simple apparent radiative effect of atmospheric TEMPERATURE. If the atmosphere were able to absorb IR, but incapable of thermalising it and holding on to it in order to actually WARM from it, then its radiative properties wouldn’t matter at all.

            But it DOES exist.

            Mathematically it exists. As an APPARENT radiative effect of atmospheric temperature it exists. But it does NOT exist as a distinct macroscopic radiant power flux (W/m^2) to the surface, thermodynamically independent of the “UWLWIR” from the surface, and rather equivalent to the solar flux, that you can treat as a regular ‘heat flux’ (without ever *calling* it that, of course) whenever it suits your story. That is, that you can place on the radiative INPUT side of the surface budget to somehow “cause” the surface temperature associated with its radiative OUTPUT (the “UWLWIR”). Use the “DWLWIR” term all you want. But bear in mind that it will always be an integrated part of the surface radiant heat LOSS (DWLWIR minus UWLWIR). What it does is reduce the surface radiant heat LOSS. And that’s it. It DOESN’T increase the surface energy GAIN. Mathematically, yes. Physically, thermodynamically? No.

            Stick with the “reduced radiant heat loss” thing, and you’ll be fine. “Extra heating by back radiation”, however, is nonsense physics, and will ONLY create and perpetuate confusion. I think that most of your (and the IPCC’s) critics object to what you’re saying because you tend to portray – intentionally or not – the “GHE” as an “extra heating by back radiation” effect, NOT as a “less radiant cooling” effect. The two are fundamentally different, thermodynamically.

            And it DOES increase the surface temperature of the Earth, as the model demonstrates.

            No, it doesn’t. It enables the surface of the Earth to equilibrate at a higher T_avg than the lunar surface, but the reason why it ends up higher is because the atmosphere has a mass, and thus is able to i) warm (space isn’t), and to ii) weigh down on the surface (space doesn’t).

            And atmospheric MASS does not create temperature.

            Spencer, IR radiation also does not “create temperature”. But atmospheric mass allows the atmosphere to gain a temperature, from absorbing energy, thermalising it and holding on to it. And it also makes sure the atmosphere (as long as it’s subjected to gravity) has a weight, a pressure and a density, and a resistance to change.

            There has to be energy input from something to give mass a temperature.

            Er, yes. Just like there has to be an energy input from something to make the atmosphere radiate.

            Statements like yours about mass causing things suggest to me you have never taken a basic physics course.

            That’s because you ‘misunderstands’ my statements about atmospheric mass, Spencer. You think I’m just another one of those nutcases who think that mass (or matter, rather) itself creates energy whenever it’s compressed. I’m not. You should rather read what I actually write.

            Where is your quantitative time-dependent model that produces, based upon well-established physics equations, terrestrial temperatures when initialized from any temperature you like, as I have done here?

            So how would you go about determining the surface T_avg of Earth without knowing it (or any other altitude-specific temps) already? You’re only back-engineering. You know of course that a simple pyrgeometer couldn’t possibly derive the “DWLWIR” if it didn’t first know the temperature of its own sensor …

          • Ball4 says:

            All mass radiates Kristian. Any mass radiates at all temperatures and at all frequencies all the time, that cannot be a difference between you and anyone promoting a radiative effect, they are joined at the hip. Untwist your pretzel, use Occam’s razor to make your points readable.

          • Ball4 says:

            “You know of course that a simple pyrgeometer couldnt possibly derive the DWLWIR if it didnt first know the temperature of its own sensor ”

            Just like a mercury thermometer could not possibly derive the temperature if it didn’t first know the expansion of a pool of mercury.

    • Norman says:

      Kristian

      I have been doing research on your Sahara-Sahel vs Congo argument.

      I believe (I could be wrong in my understanding of your words, it seems I get them wrong from what you intended) you are making the claim that the Congo receives more Net radiation than the Sahara yet is cooler so that would appear to negate a radiative GHE.

      I think what you have with your locations is more complex than just radiation alone. I believe Toneb explained it to you on another thread.

      Here is the situation:
      http://www.kxii.com/content/news/Hadley-Cell-Helps-Along-our-Summer-Heat-387365211.html

      The Equator has a high amount of convection going on (all the clouds and rain) rising air. This removes a large amount of heat from the surface of that location while at 30 N you have this air sinking. I believe Toneb said it creates a high pressure cap that limits convection at these locations so they get much hotter.

      If you look the air above the Sahara is sinking on average.

      That is why you can’t disprove a GHE by using such examples because other large factors are in play.

      Also (I could be wrong so correct me if that is the case).

      http://www.bitsofscience.org/wordpress-3.0.1/wordpress/images/2015/09/climate-system-general-circulation-jet-streams.jpg

      From this link it looks like the Equator heating much move large amounts of air up against the gravity field. This process will require work so energy that could have been used in warming the air temperature is instead used as work no heat so it could suppress the temperatures in that fashion.

      Roy Spence has a PHD in meteorology, I believe he would know what is going on with your observations.

      I think it is the Hadley cell that explains what is going on with the temperatures and it does not mean water vapor is cooling the surface. If you had no water vapor the circulation would still remove more energy from the Equator’s surface than at the location the air sinks.

      • Kristian says:

        Norman says, August 30, 2016 at 10:21 PM:

        I believe (…) you are making the claim that the Congo receives more Net radiation than the Sahara yet is cooler so that would appear to negate a radiative GHE.

        I’m not CLAIMING this, Norman. I am SHOWING it. I’m POINTING IT OUT for people to see. I’m not making this stuff up. It’s right there in the available data.

        The Equator has a high amount of convection going on (all the clouds and rain) rising air. This removes a large amount of heat from the surface of that location while at 30 N you have this air sinking. I believe Toneb said it creates a high pressure cap that limits convection at these locations so they get much hotter.

        If you look the air above the Sahara is sinking on average.

        That is why you cant disprove a GHE by using such examples because other large factors are in play.

        Norman, it appears you confuse what happens in the REAL WORLD with what the “radiative GHE” hypothesis postulates about warming mechanisms.

        I’m specifically pointing out how real-world NON-radiative processes within the Earth system readily override the purely radiative ones when it comes to changes in surface T_avg. Are you telling me you don’t see this?

        You’re only confirming that what I point out is correct, Norman.

        • Norman says:

          Kristian

          The point I am trying to make in my posts to you is that yes the other heat removing processes are important for specific regional differences but both the Congo and Sahara would be much colder than without GHE going on. It is true the convection and evaporation losses from the surface are much higher at the equator than other locations, but if you removed the 405 W/m^2 DWIR the region would be cooler.

          I no longer believe the GHE is of a 33 C value. I look at the way they calculate it and they use a high albedo for Earth. Without the clouds and ice the Earth would not have this high of albedo, If you removed water from its surface so it was more like a lunar surface you would have an albedo closer to 0.12 instead of the 0.3 they use.

          I am not sure we disagree on much except radiant energy fluxes.

          • Ball4 says:

            Norman, the natural 33C is the difference between Earth brightness temperature Te 255K and the thermometer field Ts 288K. It changes a bit with small changes in each of those observations.

            The 33C difference can also be calculated with atm. emissivity 0.8 and 0.0 as in the top post analysis. That would be with current albedo 0.3. Any other albedo changes the balance and the 33C will then become something else based on the new choice of albedo.

          • Kristian says:

            Norman says, August 31, 2016 at 5:26 AM:

            The point I am trying to make in my posts to you is that yes the other heat removing processes are important for specific regional differences but both the Congo and Sahara would be much colder than without GHE going on.

            I won’t let you get away this easily, Norman. You have specifically stated that you disagree with my argument that the DEGREE of atmospheric IR activity doesn’t matter to surface T_avg, once the atmosphere is radiatively active (at some, at any level above zero) and atmospheric ciculation thus becomes operative and stable. This whole excercise (the Sahara-Sahel vs. the Congo) is to show you – through simple empirical observations – that I’m right and you’re wrong. I could also point to other celestial bodies in the solar system, like Mars, Venus and Titan, to get a global view, and thus show you the exact same thing. Also, I could refer you to OLR data over the last 30+ years from Earth showing how there has been absolutely NO “enhancement” of any “radiative GHE” as defined, despite the significant increase over the same period in the atmospheric content of both CO2 and water vapour. In fact, I already have. But you tend to ignore these things and just go on with your bubble ideas of how the world SHOULD work.

            It is true the convection and evaporation losses from the surface are much higher at the equator than other locations, but if you removed the 405 W/m^2 DWIR the region would be cooler.

            Norman, the 405 W/m^2 of apparent DWLWIR in the equatorial region are a radiative EFFECT of the equatorial air being as warm as it is. It doesn’t thereby tell you what CAUSED that temperature to be as high as it is. You refuse to get this simple argument. I don’t know why. A child would.

            The only relevant radiative variables in the surface budget is the INCOMING radiant heat (‘net SW’, from the Sun) and the OUTGOING radiant heat (‘net LW’, from the surface). All you can say is that the apparent DWLWIR from the atmosphere to the surface reduces the outgoing radiant heat from the surface *more* in the Congo than in the Sahara-Sahel. And yet this DOESN’T make the surface T_avg in the former any higher than in the latter region. In fact, the OPPOSITE is true.

            I am not sure we disagree on much except radiant energy fluxes.

            I fear we still have some way to go. You need to open up, acknowledge and understand different perspectives from your own on certain things. Like how an atmosphere’s bulk mass works and is necessary to warm a planetary surface.

            It is hopeless discussing subjects such as this with people who absolutely refuse to concede even the simplest and most obvious points, who refuse to even TRY to understand what their opponents are actually saying, who basically just ignore it, summarily dismiss it (without even reading), or conveniently ‘misunderstand’ it, while at the same time simply rehashing over and over again the same old talking points, as if their opponents hadn’t heard them a thousand times already …

          • Norman says:

            Ball4

            The change in Albedo is my point. The Moon’s albedo is 0.12. Mars is listed as 0.25 Earth at 0.31.

            A lot of the Earth’s higher albedo is because of the effects of water vapor, clouds reflecting sunlight, snow and ice which have high albedo’s.

            I did research and the ocean basins are mostly basalt like the moon. If the Earth had no GHG it could have no water so it would be mostly basalt with higher emissivity sand in some regions but the bulk would be dark absorbing basalt. The albedo would almost certainly below 0.31 and hence the equilibrium temperature with a higher solar flux would be more.

            I do not understand why they would keep the albedo the same if they removed GHG from the system (water vapor which needs water to exist). I think they should try and figure out a more correct albedo without any GHG in the system and then see what type of equilibrium temperature such an Earth would attain. Almost certainly above the 255 K figure so far used to demonstrate our GHE.

          • Norman says:

            Kristian

            It seems you post very similar things to many different posters including Dr. Spencer. Your standing complaint is people misunderstand you.

            Here is but one sample: YOU: “It is hopeless discussing subjects such as this with people who absolutely refuse to concede even the simplest and most obvious points, who refuse to even TRY to understand what their opponents are actually saying, who basically just ignore it, summarily dismiss it (without even reading), or conveniently misunderstand it, while at the same time simply rehashing over and over again the same old talking points, as if their opponents hadnt heard them a thousand times already ”

            You must not be making yourself clear enough. It would not just be me so the flaw in communication would be on your end.

            Here is one very confusing statement you make:
            ME: “It is true the convection and evaporation losses from the surface are much higher at the equator than other locations, but if you removed the 405 W/m^2 DWIR the region would be cooler.

            YOU: “Norman, the 405 W/m^2 of apparent DWLWIR in the equatorial region are a radiative EFFECT of the equatorial air being as warm as it is. It doesnt thereby tell you what CAUSED that temperature to be as high as it is. You refuse to get this simple argument. I dont know why. A child would.

            I do not see a logical connection or bridge between the two statements. My statement was that if you removed the DWIR the area would be cooler than with it….then you talk about it is an effect of the air being warmer? What sense does that make to my statement. I really cannot follow the flow of your arguments and how they connect. I think others fail at it as well.

            If you had no GHG the mass of the atmosphere would help not at all in keeping the surface warm. If it had 5 times its current mass how will it stop radiation from leaving the surface and moving on to outer space…energy lost forever.

            That is why I can’t follow your thought process. My statement is about removing the DWIR from the system and then you come up with that the DWIR is an effect, what does that have to do with my point?

            Remove DWIR from the Sahara and what do you get? Look up at Roy’s simulation. It will show you. Even if you do not use two-way IR fluxes just the one, LW leaving Earth’s surface. With a radiating atmosphere the energy leaving the Earth’s surface NET is far less than it would be without DWIR.

            Now are you making a claim in your statements that the concentrations of GHG is not important? That is what it seems like to me but I can’t be sure since you will tell me I am not understanding what you are saying. Can you answer that one? You don’t have to but it would be nice so I can try to understand what you are attempting to say.

          • Ball4 says:

            “I do not understand why they would keep the albedo the same..”

            Simply so it matches today’s Te 255K brightness temperature. Any other albedo results in different Te.

            The moon surface is blasted to smithereens and Earth’s with atm. at 1bar is not, much larger particle size. The albedo of the moon is affected by the small particle size, 50% is 10-100 micron diameter powder which is in the IR range. About 10-20% of the powder is around 10micron diameter right in the wavelength of interest so diffraction rears its ugly head and straight Planck law is no longer applicable to either surface emissivity or albedo. Regolith testing over the spectrum is necessary to find lunar albedo and surface emissivity properties.

          • Kristian says:

            Norman says, August 31, 2016 at 7:49 PM:

            It seems you post very similar things to many different posters including Dr. Spencer. Your standing complaint is people misunderstand you.

            Spencer misunderstands because he has this seeming preconception about me being one of the “mass creates energy” dolts. And so he doesn’t really bother to read what I write, at least not with particular care. Folkerts knows quite well what I’m saying, but he doesn’t really like the implications of some of the things I point out, and so he “pretends” to misunderstand, in order to avoid having to respond directly to them. Most other people who ‘misunderstand’ my argument are basically warmist trolls in different guises. I’m not sure where you fit in, though, since I don’t consider you a ‘warmist troll’.

            You must not be making yourself clear enough.

            It should be pretty clear what I’m saying, Norman. It’s not very complicated, after all. If you WANT to understand, you WILL understand. I’m sorry, but I see no sign of you actually wanting to understand what I’m saying, Norman. You seem pretty comfortable inside your bubble. As do most other people commenting here …

            Here is one very confusing statement you make:
            ME: “It is true the convection and evaporation losses from the surface are much higher at the equator than other locations, but if you removed the 405 W/m^2 DWIR the region would be cooler.

            YOU: “Norman, the 405 W/m^2 of apparent DWLWIR in the equatorial region are a radiative EFFECT of the equatorial air being as warm as it is. It doesnt thereby tell you what CAUSED that temperature to be as high as it is. You refuse to get this simple argument. I dont know why. A child would.

            I do not see a logical connection or bridge between the two statements. My statement was that if you removed the DWIR the area would be cooler than with it…then you talk about it is an effect of the air being warmer? What sense does that make to my statement. I really cannot follow the flow of your arguments and how they connect. I think others fail at it as well.

            That’s because you (apparently) didn’t read the paragraph above the one you’re quoting, where I explained how the Sahara-Sahel vs. the Congo exercise intended to demonstrate that there’s absolutely no reason to assume that you will increase the surface T_avg by simply making the atmospheric column on top of it *more* opaque to outgoing IR. IOW, you’re not arguing about the issue at hand. You’re arguing about something that we already agree on, that the surface T_avg would be significantly lower (but importantly NOT the bulk atmosphere T_avg, it would be significantly higher) if the atmosphere were basically a non-participating medium (completely IR inactive). However, even with an IR active atmosphere, it is the fact that the atmosphere is able to WARM substantially beyond space that effectively makes it an insulative layer on the solar-heated surface. The radiation is but a tool, a means to an end. And so, if the atmosphere were IR active, but had the same temperature as space (2.7K), that is, it could hold no internal KE, then its radiative properties would make no difference. The surface could then easily shed a RADIANT heat flux worth of 53 W/m^2, even at a T_avg of 175K [53-0 W/m^2], just like it does in our current case, at 289K [398-345 W/m^2].

            So it’s not the size of the “back radiation flux” itself that determines the steady-state T_avg of the surface. It’s the steady-state T_avg that determines the size of the “back radiation flux”.

            As I’ve told you multiple times now, Norman, you would benefit from stepping out of your bubble and enable yourself to see this subject from a different angle, basically from the opposite end.

            You just start with the steady-state 405 W/m^2, but you have no clue how we got to this steady state in the first place. What actually caused the temps to rise until we got there?

            You consistently see this from the point of view where the apparent 405 W/m^2 of atmospheric ‘radiant flux’ to the surface is somehow – in collaboration with the solar heat flux – what’s CAUSING the surface temperature to be as high as it is. It’s not. It’s the other way around.

            If you had no GHG the mass of the atmosphere would help not at all in keeping the surface warm. If it had 5 times its current mass how will it stop radiation from leaving the surface and moving on to outer spaceenergy lost forever.

            Stop with the kicking at open doors, Norman! We’re done with this particular part of the discussion. You have very clearly stated that you don’t agree with me on the PROGRESSIVE “GHE”, the one where you continuously increase the surface T_avg by increasing the DEGREE of atmospheric IR activity. I say (and show) there is no such connection. And THAT’S what this is all about. THAT’S what the Sahara-Sahel vs. the Congo thing is all about.

            Can you please try to address this issue from now on, Norman.

            Now are you making a claim in your statements that the concentrations of GHG is not important?

            Er, yes, Norman. That is what I am claiming. And that claim is what I back up with empirical evidence. Looking at the Congo vs. the Sahara-Sahel. Looking at the radiative fluxes at the ToA over the last 30+ years. Looking at Venus, Mars and Titan. All pointing to the same thing: The DEGREE of atmospheric IR activity doesn’t matter to sfc T_avg. Only atmospheric mass and solar input does. Because as soon as the atmosphere is IR active, atmospheric circulation becomes operative and stable.

          • Norman says:

            Kristian

            I would not say I agree with you on this point:
            YOU “Youre arguing about something that we already agree on, that the surface T_avg would be significantly lower (but importantly NOT the bulk atmosphere T_avg, it would be significantly higher) if the atmosphere were basically a non-participating medium (completely IR inactive).”

            Have I missed your explanation of why the bulk atmosphere would be warmer if the T_avg surface where cooler? Even if the atmosphere could only cool by conduction it could not reach a temperature higher than the surface since that is its only source of heat.

            On this Point: YOU: “So its not the size of the back radiation flux itself that determines the steady-state T_avg of the surface. Its the steady-state T_avg that determines the size of the back radiation flux.”

            It is both, they influence each other. Look at Roy’s simulation and you will see (as he has removed all other heat transfer mechanisms) that radiation alone can warm an absorbing atmosphere and by doing so a warmed atmosphere will in return elevate the equilibrium T_avg surface to a higher temperature than in a system without such an IR active atmosphere. Also his simulation shows that concentration of GHG is hugely significant in setting a new equilibrium temperature. Change the emissivity and you see the equilibrium temperature will drop. His 0.8 number he uses is based upon the current concentrations of GHG. If you remove the GHG the emissivity will drop and so does the equilibrium temperature. Try it, I did and it certainly does. GHG concentration is what sets the radiation equilibrium temperature.

          • Norman says:

            Kristian

            After establishing some points with your post I will move on to your Congo vs Sahara debate.

            All your study demonstrates is the complexity of H2O in the climate system and why they have such a hard time getting models to run nearly identical results.

            Water and some effects it will have on Climate Change.
            In liquid form it is a very good IR absorber and emitter, listed as around 0.95. Also a very good absorber of SW radiation with very low albedo.

            IR emissivity chart:
            http://www.optotherm.com/emiss-table.htm

            For albedo:
            https://upload.wikimedia.org/wikipedia/commons/thumb/1/18/Albedo-e_hg.svg/800px-Albedo-e_hg.svg.png

            So depending on what form it is in water can have a tremendous impact on the climate system and there is no easy way to figure out all its various effects if the system changes.

          • Norman says:

            Kristian

            A point you made on your blog:
            “Furthermore, we already know from the main post that the radiative imbalance (SW in minus LW out) at the ToA over our two regions is vastly different: While the Sahara-Sahel imbalance is negative by 12.3 W/m^2 (more radiant heat OUT than IN), the Congo imbalance is strongly positive, by 62.7 W/m^2 (more radiant heat IN than OUT), a total difference in favour of the Congo of [12.3+62.7=] 75 W/m^2!”

            So you have much more radiative energy into the Congo than the Sahara-Sahel. Yet the Congo is cooler. The next step is to go to the nonradiative heat transfer processes.

            It is as I stated water is a complex variable in a climate system because it has effects which oppose each other. Pure water vapor in the air would only lead to a warmer equilibrium temperature for the T_avg surface by radiation alone. Even as it stands it still plays a huge role in the climate system keeping the equilibrium tempearture of the surface higher than it would be without it.

            Let us look more closely at your study.

            First link. Global evapotransporation rates.
            http://www.ntsg.umt.edu/sites/ntsg.umt.edu/files/imce/GMAO_CMGalbedo_0.05deg_GEO-th.png

            If the image links correctly you can see the Congo has a much much more evapotranspiration than the Sahara.

            Link to sight with graph:
            http://www.ntsg.umt.edu/project/mod16

            Note: enough links for one post, I will continue with the next post.

          • Norman says:

            Kristian

            Continuing the line of logic. I am establishing loss of surface energy by process of evapotranspiration.

            If the graph links. Sahara-Sahel has such small evapotransiration it is listed as zero effect (might have some in rainy season but not enough to effect the yearly amount).

            On the graph the Congo is 1250-1500 mm/year.

            If you divide by 365 you get the mm/day
            3.42 mm/day- 4.11 mm/day

            Here is a link that allows you to convert this number into MJ/m^2-day which will then allow you to convert to W/m^2 since a Watt is a joule/second.

            http://www.fao.org/docrep/x0490e/x0490e04.htm

            Table 1 of this link gives you a value that 1 mm/day evapotranspiration is equivalent to 2.45 MJ/m^2-day

            Now you can use the number above for the Congo evapotranspiration rate to find the Joules/m^2-day value

            8,379,000 J/m^2-day to 10,069,500 J/m^2-day

            divide these by 24 then 3600 to Joules/sec-m^2 or W/m^2

            96.98 Watts/m^2 to 116.54 Watts/m^2

            So you have the Congo which loses energy via evapotranspiration at the rates above while the Sahara-Sahel loses none by this process.

            Above you have that the Congo has a net positive of 75 W/m^2 over the Sahara using just radiation balance.

            So if you add the evapotranspiration loss to the positive radiation balance of the Congo you end up with a net energy surface loss (compared to the Sahara) of

            -21.98 W/m^2 to -41.54 W/m^2

            So is it any wonder that the Congo ends up being cooler than the Sahara? But without the Backradiation of both places they would both be much cooler than they are now.

            Water is both a cooling and warming effect. The GHE is still stronger than the other effects. The GHG in the Congo send 405 W/m^2 back to the surface without this energy hitting the surface the T_avg surface would be much cooler.

          • Norman says:

            Kristain

            Conclusion

            Your Congo vs Sahara study in no way demonstrates that GHE is not significant for keeping the surface temperature at a higher equilibrium temperature. It also does not prove your backward thinking that GHE is an effect not a cause.

            Reality suggests it is both. The DWIR allows the surface to reach a higher equilibrium temperature under the condition of a constant incoming flux of energy. The higher equilibrium temperature of the surface allows a higher equilibrium temperature of the atmosphere to be maintained (as seen in Roy’s simulation if you play with it a bit). It is both a cause and effect.

            Nothing in your posts has proven the GHE is wrong in the current description, nor have you come close to proving that the amount of GHG in the atmosphere is not crucial to determining GHE. I will agree you have to have some atmospheric mass to hold energy so that the GHG have a supply of internal energy that allows a continuous emission of energy.

            I am not sure if these posts will convince you of anything. I think I did demonstrate you Congo-Sahara conclusions are not valid in proving your points.

          • Norman says:

            Kristain

            Points from your blog:

            “This leads us to the conclusion that, while there is no reason to believe that the outgoing LW effect of having H2O in the atmosphere will raise the temperature, the combined (reflecting/absorbing) H2O effect on incoming solar (both SW and LW) is definitely going to cool the surface.

            This conclusion appears to be quite solidly backed up by this posts (admittedly quick and superficial) empirical analysis. More H2O in the atmospheric column will make the surface cooler. There are no empirical observations from the real Earth system supporting the notion of a net radiative surface warming effect of having H2O in the atmosphere above. The net effect is most certainly cooling”

            I disagree with your conclusions and your statements. You have a 405 DWIR in the wet area. If you changed nothing else but removed it from they system the temperature would plummet. H2O definitely has warming effect. It also has cooling effects as well.

            Also Sahara’s DWIR of 375 W/m^2 is still 92$ of the Congo’s 405 W/m^2 DWIR. Both have significant DWIR that is greatly reducing the surface radiant energy loss.

          • Kristian says:

            Norman,

            It seems you really need to have this fed to you with the tiniest of spoons.

            Let’s go back to our hypothetical planet. In the initial steady state, there is no atmosphere on top of its global surface, and so the global surface has simply equilibrated with the average radiant heat input (ASR, net SW) from the planet’s mother star, meaning, its average radiant heat output (OLR, net LW) is equal to it. In our particular case, let’s now say that the average ASR value is 296 W/m^2, and so, in the steady state, this is also the average OLR value: 296 W/m^2 IN = 296 W/m^2 OUT. This state is ideally attained at the point where the planet’s global surface T_avg has reached ~269 K. (This, of course, relies specifically on two purely hypothetical conditions to abide: The surface is 1) a blackbody, and 2) isothermal.)

            We now place a massive – and very much radiatively active! – atmosphere on top of this equilibrated planetary surface. Now, just as the surface before it could be considered to start its original journey toward its original steady state temperature (269 K) from a hypothetical initial temperature around absolute zero (or 2.7 K, rather), so could the atmosphere. We simply want to see what happens as energy accumulates inside this massive atmosphere, gradually warming it.

            So to begin with, before any energy has managed to be transferred as heat from the surface to this new atmosphere, the surface radiant heat loss is: q/A = σ (T_sfc^4 – T_atm^4) -> σ (269^4 – 2.7^4) -> 296 W/m^2, IOW just what it was before the atmosphere was placed on top of it. And remember now, this is a fully radiatively active atmosphere – it is able to absorb and emit EM radiation. The only problem is that it’s still too cold.

            But what happens as this atmosphere now absorbs more and more energy from the surface (and from the local sun), thermalising it and gradually warming from it? Its temperature rises beyond that of space itself. And as a simple consequence of this, the atmosphere is now turned into an insulative layer, basically interposing a thermal barrier between the solar-heated surface and the absolute coldness of space.

            As the atmosphere warms, its apparent DWLWIR to the surface increases. But the atmosphere warms, not from apparent, thermally generated ‘radiant fluxes’, but from the absorbed and thermalised energy transferred to it as heat from the surface and directly from the sun. And what happens when the surrounding temperature of a constantly heated object/surface all of a sudden increases? The temperature difference between the object/surface and its surroundings is reduced. And what does this lead to? It leads to a reduction in the rate of “heat loss” from the object/surface in question. This is true whether the mode of heat transfer happens to be ‘radiative’, ‘conductive’ or ‘convective’. And so, if we assume that the rate of incoming heat to the object/surface remains unchanged, then energy will accumulate (because Q_in > Q_out) and the object/surface will necessarily warm as a result, until its heat balance is restored (Q_in = Q_out). And what happens when the surface temperature rises as a consequence of this process? Its apparent UWLWIR increases. Which makes the (radiant) heat loss rate (DWLWIR minus UWLWIR) of the surface go up again. And so the surface radiant heat loss rate itself could theoretically stay constant during the entire warming process towards the new steady state temperature. It would be 296 W/m^2 in the initial state (296W/m^2 – 0W/m^2, 269 vs. 2.7 K), and it could be 296 W/m^2 in the final (steady) state also (say 398W/m^2 – 102W/m^2, 289 vs. 206 K).

            The thing, though, is that once the massive atmosphere is put on top of the solar-heated surface, then the whole surface situation changes. Moving towards the new steady state, the atmosphere will 1) make the planet’s albedo increase substantially, and 2) absorb for itself a significant portion of the incoming heat input from the sun, so that it never manages to reach the actual surface at the bottom. This will reduce the average ASR at the surface from the original no-atmo value of 296 W/m^2 to a final +atmo value of a mere 165 W/m^2. Which would mean that about 44 % (!) of the original solar heat absorbed by the global surface is now somehow made unavailable to it by the very presence of the radiatively active atmosphere resting on top of it, either reflecting it back out to space or absorbing it for itself, before it could ever reach the surface.

            This situation alone would reduce the potential steady-state surface radiant heat loss from 296 to a maximum of 165 W/m^2. But would it thereby necessarily change the surface T_avg also? No. Because the heat loss (the net LW) is only constrained by the temperature DIFFERENCE between the surface and the atmosphere, not by the individual temperature of the surface and/or the atmosphere. So we could go from 296 minus 0 W/m^2 (net LW: 296 W/m^2) in the initial steady state (at 269 vs. 2.7 K), to 296 minus a potential 131 W/m^2 (net LW: 165 W/m^2) in the final steady state (at 269 vs. 219 K).

            However, it doesn’t end there. You see, more things change at the surface once the massive atmosphere is put on top of the solar-heated surface than just the total heat balance (moving from Q_in(296W/m^2)=Q_out(296W/m^2) to Q_in(165W/m^2)=Q_out(165W/m^2)). The surface heat budget after all also stops being a purely radiative one. And this fact is an extremely important fact to appreciate, because it has obvious implications for the surface radiant heat loss, which used to make up 100 % of the total. It won’t anymore. It will naturally have to make room for other losses, NON-radiative ones. IOW: It will, by physical necessity, become significantly smaller.

            And so this really changes the whole narrative. The surface radiant heat loss ISN’T reduced because of a rise in the effective atmospheric temperature leading to a rise in apparent atmospheric DWLWIR to the surface. It is simply reduced because there is less ‘need’ for it, because other heat loss mechanisms than the radiative one are also now contributing to the total.

            So you see, the absolute magnitude of the surface radiant heat loss is thoroughly constrained first by the heat INPUT to the surface (the ASR), then by the (effectiveness of the) other heat loss mechanisms at work. It can’t be determined simply according to some perceived atmospheric level of IR opacity.

            In the end, we might have a situation where, after having emplaced a massive atmosphere around our hypothetical planet, the surface steady state corresponds to a Q_in = Q_out of only 165=165 W/m^2 (rather than one of 296=296 W/m^2), where the radiant part is further reduced to, say, 53 W/m^2. And so we’re left with the following inescapable apparent DWLWIR-UWLWIR relationship: 398 – 345 W/m^2 (net LW: 53 W/m^2), at 289 (T_sfc) vs. 279 K (T_atm). But this doesn’t tell us anything about how we got from a surface T_avg of 269 to one of 289 K. All it tells us is that 1) the surface steady-state T_avg happens to be 289 K, and that 2) the surface steady-state radiant heat loss happens to be 53 W/m^2. 289 K -> 398 W/m^2, and 398 – 53 = 345 W/m^2. This is exactly how a pyrgeometer would compute the apparent “sky radiation” (DWLWIR). But it tells us nothing about why the radiant heat loss happens to be 53 W/m^w, nor why the surface T_avg happens to be 289 K. Remember how, when we first placed the atmosphere on top of the solar-heated surface, the heat input to the surface from the sun was 296 W/m^2, and there were no other heat loss mechanisms in operation except the radiative one. Moreover, the no-(or pre-)atmo steady state surface temperature was 269 K (296W/m^2 IN = 296W/m^2 OUT), and this was also the initial situation as the massive, radiatively active atmosphere (at 2.7 K) was placed around the planet – the DWLWIR was practically zero, because the atmosphere was too cold. Then several things happened: i) the atmosphere started warming (from absorbing energy transferred to it as heat), ii) the heat input to the surface was reduced, and iii) other heat loss mechanisms besides the radiative one became available and operative as the system still grew warmer (a result of the warming atmosphere). And so, from the initial to the final steady state, we went from a surface net LW (radiant heat loss) shedding 296 W/m^2 to one at a mere 53 W/m^2. Meaning, we went from a temperature difference of [269-2.7=] ~266 K between the surface and the APPARENT “effective atmospheric level of downward radiation” to one of [289-279=] 10 degrees. Does this mean that the atmosphere somehow got immensely more opaque to surface IR, lowering the effective level of “sky radiation” to the surface by this huge amount, during the journey from t_i to t_f? No, of course it doesn’t. Here’s what happened in between: i) the atmosphere got warmer, meaning, the temperature difference between the surface and the layers of air above it grew steadily smaller; ii) the overall heat input to the surface from the sun, the ASR (net SW), grew steadily smaller, and so the surface target output value naturally decreased with it; iii) the radiative share of the total surface heat output dropped from its initial 100 %, because of NON-radiative heat loss mechanisms growing to prominence …

            To conclude:
            1)
            The increase in DWLWIR is an apparent radiative effect of the atmosphere warming. When the atmosphere warms beyond space, from absorbing, thermalising and storing up energy transferred to it as heat from the surface and directly from the sun, the temperature difference between the surface and its surroundings decreases, and so its total heat loss is naturally reduced. This forces the surface to warm so as to restore its heat balance (the heat input is assumed constant). Yes, if the atmosphere in question is completely radiatively inert, it (or the main portion of it) will eventually be thermodynamically disconnected from the surface, and so space can still be considered the surface’s only significant cold reservoir. But as soon as you make the atmosphere radiatively active, you will connect it thermally with the surface, and so now the atmospheric temperature will directly affect the total heat loss from the surface, which will be greatly reduced at any given surface T_avg relative to the former situation, once the atmospheric T_avg rises above the ‘temperature’ of space. You might fool yourself into thinking that it is in fact the increase in the apparent atmospheric DWLWIR itself that forces the surface temperature to rise in this situation. But it’s not. The absolute TEMPERATURE rise is the CAUSE. The DWLWIR is but a TOOL enabling the atmospheric temperature to connect with the surface temperature in the first place. When the atmospheric temperature rises in this situation, it will then simply be able to affect the surface temperature, because now the atmosphere is thermodynamically connected with the surface, and so it effectively replaces space as the surface’s thermal surroundings, meaning that, as it warms, the temperature difference between the surface and its surroundings goes down (this couldn’t happen with only the vacuum of space around, since a vacuum cannot warm). And as the temperature difference between the surface and its surroundings goes down, there will be a decrease in both radiant, conductive and evaporative/convective heat loss from the surface. Forcing a surface temperature rise.

            The increase in DWLWIR is simply one expression of this decrease in temperature difference between the surface and its surroundings, thus of the reduction in surface heat loss.

            2)
            The steady-state magnitude of the surface radiant heat loss is to a certain extent simply a function of the steady-state magnitude of the non-radiant heat losses (conduction and evaporation/convection), which will inevitably start ‘eating into’ the radiant portion of the total as soon as a massive atmosphere is in place on top of a solar-heated planetary surface. At some point along the continuum – from 100 to 0 % – a balance will be struck. It simply depends on how effective the radiant vs. the non-radiant losses are at ridding the surface of energy at some particular temperature. This will vary from place to place on the same planet, as it will from planet to planet. The relationship between this steady-state ratio of heat losses and the steady-state surface temperature, though, is apparently not a straightforward (linear) one. The total heat loss from the surface in the Sahara-Sahel and in the Congo are about the same, but the radiant portion of the total is much larger in the Sahara-Sahel (~100 out of 175 W/m^2) than it is in the Congo (~50 out of 175 W/m^2). This circumstance, however, doesn’t translate at all into a lower surface T_avg in the former region. You can’t just say that a region with a more effective (larger) surface radiant heat loss will necessarily end up having a lower T_avg than one where the radiant heat loss is much smaller. In fact, the surface T_avg is higher in the Sahara-Sahel than in the Congo by a significant amount.

            So how come the radiant heat loss in the Congo is only ~50 W/m^2 and around twice that in the Sahara-Sahel? Atmospheric humidity and clouds. Yes. But also evaporative heat loss and deep moist convection. The two are intimately connected.

            * * *

            I will also post this at the bottom of the comment thread.

        • David Appell says:

          Norman wrote:
          “I do not understand why they would keep the albedo the same if they removed GHG from the system (water vapor which needs water to exist)….Almost certainly above the 255 K figure so far used to demonstrate our GHE.”

          I think everyone understand this. The calculation you’re referring to, that gives the 255 K, is just a simple zero-dimensional energy balance model. The resulting temperature, 255 K, is just for that simple model. The 30 C number for the greenhouse effect is just a simple estimate.

          People who learn this basic model should be taught this, and not to take it as the final say. It’s just an approximation, just a heuristic demonstration of energy balance.

          However, you might be interested in this paper by Lacis et al, where they took all noncondensing GHGs out of the atmosphere and ran a climate model to see the resulting cooling. They get about 35 C.

          http://pubs.giss.nasa.gov/abs/la09300d.html
          http://pubs.giss.nasa.gov/docs/2010/2010_Lacis_la09300d.pdf

          • Norman says:

            David Appell

            The item in the article you linked to I am not sure of. The reason the globe cools so much is because the water supposedly turns to ice increasing albedo and a bunch more clouds form. But remember clouds are the strongest of all GHE agents with a full spectrum of IR radiating in all directions so although they would cut back on solar input, they would also act to minimize radiation loss. But the other point is why would cloud cover go up? Does the cold Antarctic or Arctic have more clouds as the temperature drops?

            This article claims cloud cover actually warms a winter Arctic.

            “Clouds play a central role in regulating the energy balance of
            the Earth, and because they are so heterogeneous, regionally
            they help determine the character of weather and climate. At
            the Arctic surface they tend to be warming most of the year,
            shifting the surface heat budget up by 10 to 50 W m−2
            from
            autumn until spring, while in summer they tend to be slightly
            cooling the surface”

            From this article:
            http://pubman.mpdl.mpg.de/pubman/item/escidoc:1517160:8/component/escidoc:1539280/acp-12-6667-2012.pdf

            Also there are a lot less clouds in winter than summer in the Arctic so I am not sure why the model they are using increased the cloud cover so much (which shouldn’t lead to cooling anyway overall since half the globe is dark).

            Also this: “Previous studies based on passive sensors and human
            observers found a total cloud cover of up to 90-95 percent
            in summer months and values around 50 percent in
            winter, with sharp transition seasons in April and October
            (Huschke, 1969; Schweiger and Key, 1992; Eastman and
            Warren, 2010).”

          • David Appell says:

            Norman: Why don’t you write a rebuttal letter to SCIENCE and its editors, refuting Lacis et al., telling professional scientistgs they don’t know about clouds, instead of just posting whatever few thoughts come to you after a 5-minute glance of the paper?

          • David Appell says:

            From the paper:

            “The results, summarized in Fig. 2, show unequivocally
            that the radiative forcing by noncondensing
            GHGs is essential to sustain the atmospheric
            temperatures that are needed for significant levels
            of water vapor and cloud feedback. Without this
            noncondensable GHG forcing, the physics of this
            model send the climate of Earth plunging rapidly
            and irrevocably to an icebound state, though perhaps
            not to total ocean freezeover”

          • Norman says:

            David Appell

            I thin this is the strong difference you and I would have in our approach to Climate Science. You will not question the output of a model but others will.

            They are not really “my thoughts”. When I read that cloud cover would increase in a much colder world (and considerably according to the model) I did a Google search on cloud covert in the Arctic and it showed that as it cools clouds are reduced not increased. So I have strong questions about the Climate model presented.

            Here is an older Roy Spencer blog post.
            http://www.drroyspencer.com/2015/11/models-vs-observations-plotting-a-conspiracy/

            I look at the output of climate models and they are all over the place. They really are not a hard science, too many variables at play. Why would I accept this climate model as a “truth” because it appears in a science publication. I read those all the time in Scientific American. They are what they are. A possibility but in this one it goes against observed patterns so I question it a little more.

          • Norman says:

            David Appell

            Forgive my typing skills, they seem to get worse as I age.

            “thin” = “think”

            “cloud covert” = “cloud cover”

            I should start rereading posts before submitting them and make corrections then. Sorry for the slop I see it a lot in my posts and will have to slow down a bit.

      • yes, you have explained it pretty well, Norman. Downwelling IR is just one portion of the surface energy budget, and as you say, energy export from the moist, deep-convective zones to the subsidence zones must also be taken into account.

  6. doctor no says:

    Another excellent post by Roy.
    Readers here should know that, in general, this topic is covered in first-year university courses on climate science. You may be interested in enrolling?

    • Ed Bo says:

      For those who have a distrust of climate science, basic engineering thermodynamics and heat transfer courses should suffice. Learn to define control volumes and control masses rigorously, then keep careful track of energy transfers into and out of these.

      Most of the confusions shown here would be alleviated by learning how to do these basic tasks properly.

      • doctor no says:

        The problem with engineers is that they are not scientists – no matter how hard they try.
        Stick to building bridges.

        • Ed Bo says:

          It is commonly said that the steam engine did more for the science than the science did for the steam engine!

          Most of the thermodynamics, heat transfer, and fluid dynamics knowledge used in climate science was taken from engineering knowledge…

          • doctor no says:

            Stick to building steam engines then.

          • Toneb says:

            Yes, it is *interesting* how many “engineers” make their misconceptions of what happens in climate physics known on various (usual suspects) Blogs.
            PS: I studied engineering to degree level before joining the UKMO.

          • Ed Bo says:

            Most of the people posting ridiculous nonsense here have science backgrounds, not engineering…

        • crakar24 says:

          Dr No,

          The general rule of thumb is an architect is a civil engineer who failed the maths subjects.

          A mechanical engineer builds the bombs and the civil engineer builds the targets.

          cheers

        • Rosco says:

          Thank God for that – Climate Scientist designed structures would be failing all over the globe.

  7. David in ct says:

    The chair resists collapse because it is solid and it’s hard to deform.
    The atmosphere is not a solid and yes of course if the individual molecules lost their kinetic energy they would collapse back to the surface.
    maybe a high school physics text would be helpful …

    • Massimo PORZIO says:

      Thanks David,
      I responded to Ed above just because I didn’t read your right message here before.

      Have a great day.

      Massimo

    • Ed Bo says:

      David:

      The earth’s surface resists collapse also — it is not a gas either! Without GHGs, the atmosphere cannot lose any energy to space, so the surface does not need to transfer any energy to the atmosphere (net) to maintain its temperature.

      • Massimo PORZIO says:

        Hi Ed.

        Do you really believe that once heated by the ground the GHGs free atmosphere can’t be cooled by the same ground which is in contact?

        Note that, if you admit that by night that atmosphere cools anyways, you must admit that that atmosphere is continuously exchanging energy at the ground interface.

        Have a nice day.

        Massimo

  8. DH MacKenzie says:

    Dr. No
    “A good scientist is a person with original ideas. A good engineer is a person who makes a design that works with as few original ideas as possible. There are no prima donnas in engineering.”
    Freeman Dyson

    • Nabil Swedan says:

      There are exception to any rule.

      • Mike Flynn says:

        Nabil,

        /humour on

        It’s not a rule if you find an exception.

        /humour off

        Cheers.

        • Nabil Swedan says:

          I do not want to go off topic, but what you are saying that the profession makes a person who he is. I disagree with this. People who seek the truth are born with this trait, they are true scientists. Look at the history, we have had priests with no formal education and brought to the world some of the greatest ideas.

          • Lewis says:

            Nabil,
            I have a saying about the psychology of a similar issue.

            I like to be right.
            He needs to be right.

            You see much of that here.

  9. Nabil Swedan says:

    Dr. Spencer,

    The time for atmospheric air to reach steady state from absolute zero is exactly 365 days, or one revolution of the earth around the sun. This number of days may be used to cross check or “calibrate” the model.

  10. Mike Flynn says:

    With respect, the surface has never been absolute zero – anywhere.

    Absorbed solar at night is zero.

    Maybe the model should average day conditions, model average night conditions, and average the resultant averages.

    Over the last four and a half billion years, the result should agree with the observation that the surface has cooled.

    When the atmosphere absorbs energy, it gets hotter. When it emits energy, it gets colder. If the surface is losing energy faster than the atmosphere, a low level inversion follows. This occurs even though the atmosphere is radiating more intensely than the surface, as it has a higher temperature.

    Tyndall gave a good explanation of the process more than 150 years ago. I assume this is part of university level meteorology courses, but may not be obvious to non meteorologists.

    Cheers.

    • “the surface has never been absolute zero anywhere”

      OMG, Mike. The straw men just keep getting erected, don’t they?

      You can initialize the model at any temperature you like, 0 K, 100 K, 1000 K…the final resulting equilibrium temperature will still be the same.

      Seriously, dude, such objections are just a waste of everyone’s time.

    • David Appell says:

      Mike Flynn says:
      “Over the last four and a half billion years, the result should agree with the observation that the surface has cooled.”

      It hasn’t cooled over the last 150 years.

      So what’s wrong with your theory?

  11. TheColdSun says:

    I didn’t think Dr Spencer would put his foot in it again quite so soon when I said my goodbye’s, so it seems I have to add just this one brief comment.

    Kristian is right – regions with more radiation from water vapor are cooler, as data from many regions confirms. You can’t expect water vapor to warm the surface and at the same time make the temperature gradient less steep, now can you?

    As Kristian also says, radiation is not always thermalized. You can read why in the paper in which you will find this on page 167:

    “a blackbody thus acts in a very simple way: it absorbs all radiation, emits absorbed frequencies below cut-off, and uses absorbed frequencies above cut-off to increase its temperature.”

    The re-emission of “frequencies below cut-off” is the “pseudo scattering” we are telling you about.

    See: http://www.csc.kth.se/~cgjoh/blackbodyslayer.pdf

    Kristian has also explained that the surface temperature is a function of atmospheric mass. Strictly speaking I now realize that it is in fact due to the force of gravity acting on the mass of the atmosphere and creating a stable temperature gradient, so this 2003 paper in the journal “Energy and Environment” is close to the mark, but not spot-on I admit:

    http://ruby.fgcu.edu/courses/twimberley/EnviroPhilo/FunctionOfMass.pdf

    • Toneb says:

      “Kristian is right regions with more radiation from water vapor are cooler, as data from many regions confirms. You cant expect water vapor to warm the surface and at the same time make the temperature gradient less steep, now can you?”

      Err, you can my friend if you understood meteorology…..
      And you plainly don’t.
      Lumping one *effect* to make the whole Earth work by it’s principles, shows staggering ignorance of the science.
      I’m afraid to say you are not alone on here.

      • Toneb says:

        “Kristian has also explained that the surface temperature is a function of atmospheric mass. Strictly speaking I now realize that it is in fact due to the force of gravity acting on the mass of the atmosphere and creating a stable temperature gradient”

        Gravity does not create the LR.
        In the atmosphere it is the movement of air up/down that does – modified by moist convection.
        This is achieved via the addition of heat to the Earth’s surface.
        Without that the atmosphere would become isothermal to the tropopause.
        That happens in winter over Antarctica.

        Gravity is not magic. It performs work on the atmosphere (air) just as your bicycle pump does when inflating a tyre. It compresses it and warms it. But it doesn’t forever stay warm – neither does gravity when compressing the atmosphere. It is a “one-shot event”.

    • Kristian says:

      TheColdSun says, August 31, 2016 at 12:50 AM:

      As Kristian also says, radiation is not always thermalized.

      That’s not what I said.

      Kristian has also explained that the surface temperature is a function of atmospheric mass.

      I absolutely have not. This is YOUR idea. Not mine.

      Strictly speaking I now realize that it is in fact due to the force of gravity acting on the mass of the atmosphere and creating a stable temperature gradient (…)

      No, it isn’t.

  12. Bryan says:

    Roy says

    “The radiative fluxes use the Stefan-Boltzmann equation (sigma T^^4),”

    This is fine for your present simplified model

    However the pyrgeometer uses the same equation.
    The pyrgeometer is the only officially recognised instrument for measuring downwelling LW radiation.(Nabil Swedan )
    I have some concerns about that.

    Consider the simplest case i.e. at nighttime.

    The SB equation is acceptable for the surface up radiation which has a continuous spectrum

    But the atmosphere down radiation spectrum is not continuous.
    The ‘atmospheric window’ is good evidence that it is band or line spectrum.

    So how can the SB equation give realistic results?

    Some commentators say that really the equation used is known as Schwarzschilds Equation applied to each wavelength line by line.

    However the pyrgeometer is not calibrated with the Schwarzschild Equation.

    My main concern is that there seems to be a mismatch between the results of your month by 30 year satellite survey showing no accelerating global temperature and a simplistic expectation of such acceleration based on the CO2 atmospheric greenhouse theory.

    The use of such a crude instrument as the pyrgeometer to base major worldwide economic changes seems more like climate religion than climate science.
    Kristian’s points about a simplistic overemphasis of the radiative contribution also seems justified.

    • MikeB says:

      You are right that the atmosphere does not radiate like a blackbody across a continuous spectrum and, if you want to infer the temperature, you need to make some assumptions.

      But the pyrgeometer is measuring the total energy coming down and needs to make no assumptions on temperature to determine the extent of back-radiation.

      I do not know about the pyrgeometer being the only officially recognised instrument for measuring downwelling LW radiation; it sounds like nonsense, just like its source.

      You can also measure back-radiation with a Fourier transform infrared spectrometer ( a much more expensive piece of equipment).This provides a spectral analysis of the down-welling radiation in terms of wavelength or frequency.

      The following result is an example.

      https://scienceofdoom.files.wordpress.com/2010/04/longwave-downward-radiation-surface-evans.png

      To understand the significance of this result it is necessary to know that CO2 emits radiation around a wavelength of 15 microns. We can see from the measured result that this where most of the back-radiation cones from. This is the unmistakable fingerprint of CO2 as a major component of back-radiation

      Notice that there is no back-radiation from oxygen or nitrogen

      • Bryan says:

        Mike B

        It seems that Nabil Swedan might be correct

        https://www.pmodwrc.ch/pmod.php?topic=irc

        You say

        “But the pyrgeometer is measuring the total energy coming down and needs to make no assumptions on temperature to determine the extent of back-radiation.”

        If you look at the user handbook of for example

        http://www.kippzonen.com/Download/32/CG-3-Manual

        On page 33 it appears they are using atmospheric temperature and the SB equation.

        It’s a pity that there seems to be no way of independently verifying pyrgeometer readings.
        There is concern that the very large readings of downwelling flux may be overestimated.
        This then combined by several other ’rounding up’ opportunities can cause undue alarm and despondency.
        This in turn persuades governments to phase out fossil fuels at great cost and inconvenience.

        • The Eppley PIR instrument is the most widely used one for measuring downwelling IR from the sky.

          My understanding is that DWIR cannot be measured directly, but as a residual. The instrument measures the NET IR flux, which impacts the surface temperature of a thermopile. They compute the upwelling flux based upon the instrument’s temperature and emissivity. The downwelling flux is then the difference between the two.

          Small corrections are made for the IR-transparent dome, which is not 100% transparent…so the temperature of the dome is also monitored.

          NO assumption regarding the temperature of the atmosphere is needed.

          Over the years papers have been written about small corrections needed to the measurements, on order of 5%. But it’s hard to imagine something as large as 350 W/m2 or more being totally fictitious.

          Others can correct me if I’m wrong about any of this.

          • Nabil Swedan says:

            “My understanding is that DWIR cannot be measured directly”

            You are right Dr. Spencer, and this what pyrgeometers do: calculate DWIR.

            We are humans. Science as far as I know begins with observations and calculations then follows. In the pyrgeometer case we acted as God instead and did exactly the opposite. We assume DWIR exists then calculated it. This is not science.

          • except that if you use a cooled detector, you will get much closer to a “direct” measure of DWIR, because the detector is emitting so little of it’s own energy.

            Use liquid nitrogen, even better.

            Use liquid helium, even better still.

            And the DWIR would STILL be about the same magnitude.

            The reason it isn’t measured so directly in practice is that it’s expensive.

            And I’m sure there are many things in nuclear physics and other fields that can only be measured rather directly with expensive equipment.

            Are these things not “science” either?

          • Kristian says:

            Only, in this case you are still not detecting “back radiation” from the cool sky to the warm surface, Spencer. You are detecting a radiant HEAT flux from a cool sky to a much, much colder, cryogenically cooled, detector.

          • Sigh. Sounds like we are back to arguing over whether a cold object emits IR in the direction of a warm object, then.

            So….

            Did we ever establish if you believe that the magnitude of IR flux from a warmer object (say 300k) to a colder object (say, 250K; assume both have emiss=1) depends on the temperature of both objects?

            Can you please answer that question for me as a yes or no?

          • Ed Bo says:

            Roy:

            Here’s another thing you don’t understand. Since we only have differential pressure measurements, there is no such thing as absolute pressure.

            So if you tried to determine the supposed “absolute pressure” in your car tire by using a standard gauge, measuring the differential to the atmosphere, and adding to that value the value for a barometer (differential to [almost] vacuum), that would be completely wrong, conceptually!

            (Need I say ?)

          • Ed Bo says:

            The “slash-sarc” (sarc off) didn’t come through at the end! Sorry!

          • Kristian says:

            Roy W. Spencer, Ph. D. says, August 31, 2016 at 12:17 PM:

            Sigh. Sounds like we are back to arguing over whether a cold object emits IR in the direction of a warm object, then.

            No, Spencer. We’re back to the tired old “We know that DWLWIR exists because it has been measured directly a million times” argument. It is NOT measured directly, Spencer, and it never has been. Only temperatures and radiant HEAT fluxes are ever measured (as in ‘physically detected’) in radiative thermal processes. All else is merely assumed and computed.

            How hard is this?

            Did we ever establish if you believe that the magnitude of IR flux from a warmer object (say 300k) to a colder object (say, 250K; assume both have emiss=1) depends on the temperature of both objects?

            Can you please answer that question for me as a yes or no?

            Are you being serious? OF COURSE the temperature of both objects matter. Do you think I’m Joe Postma!?

            q/A = σ (T_h^4 – T_c^4)

            If T_c goes up and T_h remains unchanged, q/A will naturally go down.

            All I’m pointing out is that you need to keep the two temperature terms on the right together at all times. You can’t put them on either side of the equal sign. Even if that’s possible mathematically. Because physically they’re inseparable. Inside the radiative heat transfer.

          • Ball4 says:

            Kristian, “Because physically theyre inseparable. Inside the radiative heat transfer.”

            This is where Kristian’s confusion over heat results in a false conclusion by Kristian.

            q is not heat, Kristian, as it is not KE. As you agreed couple posts ago. The eqn. you write is in units of energy. What Kristian is describing is radiative energy transfer so the terms are associative (additive, 1LOT) so they can indeed be physically meaningful when moved to the opposite side of the eqn.

            What is not physically possible and, I believe, Kristian’s intent to convey, is that the two energy flows cannot be separated to do anything useful by 2LOT as they are diffuse and incoherent, universe entropy would have to decrease in the separation to be useful so that separation of the two streams is not possible.

  13. gbaikie says:

    “The heat capacity of the surface and atmosphere are assumed to be the same, equivalent to 1 meter of water for simplicity (the atmosphere is really more like 2 m of water effective heat capacity).”

    I doubt it make any difference what heat capacity, you plug in.
    But curious why is it simpler. Air is 1 KJ there is 10 tons
    of air per square meter, why is more complicated to use
    10,000 KJ per square meter- or if like a 1/10th of it- 1000 KJ
    per square meter.

    But there is far more heat capacity than 10,000 KJ per square even if you just count the atmosphere- if include the latent heat of water

    Also see no reason to start at absolute freezing, by merely being a planet prevents such temperatures.
    Why not start at 100 K?
    That way, it’s simpler as you could ignore all the latent heat connected to the atmospheric gas starting out as frozen, melting then the liquid boiling to become gas [which is really massive amount of heat needed].

    But question does it make a difference in the simple model if
    start with the heat capacity of atmosphere equal to 10 cm of water?
    Or does it change if 2 or 3 meters of water?
    Or other than shorten or lengthen the amount of days needed,
    it should not make a different in the result of the model.

    • assumed heat capacity only affects how long it takes the system to reach a constant temperature.

      You can initialize the model at any temperature you like 0K, 100K, 1000K. It still equilibrates at the same temperature.

      Try it!

      • gbaikie says:

        Oh, I didn’t know I could change the variables and I thought you said could only change the emission factor [which tried to change and it didn’t seem work for me- as I am not very adept with Excel, but also I can’t say made much effort at it].
        Anyhow, I think heat capacity, the planet’s rotation rate, axis tilt, orbital eccentricity and gravity and transmission of sunlight thru meters of ocean water are important variables.

        But to keep it simple, I also think the intensity of sunlight is
        important- and averaging it with 240 watts, ignores this.

        • gbaikie says:

          Or also to make simpler, an actual greenhouse and UHI in terms of night time temperatures are only about heat capacity.
          Or are deserts at night only about the lack of heat capacity and the lack latent heat of water vapor.
          Or contrary to common belief trees increase UHI- though they provide shade and moderate daytime high temperature in small locality- but in terms larger region and night time temperatures, they add to the average temperature.

  14. Mike Flynn says:

    Dr Spencer,

    You are correct of course. As I say, 9996 parts of 02/N2 don’t need to emit much IR to exceed the amount emitted by 4 parts of CO2.

    MikeB asked where the silly claim (that CO2 absorbs 1000 times more IR than O2/N2) comes from. John Tyndall, actually. I was a bit conservative. Tyndall’s experimental figure (at 1 psi) was 750 to 1.

    Cheers.

    • MikeB says:

      Rerference please. I certainly do not see that in any of tyndalls work. And, if I missed it, what frequency is he talking about, becaise that ids the key.

      • Norman says:

        MikeB

        I agree with you. I would also like a reference to this experiment. If you go to the Hitran source it shows N2 emits about a trillion times weaker than CO2. If I read the Y-axis correctly and the difference in exponents.

      • Mike Flynn says:

        MikeB,

        When you have reread Tyndall’s “Heat – a mode of motion” 6th edition, if you can’t find the relevant table, let me know. As to wavelength, Tyndall used a copper plate heated to about 270 C for his heat source in that particular instance. He used a variety of heat sources at different times, ranging from boiling water to carbon arc emissions.

        Setting all that aside, the atmosphere prevents roughly 30% of insolation from reaching the surface. If you choose to think that this due to CO2, nobody can stop you.

        The Earth has been cooling for four and a half billion years. Dr Spencer doesn’t like hearing it repeated either. It’s true, nevertheless. Trying to convince yourself that the Earths surface is getting hotter, day by day, due to the interaction of sunlight and CO2 might take more and more effort as the century progresses.

        More CO2 and warmer conditions would seem to create more plants. More plants mean more food. Warmists seem to be opposed to better conditions, but I can’t understand why.

        Cheers.

        • Tim Folkerts says:

          “When you have reread Tyndalls … “
          Why would you use such an old reference when new measurements with MUCH better equipment are available? That would be like using using Ole Rmer’s first measurement of the speed of light (2.2E8 m/s) rather than the modern value (2.9979E8 m/s).

          “the atmosphere prevents roughly 30% of insolation from reaching the surface. If you choose to think that this due to CO2, nobody can stop you.”
          Why would anyone choose to think this? Assuming you mean the ~ 30% after accounting for reflection, H2O absorbs the most, with both O3 and CO2 absorbing significant amounts.

          “The Earth has been cooling for four and a half billion years. “
          This is either irrelevant or it is wrong.
          * If you are talking about the core and mantle, then yes they have been cooling for 4.5 billion years. But this is irrelevant to climate.
          If you are talking about the atmosphere, you are wrong. The atmosphere has warmed and cooled and warmed and cooled many times in the last 4.5 billion years. The multiple glacial periods in the last million years are obvious examples.

    • gbaikie says:

      — Mike Flynn says:
      August 31, 2016 at 6:59 AM

      Dr Spencer,

      You are correct of course. As I say, 9996 parts of 02/N2 dont need to emit much IR to exceed the amount emitted by 4 parts of CO2.–

      I don’t think the O2, N2, or Argon emit much energy.
      But also don’t think the gases of Co2, and etc emit much energy.
      But do think the liquid droplets of mostly H20, and particles in the air would amount to very significant amount- far more than all the gases in atmosphere.

  15. Mike Flynn says:

    As a matter of interest, anyone who believes that thermal imaging devices reflect reality, might care to search for IR pictures of a Leslie cube. Manufacturers make emissivity assumptions based on averages. Temperature is not energy, and many people seem to be confused about the difference between the two.

    Cheers.

    • IR imager manufacturers allow you to adjust the assumed emissivity of the object being measured (except in the cheapest instruments, where it’s fixed, probably around 0.95).

      This is no secret, Mike.

      And, it does not affect the conclusions I have made in my posts.

      • TheColdSun says:

        Dr Spencer:

        It’s still all totally wrong I’m afraid. The Sun cannot heat the surface enough, nor can the back radiation or a combination of the two. No amount of “insulation” enables radiation to warm anything more than it could warm a blackbody in Space, and different fluxes of radiation cannot be combined, as you could easily show with a home experiment.

        I will give it one more attempt to teach you what we physicists have been discussing for at least four or five years now. It is correct what we have deduced and it is supported by empirical evidence, as all science should be.

        But firstly, I need to respond to your request on the previous post where you wrote: “And please do share with us the original NASA energy budget diagram that doesnt include downwelling IR radiation from the sky?”

        Well that’s seen in this post which is worth reading:

        http://hockeyschtick.blogspot.com.au/2010/05/nasas-earth-energy-budget-contradicts.html

        If a photon emitted by the atmosphere has the same frequency (and thus energy) as a photon that could be emitted by the warmer surface, then it will temporarily raise an electron through one or more quantum energy states in some surface molecule that is capable of emitting a photon with exactly the same energy.

        This can be considered a resonating process, and that it how the surface distinguishes between radiation from the atmosphere (which does not penetrate the oceans) and strong overhead solar radiation on a clear day in summer which can indeed heat a small portion of Earth’s surface.

        So the energy from the incident photon is temporarily stored as electron energy (not converted to thermal energy) and then the electron returns to its original state and, in so doing, emits a photon which is identical to the incident one. So the process looks like scattering and is thus called “pseudo scattering” by physicists. It slows the cooling of the surface (though only that by radiation) because that emitting molecule is occupied with the photon just received, and so does not have the capacity to emit a photon that used some of the thermal energy in the surface.

        Now the above-linked NASA energy budget diagram does indeed have balanced energy, but the solar flux into the surface is the only flux that could raise the surface temperature. The problem was that it just was nowhere near enough to explain observed temperatures, and that is reality. They thought the missing energy must be coming from back radiation, but it had to be 324W/m^2 because all the other values were fairly well locked in. But a gas with average temperatures below zero in th eatmosphere just simply could not emit that much, and whatever it does emit downwards it should also emit upwards, but it is not shown as doing so. Surely you can see there is something seriously wrong in the IPCC diagram.

        Whilst the atmospheric radiation is clearly overstated in the IPCC energy budget diagrams (and was not even shown in the NASA diagrams around 2010) it is irrelevant anyway in relation to the question as to why the surface temperature is what is observed. Only an understanding of how heat can be transferred INTO the surface by processes other than radiation will lead you to a correct understanding of the surface temperature and why it is not affected by radiation from carbon dioxide, that also being pseudo scattered.

      • TheColdSun says:

        Meteorologists know that this heat transfer into the surface can happen when there is a temperature inversion. By their own definition, an inversion occurs when the temperature gradient is less steep than normal, so the upper temperature can still be colder than the lower one.

        You need to think about that process. What it implies is that heat transfer is only upwards when the temperature gradient is steeper than the equilibrium value (= the environmental lapse rate) and this heat transfer is downwards when it is less steep.

        So entropy increases as heat transfers (via molecular collisions) occur downwards into the surface, thus warming the surface so as to correct the less-steep gradient and bring it back to the normal environmental lapse rate that is the equilibrium state with maximum entropy.

        And therein lies your answer as to why you don’t need ANY of the 324W/m^2 overstated back radiation that is pseudo scattered anyway.

        • yes, I am in fact a PhD meteorologist, and I know that there is a downward component of transfer of heat when an inversion forms, due to turbulent mixing by even small wind currents.

          But the fact that the inversion formed in the first place means there was a net UPWARD transfer of heat, by IR emission from the surface cooling at night. Mixing of air and downward transfer of sensible heat across the inversion occurs, as you say,…but it is SMALL because the inversion is still there!

          I’d like to see a published paper describing this “pseudo scattered” radiation. Sounds very mystical.

          And, again, you remain anonymous…even though it appears you authored a book? Wassup wit dat?

          • TheColdSun says:

            No, these are imperceptible downward heat transfers (via molecular collisions only) without necessarily exhibiting measureable net bulk air movement. There can be situations in the morning (for example) when the Sun can only warm the top of clouds or regions in the upper troposphere and the stratosphere. There can be situations where (soon after dawn) valleys are still in shade whilst the solar radiation may warm the tops of high mountains and thus the surrounding air. There may be only a reduction in the gradient from, say, 6 to 5 degrees per Km and it may not be noticed by meteorlogists. Where it is most apparent is in planets with high atmospheres that do not have much solar radiation reaching lower layers, and yet these lower layers still get warmed by this process and still exhibit temperature gradients based on the force of gravity divided by the mean specific heat of the gas.

            Regarding “pseudo scattering” I’ve given you a link to the paper which discusses radiation being not thermalized when it is below a certain cut off frequency and being re-emitted. Regardless of what you call the process, it happens, and the Second Law would be violated if it didn’t.

          • once again, magical.

            “Imperceptible heat transfers”, under special conditions at certain locations, and specific times of day, that suddenly remove the need for 300+ W/m2 energy flux.

            Amazing.

            So, show me the equations with numbers in a model that actually produce terrestrial temperatures using such magic.

            (By the way, when the sun rises, even when its cloudy, the surface warms. It starts even before the sun peeks above the horizon. You can measure it, as I have done. It’s because there is still scattered sunlight reaching the surface…which is why you can see outside.)

          • TheColdSun says:

            There’s nothing magical about the process of thermal heat diffusion via molecular collisions: you could observe it in your lounge room. Molecules move at around 500 meters per second and can pass on kinetic energy via collisions: you must know that.

            And no it’s not because of scattered sunlight reaching the surface, most of which would be at a small acute angle to the surface and virtually all of which would have insufficient flux to raise the existing temperature of the surface. Where the surface is 15C that flux would have to be over 500W/m^2 even if it were orthogonal to the surface. Otherwise, divide that by the sine of the angle made with the horizontal, and you might be needing over 1,000W/m^2.

            What you have described about the surface warming even under thick and extensive cloud cover is a perfect example and evidence of what I am talking about. The warming of the tops of clouds can start this process, and thermal energy passes through the cloud by molecular collision (and some radiation between water molecules) and then continues on from the base of the cloud to the surface.

            Surely you must know that the thermal conduction by air is extremely small, and the thermal collision you talk about occurs over EXTREMELY short distances…look up “mean free path”…Wikipedia gives 68 nanometers as the average for air at sea level pressure. As a result, totally still air can transfer very little heat through non-radiative processes. -Roy

          • Eli Rabett says:

            FWIW, the time it takes for an isolated vibrationally excited CO2 molecule to emit a photon is a bit more than a second (measures). The time is takes for a vibrationally excited CO2 molecule to degrade vibrational to translational energy is a few microseconds (measured). It takes about 1000 collisions (measured_.

            Very few (like one in a million) of CO2 molecules that absorb IR photons directly emit those photons.

        • Ed Bo says:

          TheColdSun:

          “pseudo scattered”? There’s only one person who advocates this silly fiction. You’ve given yourself up!

          • so, has Dou6 C0++0n shown up again with yet ANOTHER fake identity??

          • Norman says:

            Ed Bo

            Yes it is HE, the Australian is back to visit.

            This: “Theres nothing magical about the process of thermal heat diffusion via molecular collisions: you could observe it in your lounge room. Molecules move at around 500 meters per second and can pass on kinetic energy via collisions: you must know that.”

            Is his h*e*a*t*c*r*e*e*p* in a disguised fashion. He has also frequently used 500 meters/sec as the molecular velocity in past posts.

            OK, well he fooled me with the Cold Sun moniker. His Aussie IP address should have tipped me off. He skillfully avoided his usual ways of phrasing things. Well done, D0u6! -Roy

          • Eli Rabett says:

            500 m/s is not a bad estimate. OTOH the time between collisions at atm pressure is about 10^(-10) sec so the mean free path is of the order of 5x 10(-8) m or 50 nm.

  16. OleKlemsdal says:

    Thank you for your always interesting posts!
    Even if oversimplified, how do you think your Fig. 1 with energy fluxes harmonize with NASA’s presentation: “The atmosphere radiates the equivalent of 59% of incoming sunlight back to space as thermal infrared energy, or heat. Where does the atmosphere get its energy? The atmosphere directly absorbs about 23% of incoming sunlight, and the remaining energy is transferred from the Earths surface by evaporation (25%), convection (5%), and thermal infrared radiation (a net of 5-6%). The remaining thermal infrared energy from the surface (12%) passes through the atmosphere and escapes to space. (NASA illustration by Robert Simmon. Astronaut photograph ISS017-E-13859.)”
    http://earthobservatory.nasa.gov/Features/EnergyBalance/page6.php
    best regards; Ole

    • I use the widely-quoted value of absorbed solar flux of 240 W/m2 as an input. But I assume that is all absorbed at the surface, since I ignore the portion of it absorbed by the atmosphere.

      I ignore evaporation and convection.

      The thermal IR coming out the top of the model atmosphere ends up matching the assumed solar absorption, which it must in order for energy balance to be achieved.

      The assumed IR absorptivity of 0.8 is probably a little less than it is in the real atmosphere. But you can adjust that in the model yourself to see what impact it has.

  17. jimc says:

    How to assure a debate win in the new university:
    Professors tell students: Drop class if you dispute man-made climate change
    http://www.thecollegefix.com/post/28825/

  18. Climatechange4realz says:

    Hello dr spencer

    It’s good to be back. Did you take a look at professor murray Sally’s films on youtube including his latest one which reduces climate sensitivity to only 0.03C at most! This is because the emissions we emit lack carbon 14 which is a lot thicker in density then carbon 12 and 13 which is what is admitted by today’s fossil fuels. Take a look at it while you can and shoot me an email via- [email protected] whether you agree with his statements or not. All in all can’t wait for your temperature update on the UAH! Look foward to speaking with you again soon

    Sincerely

    Climatechange4realz

  19. Travis Casey says:

    Hi Dr. Spencer.

    Is it possible to estimate climate sensitivity by using the amount of energy that hits the Earth in Summer vs. Winter and compare that to the average temperature in January vs July?

    My understanding is the IPCC claims a doubling of CO2 will cause an increase of 2 wm^2, whereas the difference from Summer to Winter is 90 wm^2. I found online that in the Northern hemisphere the average temperature difference in Jan. vs. Jul. is 25.6F.

    25.6/90= .284F for each wm^2

    .284 x 2 wm^2 = .568F expected increase from a CO2 doubling.

    This is pretty amateurish stuff, but it would nice if you or someone else could point out the flaws in my thinking. First off it doesn’t pass the smell test because the number is so much smaller than the scientists who actually study this come up with.

    Cheers

    • That’s exactly what prof Murray salby said was the doubling temperature from atmospheric carbon 14!

    • No, because the the sun sees a different Earth during summer versus winter. In NH summer, the sun is farther away but sees mostly land, which warms up faster than ocean. In NH winter, the sun sees mostly ocean (the SH). So, the average surface temperature of the Earth is actually greater when there is less sunlight falling on it.

      If the Earth was all ocean, or all land with uniform surface characteristics, then you might be able to get a decent climate sensitivity estimate.

    • Joel Shore says:

      Travis:

      In addition to Roy Spencer’s points, I might add that the proposed estimate is naive for other reasons too: It neglects the considerable transfer of heat around the globe. Also, if you cycle a forcing at a frequency that is faster than some of the relaxation times of the system, you will get a reduced response relative to if you cycle it more slowly. (I.e., the oceans have a large heat capacity and hence it takes a while to cool or heat them!)

      But perhaps the best test of your idea is this: If your were correct that the climate sensitivity could be estimated in this simple way and is really much lower than the IPCC believes, that means that all of these climate models that have these higher climate sensitivities would presumably have some ridiculously-large seasonal cycle that has no relationship to reality. However, this is not the case. In fact, the study that I know of that tried to constrain the climate sensitivity using the seasonal cycle found a sensitivity in the range of the IPCC estimates: “it is found that climate sensitivity is very unlikely (5% probability) to be either below 1.52 K or above about 56.5 K, with the best agreement found for sensitivities between 3 and 3.5 K” (http://journals.ametsoc.org/doi/abs/10.1175/JCLI3865.1)

      • Joel Shore says:

        Sorry…There were some formatting problems when I cut and pasted that quote from the abstract of the paper. The lower and upper bounds were not 1.52 K and 56.5 K but rather 1.5-2 K and 5-6.5 K.

  20. Bindidon says:

    I don’t understand why my comment just sent a minute ago disappeared. Yesterday the same phenomenon.

  21. Bindidon says:

    Sent again, dropped again… What is this for? Was the reference to “HITRAN” a problem?

    • Norman says:

      Bindidon

      I couldn’t post HITRAN links on another thread. The site does not allow some links. If I make a long post with links I copy it before sending, then if it does not go I remove the links one at a time until I find the offender. I don’t think HITRAN links will post.

      • Bindidon says:

        A trial:

        – H2O: 4968 lines peaking at 0.065 cm-1 / cm
        – CO2: 106,000 lines peaking at 0.0025 cm-1 / cm
        – N2: 2 lines peaking at 6e-20 (!!!) cm-1 / cm
        – O2: 0 lines

  22. Bindidon says:

    I don’t understand why so many messages lacking any problematic content (insult, refs to Do*ug Cot*ton etc) simply disappear after having been sent.

    • seems to be something about your lists of spectral lines that the spam filter doesn’t like. Nothing in my blacklist of terms is tripping it that I can find.

      Are you selling spectral lines at cut-rate prices or something?

      • Bindidon says:

        Of course I’m not, Mr Spencer.

        The 4 links to jpeg files containing plots out of pdfs made by spectralcalc for CO2, H2O, N2 and O2 were eliminated first; that didn’t help.

        No idea of why some comments pass and some don’t. I guess it is possibly due to a combination of characters.

        But: should these last comments not have landed somewhere in a junk post directory at your site?

        *

        Many thanks anyway for this good, instructive sequence of articles about this problematic theme.

  23. Bindidon says:

    CO2 s ability to absorb and emit IR between 9.5 and 23.5 is about 4,000,000,000,000,000 higher than N2 s

    If this comment is published, then I guess comments containing single quotes are automatically eliminated…

  24. Travis Casey says:

    Thank you sir.

  25. phi says:

    Dr. Spencer,

    You wrote :

    “It is only meant to demonstrate the most basic components of the atmospheric “greenhouse effect”, which act to:

    1) make the Earth’s surface warmer than it would otherwise be, and

    2) warm the atmosphere, but still keep it cooler than the surface (since the atmosphere cools radiatively to deep space).”

    1) is ok but “2) warm the atmosphere…” is clearly wrong because without greenhouse gases, the atmosphere could only be cooled by conduction at its base but could always be heated by thermals in warmer regions. Furthermore, GHGs are net emitters of IR, so they cool, not warm, the atmosphere.

  26. crakar24 says:

    As I am not trained in this field (if you want your radar fixed I am your man) I do find it hard to keep up with what is being said.

    My current understanding is as follows:

    Electro magnetic energy in the form of visable light from the sun strikes the surface (ignoring albedo etc)and is absorbed. So it is now energy in the form of heat (kinetic energy??), the surface then releases this energy as infra red energy.Therefore at this point the net energy gain/loss at the surface is (theoretically zero).

    The IR energy then bounces aroung amongst the ghg’s as it escapes to space, at this point I dont understand how “back radiation” can make the surface warmer because the surface would have cooled when the ir originally left it (does this make sense?)

    Anyway moving on, the main emission layers are surface, cloud tops, water vapour and CO2. If you increase CO2 then you raise the height of the emission layer, when this happens it cools meaning less IR emitted and so in response the surface warms to allow it to emit more IR (keep energy balance as it was etc).

    Is the above basically correct or am I missing something?

    Regards

    Craig

    • Ed Bo says:

      Test — Having trouble posting…

    • Ed Bo says:

      crakar24 – I have a long response that the site won’t let me post. I’ll try again later.

      • Ed Bo says:

        [For some reason, the spam filter seems to be catching this. I’ll try posting part of it…]

        crakar24:

        You’re pretty close! If you haven’t had formal background in thermo and heat transfer, it can be pretty confusing (witness many of the commenters here!).

        You may find it easier to use some analogies. In formal systems analysis, there are “potential” variables and “flow” variables. In thermal systems, temperature is the potential variable, and heat flux is the flow variable.

        In electrical systems, voltage is the potential variable, and current is the flow variable.

        In fluid systems, pressure is the potential variable, and fluid flow is the flow variable.

        For completely non-technical people, I like to use financial accounting, where your bank balance is the potential variable, and money income and outgo are the flow variables.

        In a system at steady state, the incoming flow matches the outgoing flow, so the potential value is constant.

        For a planet without an atmosphere or with transparent atmosphere, in the steady state, the infrared radiation output from the surface would have to match the shortwave radiation input from the sun. Such a planet would be much colder than the earth (as the moon is on average).

        I’ll start with the financial analogy. You earn $240 a week, and you spend $240 a week — 12 purchases of $20 each. Your bank balance is constant week to week.

        But now you start getting $5 in change or a rebate from each $20 purchase. Note that you only this money if you make a purchase, and it is always less than the purchase. But still, your bank balance is higher with you getting the change than without.

        So the “back radiation” from the radiatively active gases works like the change from your purchases.

    • MikeB says:

      crakar24

      Essentially correct. Just to fill in some detail. The energy loss from the surface is not just by radiation. The surface also loses heat by conduction/convection and evaporation. So, if there were no greenhouse gases, the surface would receive one energy input (from the Sun) and lose energy by radiation, convection and evaparotion. But the greenhouse gases enable the atmosphere to absorb part of the outgoing radiation and warm the atmosphere. The warm atmosphere emits its own radiation in turn and some of this is directed back to the surface. This back-radiation is an ADDITIONAL heat input! If there were no greenhouse gases it would not be there.

      So the surface now has two heat inputs, from the Sun and from the atmosphere. Its temperature must therefore rise relative to the one input case. As it rises the heat losses from radiation, convection and evaporation will also rise The temperature will continue to rise until a new balance is found between the incoming radiation and the outgoing heat flows.

      The alternative way to look at it is, as you say, extra CO2 raise the Effective Radiating Height to a cooler altitude. To maintain the lapse rate, the surface temperature has to rise.

      • Ball4 says:

        MikeB,”This back-radiation is an ADDITIONAL heat input!”

        That is incorrect, the source of much confusion causing so much trouble MikeB, this back-radiation is an ADDITIONAL absorbed ENERGY input!

        Back or forward radiation is energy, not heat as radiation contains no KE.

        • Tim Folkerts says:

          “Back or forward radiation is energy, not heat as radiation contains no KE.”

          Ball4, I was with you until here! Where did you get the notion that heat “contains KE”? When addressing Mike, you seemed to be using the strict thermodynamic sense of heat, Q, as the net energy transferred from hot to cold. But heat, Q, can no more “contain KE” than work, W, can “contain KE”.

          • Ball4 says:

            Tim, my longer reply went to dustbin. Objects do not contain heat, they contain measurable KE. Radiation (EM) does not contain KE, so radiation does not contain heat. In science, heat is a relic, no corporeal existence anymore.

          • Tim Folkerts says:

            That didn’t improve the clarity.

            “Objects do not contain heat, they contain measurable KE.”
            Yes, exactly. Stop there. *NOTHING* “contains” heat, Q, in the modern sense of Q. Saying something “contains” heat makes no more sense than to say something “contains” work. No one would say “My hand contained some work, and then after I threw a ball, the ball contained some more work”.
            Things “contain” internal energy, U. They could have PE or KE. They can’t contain Q.

            Thus, “Radiation (EM) does not contain KE, so radiation does not contain heat”. (I hope the formatting comes through). No qualification is needed.

            Thus “In science, heat is a relic, a process that reduces the internal energy of one thing and increases the internal energy of a cooler thing; no corporeal existence anymore ever.”

            So thermal radiation — the process of energy being transferred from the internal energy of warm objects to the internal energy of cool objects (via EM waves/photons) — is heat, Q.

          • Ball4 says:

            Tim, my longer replies are not making it. Thermal radiation is not heat either nor is thermal radiation Q. The term thermal is short for (U) therm -odynamic intern-al energy, U is not EM radiation. U is the measurable KE of constituents of an object. Get rid of heat in any explanation and your clarity immediately improves. No need for heat, it really, really is just a relic, not even a process. KE transfers that’s all you ever need.

          • Ball4 says:

            Q is U (KE) change by virtue of a temperature difference.

            W is U (KE) change by work done to/output from

            KE transfers by radiative, conductive, convective processes, heat should be left in the 1800s where it belongs. Even though Kristian twists himself in a pretzel to give heat some kind of corporeal existence, there is no success only confusion. The dictionary does so also, what humorous attempts are made – even by you. If heat doesn’t exist in an object as you admit, then heat cannot possibly transfer. KE exists, can transfer.

          • Tim Folkerts says:

            I agree that we are getting into more of a philosophical discussion, and that there are various ways to interpret “heat” and “work”. And that often these discussions turn out to be ineffective.

            I would leave you with two thoughts.
            1) since photons have no rest mass, then you could consider ALL their energy kinetic.

            2) The heat capacity for a solid clearly shows that the internal energy consists of both the KE of the vibrating atoms AND the PE of “springs” that hold the atoms in place. As such, you can’t define U as merely the KE of the particles.

          • Ball4 says:

            Tim, the discussion would only be pure philosophy if the misuse of the heat term did not cause faulty conclusions.

          • Ball4 says:

            1) Come on Tim, you know photon energy is hf not mv^2.
            2) I only mentioned U changes from Q,W. Total U change includes the change in many more terms chemical, nuclear, positional, rotational so forth, check out the whole Hamiltonian list. Of main importance to an atm. are Q,W.

          • Kristian says:

            Tim Folkerts says, September 1, 2016 at 3:54 PM:

            Ball4, I was with you until here!

            Ok. So, since you’re seemingly “with” Ball4 concerning the postulated “back radiation flux” from the atmosphere to the surface being an “additional absorbed energy input” only: How come it gets to raise the equilibrium surface T by many tens of degrees, directly and all by itself, if it’s not really an additional absorbed “heat” input to the surface?

          • Ball4 says:

            Because, Kristian, it’s really an additional absorbed “energy” input to the surface.

    • mpainter says:

      Craig, you have touched on the main points of the AGW hypothesis. These are in the main, gross over simplifications, even to the point of misconception.

      Take for example the “effective radiating level”, or “ERL”. This is a hypothetical concoction that mistates reality. The atmosphere radiates to space from all levels, including the surface.

      Also, clouds and water vapor radiate at the same wavelengths as CO2. This fact is utterly ignored by AGW proponents. There is no wavelength pertaining to CO2 that is not shared by atmospheric water. In short, radiation to space is not affected by addition of CO2 to the atmosphere, as its radiative properties are redundant to atmospheric water.

      • Geoff Wood says:

        mpainter;

        “Take for example the effective radiating level, or ERL. This is a hypothetical concoction that mistates reality. The atmosphere radiates to space from all levels, including the surface.”

        When you view the Earth “answering” to space in the IR, the radiation observed appears to come from a range of emitting bodies, the bulk of which are above the surface. Averaging these effective height along with retrievable mean emissivities gives the most sensible answer to the other (most commonly) averaged number, temperature, which is often mistaken in energy balance equations for a surface temperature. The most accurate flux balancing required temperature appears to come from some point above the liquid or solid surface; the ERL. It’s just a sensible physical mean.

        What’s wrong with that?

  27. ren says:

    Sorry.
    The first cyclone over Hawaii. Is approaching the second.
    http://www.accuweather.com/en/us/south-shore-hawaii/weather-radar-r1h?play=1

  28. gbaikie says:

    — crakar24 says:
    August 31, 2016 at 8:54 PM

    As I am not trained in this field (if you want your radar fixed I am your man) I do find it hard to keep up with what is being said.

    My current understanding is as follows:

    Electro magnetic energy in the form of visable light from the sun strikes the surface (ignoring albedo etc)and is absorbed. So it is now energy in the form of heat (kinetic energy??), the surface then releases this energy as infra red energy.Therefore at this point the net energy gain/loss at the surface is (theoretically zero).

    A surface emits a certain amount of energy depending on temperature of material.
    Energy is either kinetic or potential energy. A charged battery is example of potential energy. Or a rock balanced on a hill is potential energy- because it can rolled down the hill- it rolling down a hill is kinetic energy. But perhaps in this context kinetic energy might refer to gas- the velocity of gas molecules is related to it’s temperature. Air around you is traveling at about 400 m/s, but going very short distance before it collides
    another gas molecules or surface of liquid or solid. And the energy it transfer is “mechanical motion” like the rock rolling down a hill. All the energy of a gas is related to it’s mass and velocity. Gases are explained by ideal gas law, which assumes that “zillions”- a lot more than billion- are hitting each other and have an average velocity and they don’t lose energy in the collision with other gas molecules [there is no frictional loss involved with the collisions- but they transfer their kinetic energy- they gain and they lose in fractions of nanoseconds, but “zillions” of them would have average velocity- and room temperature air average velocity is faster than a bullet- or if a “zillion” air molecules have same combined mass as bullet, they have same energy as the bullet [actually more because velocity is faster]. Now when gas molecules hit a surface of solid or liquid, they are colliding material which is bonded together into some kind molecular structure, and molecular structure gain kinetic energy or give kinetic energy from gas molecules- depending on solid or liquid temperature as compared to average velocity of zillions of gas molecules. Or if gas is hot enough it can melt or evaporate the solid or liquid. Or if solid or liquid is cold enough it might condense the gas [make the gas into liquid or solid]
    Anyway sunlight can be 1000 watts per square, and surface- say a rock might be at temperature that without the 1000 watts it emits say 10 watts per square meter [a cold rock- something like -150 C]. So when cold rock has this 1000 watts per square of sunlight warming it, rock, could reflect or absorb a certain amount of 1000 watts per second. So warms the surface and if surface is warmer beneath the surface of rock, the heat surface can conduct heat toward the interior of rock.
    Lets say the rock reflects 100 of 1000 watts of sunlight- that 100 watts of energy not absorbed by the rock. So surface heats up and the higher the temperature of surface compare to temperature say 1 mm inside rock the more heat is conducted into the rock.
    As rock warms it radiate more energy into such time as it’s radiating as much energy as the sunlight- so 900 watts is, a blackbody at 360 K radiate 952 watts per square. And because rock reflecting sunlight, it’s not blackbody. So once rock is about 360 K [around 85 C or 185 F] it can’t get warmer from the sunlight. Now if warming rock is in atmosphere, the gases will colliding with it, as rock warms up it heats the gases- or the heated rock has convection heat loss to the gases. Amount of convectional heat loss to the atmospheric gas also depends how big the difference is between rock and the air.
    And when sunlight is no longer shining on the rock, the surface rock cools by conduction, and convectional lose. And then once surface is cooler than inside the the rock, the interior rock conducts heat to cooler surface. Etc

    • crakar24 says:

      Thanks for your detailed response gbaikie, I kind of understand what you say about the rock but I have no idea how this explanation translates back to my original comment and are therefore still none the wiser as to whether my basic understanding of how GHG’s retain/trap heat in the atmosphere.

      Regards

      Craig

      • gbaikie says:

        — crakar24 says:
        September 1, 2016 at 5:15 PM

        Thanks for your detailed response gbaikie, I kind of understand what you say about the rock but I have no idea how this explanation translates back to my original comment and are therefore still none the wiser as to whether my basic understanding of how GHGs retain/trap heat in the atmosphere.–
        Well it getting long winded, but quick answer is there is no agreed upon answer, rather there are different ways it’s explained and understood.
        It sort of like, how do get a unicorn? Some people have different opinions but quite few believe you get one.
        Spencer thinks greenhouse gases warm the atmosphere and believe such a warmed atmosphere inhibits radiation from leaving the surface and getting to space.
        Other believe the greenhouse gases heat the earth surface- that it’s more just matter of slowing the rate radiation leaves Earth.

        I tend to think of it like engineer, how would you make a planet warmer.
        The GHE theory explains how to warm the planet Mars. One adds greenhouse gases. That according to GHE theory is the only way that one could warm Mars.
        Now, it seems to me that most believers of GHE theory think a greenhouse on the Moon would be hot during it’s long daylight hours. So to be clear a greenhouse lacking any greenhouse gases would be hot.

        So if had a warm rock and placed it on my driveway, Spencer think the rock would cool faster if my driveway was on the Moon.
        So night time, same temperature driveway, place a warm rock on it, and the moon rock cooler quicker.
        Not sure how much quicker the Moon rock is suppose to cool- but pretty sure Spencer would say moon rock cools faster- but not sure all believers would agree.
        Now I think most would agree that the bigger the rock [a rock with more thermal mass] the slower the entire rock cools.

  29. Tim Folkerts says:

    I mostly like what gbaikie said, but a couple points bear a little more discussion, particularly “So once rock is about 360 K it cant get warmer from the sunlight. ”

    This is a good answer for the given conditions — where the only energy loss is radiation to the cold 2.7K background of outer space.
    * When conduction/convection/evaporation are included, the rock would of course be limited to a temperature below 360K.
    * More importantly to this discussion, if the rock is radiating toward something that is warmer than 2.7K, then the rock could get WARMER than 360K!

    The spreadsheet shows this. If you change the power in to 900W/m^2 and set the atmosphere clear (emissivity = 0), the surface does indeed get to ~ 360K. IF you then set the atmosphere back to 0.8, the surface warms to a little over 400K.

    The combo of sunlight and atmosphere can warm the rock above 360K — even though the energy all ultimately came from the sun.

    • gbaikie says:

      — Tim Folkerts says:
      September 1, 2016 at 8:17 AM

      I mostly like what gbaikie said, but a couple points bear a little more discussion, particularly So once rock is about 360 K it cant get warmer from the sunlight.

      This is a good answer for the given conditions where the only energy loss is radiation to the cold 2.7K background of outer space.
      * When conduction/convection/evaporation are included, the rock would of course be limited to a temperature below 360K.
      * More importantly to this discussion, if the rock is radiating toward something that is warmer than 2.7K, then the rock could get WARMER than 360K! —

      Hmm, I didn’t realize that it was that important.
      It depends. The sun at certain distance has a not magnified highest temperature. Or also if magnified it has a highest temperature it can become- which is the temperature of the sun- at the sun.

      So if star is 6000 K it it’s surface, then highest magnified temperature one could can get is 6000 K.
      Unless you not measuring the sun’s temperature correctly- or you doing something which causes the sun’s temperature to increase.
      Or highest temperature one can get from lightbulb is the temperature of the filament of the lightbulb- about 3200 K. You could do stuff which would increase the filament temperature- but as practical matter, humans can’t increase the temperature of the sun- or our current capability would require a lot of money to do this- as in, Bill Gates does not have enough money to effect the sun temperature by any significant amount.
      So quite simply the sun energy does not increase with distance and at 150 million km from the sun it’s cooler than at 10 million Km from it.
      Or we have no way to directly measure the temperature of the sun, but we can measure it’s temperature at distance and we know that energy is conserved.

      And once you absorb the sun’s energy, you changed the sun’s energy- you converted the energy into IR rather than the blackbody spectrum- which includes X-rays, UV, visible light, and IR light. And practical level the sun is very hot- it’s temperature turns everything in to very hot plasma. Or there is reason lightbulb are limited to about 3200 K- and only emit small portion of their light into visible light.

      And this is one reason why GHE doesn’t explain Venus.

      • Tim Folkerts says:

        “The sun at certain distance has a not magnified highest temperature. “

        More specifically, you seem to be describing the highest temperature that non-magnified sunlight could cause for a blackbody radiating to the 2.7 K background of space. In other circumstances, non-magnified sunlight could cause much lower OR MUCH HIGHER temperatures.

        For instance as a baseline, if 960 W/m^2 falls on a BB surface in the depth of space (and the “back” is well-insulated), the surface could get up to 360K — as you stated earlier.

        If the surface reflected 90% of incoming light but was still a BB for IR light (emissivity = 1 for 4+ um; emissivity = 0.1 for 0-4 um) , the temperature would only reach ~ 200K. Conversely, if sunlight is absorbed but IR is not emirtted well ( (emissivity = 0.1 for 4+ um; emissivity = 1 for 0-4 um), the temperature could rise above 600K.

        Or if the BB surface was in a room at 360K, the sunlight would further warm the surface to ~ 430K. Or if the BB surface was in a room at 430K, the sunlight would further warm the surface to ~ 475K.

        The only theoretical limit for non-magnified sunlight is the temperature of the sun. Or put another way, no matter how far you go from the sun, the peak color is yellow because the photons themselves are still at 5700K = surface temp fo the sun.

        • gbaikie says:

          –Or if the BB surface was in a room at 360K, the sunlight would further warm the surface to ~ 430K. Or if the BB surface was in a room at 430K, the sunlight would further warm the surface to ~ 475K. —

          Should be easy to prove. Can’t fry eggs on sidewalk. Fry eggs in greenhouse using non magnified sunlight
          If magnifying sunlight [using reflectors] one can easily fry eggs on sidewalk with sunlight- you could bake a cake. So that is not the issue.

          • Tim Folkerts says:

            I have thought about doing exactly that. Much like Dr Spencer’s last post, it would be a simple experiment using inexpensive stuff from a hardware store. If I have some extra time, I will have to do it.

  30. Nabil Swedan says:

    I respect and understand Dr. Spencer’s position in defending the current understanding. I cannot think of one person who has not been right and wrong at the same time relative to the climate including me. It took me over 20 years to accept the fact that variation of small amount of carbon dioxide can change climates.

  31. Chic Bowdrie says:

    Dr. Spencer,

    If I understood the two models’ assumptions, you are comparing a planet without an atmosphere (a=e=0) to a planet with an atmosphere containing IR-active gases. So the case of an atmosphere, but without IR-active gases, is bypassed. And the IR-active atmosphere is assumed to have no conduction or convection permitted.

    I don’t doubt the surface would be warmer with IR-active gases than without. However, the cause of the difference would be due to Holder’s inequality. IOW, a more temperate climate is warmer than one which has more extreme temperatures.

    It should be possible to modify your model to allow a portion of the energy flux to be attributed to conduction (and convection if input energy is not constant). The extreme case of 100% conduction is analogous to an atmosphere without IR-active gases. I don’t think there is a 0% conduction case unless there is a vacuum layer separating the surface from the atmosphere.

    So you have presented further evidence that a planet with an atmosphere is warmer than one without. The question remains: how much warmer, if any, would a planet be with IR-active gases compared to a similar atmosphere without IR-active gases? More importantly, will an increase in IR-active gases above a critical concentration make the surface of a planet any warmer?

    • Joel Shore says:

      Chic,

      I believe Spencer’s model is “zero-dimensional”, so Holder’s Inequality is irrelevant…There is just one surface temperature.

      In the real world, I agree that an atmosphere without IR gases can raise the average temperature by virtue of making the temperature more uniform, i.e., it can change the average temperature T while not changing the average of T^4. But, you need an IR-active atmosphere to actually get what is observed on the real earth, which is temperatures at the surface that lead to IR emission far in excess of that which would be supported in the absence of a greenhouse effect. In particular, the average surface temperatures are about 33 K warmer than could be possible with any temperature distribution in the absence of an IR-active atmosphere (given the actual albedo, etc.).

      “More importantly, will an increase in IR-active gases above a critical concentration make the surface of a planet any warmer?”

      Well, there are two questions here:

      (1) What is the effect on radiative balance of adding more greenhouse gases at current concentrations? That is a solved problem…Every serious scientist (including AGW skeptics like Dr. Spencer and Dr. Lindzen) agree that doubling CO2 produces a forcing of ~4 W/m^2. [And, there is no “critical concentration”…The effect of adding more IR-active gases does not saturate, although there are diminishing returns; in particular, the effect of CO2 is approximately logarithmic in concentration in the concentration regime we are in. This is why one talks about the effect of doubling CO2 rather than the effect of increasing CO2 by a certain additive amount.]

      (2) How much warming will that lead to? That is a more difficult question, since it requires an understanding of the various feedbacks in the climate system and the resulting ultimate climate sensitivity. And, this is where most of the climate science community disagrees with the skeptics, with the latter saying the sensitivity is about 1.5 to 4.5 K for a CO2 doubling and the skeptics believing that something (usually a negative feedback from clouds) results in a lower value.

  32. Ball4 says:

    Chic, “how much warmer, if any, would a planet be with IR-active gases compared to a similar atmosphere without IR-active gases?”

    The above spreadsheet allows the emissivity of Earth atm. to be increased above 0.0 which means adding IR active gases all the way up to current global 0.8 – so you can answer your own 2 questions as you raise atm. emissivity above 0.0.

    • MikeB says:

      Hmmm. About 33 degrees Celsius, it is an easy calculation

      • Ball4 says:

        Very good, MikeB, now let’s see if Chic 10:15am can answer also.

      • Chic Bowdrie says:

        MikeB,

        The 33 degree calculation comes from a comparison of a planet with an IR-active atmosphere compared to one without. It is not so easy to calculate the difference between a planet with an IR-active atmosphere and a planet with the same atmosphere sans IR activity.

    • Chic Bowdrie says:

      The spreadsheet only deals with radiation, not conduction or convection, and can only simulate the atmosphere of a planet with IR-active gases and compare it to one without an atmosphere.

      • Ball4 says:

        Compares to one WITH 1bar atm. but no IR active gas at emissivity 0.0 which is not possible but close enough to N2,O2 atm. for government work.

        • Chic Bowdrie says:

          Do you deny that some energy from the surface will be transferred to the 1bar atmosphere without IR-active gases? Emissivity of 0 does not prevent the inert atmosphere from warming. This makes a planet with an inert atmosphere warmer than one with no atmosphere.

          • Ball4 says:

            Conduction updrafts exactly balance downdrafts, convection does not get to space at emissivity 0.0 or 0.8. Thus no net energy change due conduction/convection, no global Tmedian change.

      • Ed Bo says:

        Chic:

        The only mode of energy transfer the earth or its atmosphere is radiation. So an atmosphere without radiatively active gases cannot transfer energy to space.

        Any convection in such an atmosphere is simply moving energy around within the atmosphere. Such an atmosphere must transfer as much energy to the surface as it receives from the surface (on average over the long term).

        Contrast that to the real atmosphere, which can radiate to space. Here, when the surface conducts energy to the bottom of the atmosphere, and this rises up to a great height where it can radiate more easily to space, the energy returned to the surface in the downward part of the convection cell is less than what the atmosphere absorbed.

        So in our atmosphere, convection provides a net cooling effect to the surface. In a transparent atmosphere, it would not.

    • Chic Bowdrie says:

      Although an inert atmosphere can’t radiate much at altitude, it will increase in temperature via conduction and convection. The increased thermal energy has no place to go. Therefore the average surface temperature will increase until an equilibrium is reached where the energy radiated to space matches the solar input. It won’t be as warm as an IR-active atmosphere, because extreme temperatures radiate more than moderate temperatures which radiate more than a uniform temperature the same as the average of the more extreme temperatures.

      • Ball4 says:

        Chic, it doesn’t rain in space and no convection gets to space, thus they do not affect the overall system energy equilibrium. Their energy (LH & SH) is removed from, and returned to, the surface in equal emounts. Dr. Spencer could easily add them in and subtract them right out, there would be no difference in Tmedian in that simple program.

        • Chic Bowdrie says:

          It seems you aren’t grasping my point. The inert atmosphere will warm by conduction. There will be some convection although not as much as there would be with help from IR-active gases. The surface temperature extremes will be greater on the inert atmosphere and therefore not as warm as the surface of a planet with an IR-active atmosphere, but still warmer than a planet without an atmosphere.

          • Ball4 says:

            The surface temperature extremes will indeed increase as the sun will be stronger on the surface, but there is no energy created nor destroyed to change Tmedian 255K as calculated above.

          • mpainter says:

            SST is determined by insolation. Never is the sea surface in “equilibrium” radiatively. This meme is more AGW myth.

          • mpainter says:

            The above a response to B4.

      • Joel Shore says:

        Chic,

        The average temperature of 288 K is ~33 K warmer than the Earth’s surface could possibly be with any temperature distribution if the atmosphere were not IR-active. I.e., 255 K is the upper bound for how warm the surface could be (given current albedo). Yes, it is true that the average could be colder (via Holder’s Inequality) if the distribution were uneven but it couldn’t be any warmer.

        • mpainter says:

          Joel, you repeat the error. This meme ignores the fact that SST is determined by insolation alone. The sea surface is not a black body, nor a gray body, nor does it ever achieve radiative equilibrium nor does it cool radiatively, in the main. The GHE makes no contribution to SST.

          • Ball4 says:

            To write the sea surface does not cool by radiating is wrong mp, you exhibit the extreme shallowness of your accomplishment in this field. Try to do some work better understand the analysis in Dr. Spencer top post, the L&O surface measured 0.96 emissivity so rounded to 1.0 means the L&O surface does cool by radiation, & by conduction, convection.

          • mpainter says:

            B4, right, the sea surface cools radiatively. It cools mainly by evaporation, according to partition studies of its energy emissions. That was my intended meaning when I put ” nor does it cool radiatively, in the main.”

            I have discussed this all in detail in previous threads. Sea surface cools mainly through evaporation, 60%. Radiative cooling is put at half that: 30%.

          • mpainter says:

            My point that SST is determined by insolation alone stands.

          • Ball4 says:

            That would be 100% yet mp just partitioned 60/30 can’t even keep story straight comment to comment.

          • mpainter says:

            Did not consider you would need help on these concepts, B4.
            What is it about partitioning that you don’t understand?

          • Tim Folkerts says:

            mpainter says: “This meme ignores the fact that SST is determined by insolation alone.”

            This is rather bizarre! No temperature anywhere is determined by any one power input alone. Temperature is a response to ALL inputs as well as ALL outputs!

          • Ball4 says:

            mp asks, “What is it about partitioning that you dont understand?”

            Nothing. What I don’t understand is that mp writes repeatedly “SST is determined by insolation” that’s the 100%. Then “Sea surface cools mainly through evaporation, 60%”

            1st grade arithmetic says that’s 160% determination and then…THEN mp writes: “Radiative cooling is put at half that: 30%.” so that’s 190% of SST determination.

            Did you get held back at 1st grade mpainter? At least the published balances you nonsensically whine about can do accurate arithmetic.

          • mpainter says:

            In fact, it is generally admitted at RealClimate that back radiation makes no direct contribution to SST because of its opacity to LWIR. They put a different means of IR effect for warming the oceans. See the link provided by Toneb below. B4, Tim Folkerts, and others, get up to speed on the physics of the sea surface.

          • mpainter says:

            B4, I’m concerned that you have a type of dyslexia, or so it seems. Try reading it again, and strive for better comprehension.

          • Ball4 says:

            Do not be concerned mpainter, do try to catch up with basic experimental science. And study up on arithmetic and percentages, try to make your 1st grade teacher proud.

  33. sky says:

    Although I cannot take the time to read all comments here, it seems that no-one has convincingly explained how a nearly constant TOA stream of solar energy at a density, say, X W/m^2 can maintain a system-wide stream > X, without additional–non-existing–SOURCES of energy. And that violation of energy conservation is what makes the purely radiative explanation of GHE physically incredible. Clearly, being an intensive variable, temperature can vary widely in the real world without effecting planetary energy budgets. But that variation must be the result of other, non-radiative mechanisms.

    BTW, it seems to escape many that it’s only the NET between the upwelling and downwelling LWIR that is constrained by the energy budget. ANY such pair of radiative intensities that produces the same net value will maintain the budget. Contrary to claims here, the surface intensity (gray body temperature)thus remains undetermined.

    • Ball4 says:

      “ANY such pair of radiative intensities that produces the same net value will maintain the budget.”

      The surface balance has to be consistent with Tmedian, sky. The one and only correct balance pair is set by measured Tmedian field, all other pairs are thus excluded. No energy is created in the balances, they are consistent with 1LOT and 2LOT as they match observations to a reasonable CI.

      • sky says:

        The one and only correct balance pair is set by measured Tmedian field, all other pairs are thus excluded.

        The necessary reliance upon empirical measurements only underscores my point about the inadequacy of the radiation-only model. BTW, it’s not the median but the time-average temperature values that are employed for that face-saving purpose.

        • Ball4 says:

          An average need not exist sky, a median must exist. BTW, the radiation only model is not at all inadequate, confirms Tmedian easily enough and its use in RTM is very useful.

    • Joel Shore says:

      “And that violation of energy conservation is what makes the purely radiative explanation of GHE physically incredible.”

      There is no violation of energy conservation. All models, whether they be simple models like Dr. Spencer’s or full-blown GCMs impose energy conservation. The fact that you think there is a violation of energy conservation is simply a statement that you lack the understanding and so you are making up artificial rules that you think energy conservation imposes when it doesn’t.

      Your claim is akin to saying that it is impossible to create more aluminum products in a year than the amount that can be created from the raw ore that is mined. It ignores the fact that you can recycle aluminum. Recycling aluminum does not violate mass conservation.

      • mpainter says:

        Joel Shore, There’s no recycling of energy in a surface-atmosphere energy flux, anymore than in a cloud-atmosphere energy flux, or as in the atmospheric energy flux. Yours is a poor analogy, but typical of the sort of misconceptions engendered by the “back radiation absorbed by the surface” energy budget diagrams.

      • Joel Shore says:

        There is recycling of energy in both the surface-atmosphere and cloud-atmosphere energy flux. Saying there is none doesn’t make it so.

        As for misconceptions, it is interesting that those who have a good background are the ones who you think have misconceptions while those with poorer backgrounds in physics are the ones who deny the greenhouse effect.

        • mpainter says:

          You cling to your misconceptions and justify them with a snarky dodge.

        • Tim Folkerts says:

          I wouldn’t have thought of “recycling” as an analogy, but it works quite well.

          The surface generates “waste IR” energy that is discards off toward the “landfill” of outer space. Without any “recycling”, the current ~ 390 W/m^2 of waste IR energy would all simply disappear forever (leading to dramatic cooling). Instead, the atmosphere intercepts the waste stream and takes in most of the discarded waste energy for its own use (~ 350 W/m2 out of the total 390 W/m^2). Eventually, the atmosphere discards its energy in the form of its own “waste IR”. Some of the atmosphere’s waste IR goes off to space, but some of it heads back toward the surface, where it gets absorbed and added to the surface’s energy budget.

          (In the real atmosphere this is complicated by other methods of “recycling” (convection, conduction, evaporation), but the basic idea still applies.)

          • mpainter says:

            More of the egregious AGW physics that purport a non-existent “radiative equilibrium” at the sea surface and which ignores the cumulative process of the ocean with regard to solar energy.

        • Ed Bo says:

          Tim:

          I like your analogy! Mind if I steal it? I’ll add it to my “starting to get change/rebates on purchases when you have a steady income” financial analogy.

          It’s amazing to me the number of people who don’t understand the implications of a “conserved quantity”, whether it is energy, mass, electric charge, or money.

      • sky says:

        Joel Shore et al.:

        You miss my point. Of course there is thermal energy storage (“recycling”) in the climate system, mostly in the oceans. What I question, however, is the ability of the climate system to “maintain a SYSTEM-WIDE stream” of energy greater than that that provided by thermalized insolation. Of course it doesn’t, as the NET radiative heat transfer clearly shows. But that unmistakable physical fact is obscured by miscasting the unidirectional radiative flux intensities as heat transfers, instead of as inseperable components of the LOCAL temperature. The 390 watts/m^2 emitted at the surface doesn’t reduce otherwise.

        • Joel Shore says:

          “What I question, however, is the ability of the climate system to ‘maintain a SYSTEM-WIDE stream’ of energy greater than that that provided by thermalized insolation.”

          So, in other words, even if we achieve 90% aluminum recycling, it is impossible to get more aluminum from recycling than the input of new aluminum made from bauxite each year?

          “But that unmistakable physical fact is obscured by miscasting the unidirectional radiative flux intensities as heat transfers, instead of as inseperable components of the LOCAL temperature.”

          I have no clue what that sentence actually means. Perhaps you need to actually do what Spencer (and other scientists) do and make real mathematical models rather than confusing sentences.

        • sky says:

          I have no clue what that sentence actually means.

          That’s an implicit admission that Joel Shore knows not the difference between actual heat transfer and directional radiative intensity. The former is an energy flux governed strictly by energy conservation, whereas the latter is merely a localized intensive measure, not subject to any conservation laws. Contrary to the paradigm currently taught to undergraduates, the two cannot be properly equated nor can radiation be perpetually “recycled” without losses to entropy. Local differences in temperature notwithstanding, the rate of flux of thermal energy through the climate system thus is constrained by the thermalized insolation and other genuine (but negligible in practice) sources of heat. Unlike the sun and geothermal processes, the atmosphere produces no heat whatsoever.

          • Joel Shore says:

            “The former is an energy flux governed strictly by energy conservation, whereas the latter is merely a localized intensive measure. Contrary to the paradigm currently taught to undergraduates, the two cannot be properly equated…”

            As near as I can tell, it sounds like you are making Postma-like arguments where you get all confused about issues like expressing things in W/m^2, not understanding that an average intensity value in W/m^2 over an area, multiplied by that area, gives a power (and multiplied by a time gives an energy).

            “nor can radiation be perpetually ‘recycled’ without losses to entropy”

            I don’t even know what that means. Energy is not converted to entropy. (What is true from the Second Law is that you cannot completely convert all heat input into a heat engine into useful work because the fact that entropy cannot decrease means that a certain amount of the energy output must be in the form of heat. But, how you think that relates to what we are talking about here is not at all clear.)

          • sky says:

            Without grasp of the thermodynamic processes that are patently absent in the simplistic GHE paradigm, this discussion is going nowhere.

            The units in which power flux densities are denominated is not at all the issue here; it’s the heat transfer mechanisms between surface and atmosphere, wherein the radiative exchange between those domains is but a minor net contributor in a process dominated in by moist convection.

            While energy indeed can be stored internally in matter as heat, radiation propagates until it is absorbed, re-emitted or scattered. Plainly the outward LWIR radiative field in the atmosphere is far from homogeneous, reducing in intensity with altitude from the surface value to that found at TOA. The notion that it can be forever “recycled” to augment the power available for thermalizing the surface is a fallacy akin to a perpetuum mobile of the second kind.

          • gbaikie says:

            “Unlike the sun and geothermal processes, the atmosphere produces no heat whatsoever.”
            Wind energy.
            Not that wind energy is viable.

            The atmosphere produces heat when it’s air masses descend.
            Atmosphere generates heat when condensable gases form into liquids
            and solids- H20 gas mostly.
            The latent heat of H20 is the most significant heating done by the atmosphere.

          • Ball4 says:

            “While energy indeed can be stored internally in matter as heat”

            The energy is stored internal to an object as kinetic energy sky, heat is only a measure of that KE.

            “..radiation propagates until it is absorbed, re-emitted or scattered.”

            Radiation propagates until it is absorbed sky, radiation continues to propagate when it is reflected or transmitted.

            “The notion that it can be forever recycled..”

            sky’s words only, not in atm. thermodynamics.

  34. Tim Wells says:

    I haven’t believed in global warming since 2006, we have had less than an average summer this year in the UK. Don’t believe the lies of the establishment, as in BREXIT, we were fed statistical lies by all the major institutions, since Brexit our economy is starting to grow. Too many people have their snouts in the trough, making money out of AGW and useless energy production methods.

  35. phi says:

    A description of the greenhouse effect based on backradiations is consistent with the notion of forcing. The problem is that this notion of forcing makes sense only with the assumption of the invariance of the thermal gradient. In this case, any imbalance (change of albedo as adding greenhouse gases, etc.) is recovered by translation of the thermal profile. This invariance hypothesis is at least strange and unfounded.

    The purely radiative model is ineffective because, in the real atmosphere, greenhouse gases play mainly a different role, they constitute the heat sink for convection.

    A change in the concentration of GHGs therefore necessarily affect convection and gradient.

    • Joel Shore says:

      phi: Like Stephen Wilde does, you’ve strung together a lot of physic-sy sounding words but that doesn’t get you past the basic facts: Convection et al. doesn’t occur in space so it can’t explain the “top of the atmosphere” energy balance (i.e., the energy balance between the Earth + atmosphere and the rest of the Universe). By conservation of energy, the Earth’s surface simply can’t be as warm as it is (and hence radiative as much as it does) unless some of that radiation is absorbed by the atmosphere and hence doesn’t make it out into space.

      As for convection: Convection does indeed reduce the radiative greenhouse effect (by about a factor of 2 in temperature, as I recall). If it were not for convection, then the Earth’s surface would be even warmer than it is. The reason why convection can only reduce and not eliminate the radiative greenhouse effect is that convection can only drive the atmospheric temperature profile (i.e., the lapse rate) down to the adiabatic lapse rate; it cannot drive it down to an isothermal profile (which is what is necessary to completely eliminate the greenhouse effect).

    • Joel Shore says:

      Just to add, the gradient (lapse rate) is essentially set by the adiabatic lapse rate. So, to a first approximation it remains constant as GHGs increase. [To a better approximation, the lapse rate is reduced a bit because the saturated adiabatic lapse rate is a decreasing function of the surface temperature. This reduction is the so-called “lapse rate feedback”, which is a negative feedback present in climate models. It does reduce, but does not eliminate, the effect of increasing GHG concentrations.]

    • mpainter says:

      SST is determined by insolation alone, not by GHE. Simple enough to see unless your views are rigidly locked on the AGW meme. I will help you understand, if you need help, Joel Shore.

      • Toneb says:

        No, both TSI and the GHE impact SST’s.
        Simple enough to see unless your views are rigidly locked on the denialist GW meme.

        And don’t give me the b****cks about IR only penetrating x microns…. same on land and so does all solar wavelengths there.
        This of course if you even admit to the GHE existing in the first place.

        • mpainter says:

          “And dont give me the b****cks about IR only penetrating ons.”
          ###

          Since you raised the topic, water is opaque to LWIR. The x microns for the 15 micron wavelength (CO2) IR is 4 microns maximum penetration.

          The whole of the DWLWIR (back radiation) is absorbed at or within a few microns of the air/water interface and quickly returned to the atmosphere by evaporation or radiation.

          To appreciate this one need only consider that water radiates to the atmosphere from its attenuation depths.

          By these considerations we know that the incident energy of back radiation is not conveyed to depth.

          • Toneb says:

            “The whole of the DWLWIR (back radiation) is absorbed at or within a few microns of the air/water interface and quickly returned to the atmosphere by evaporation or radiation.”

            Nope.
            No thermodynamic process is 100% efficient.
            And the conversion of LWIR by the surface microns of the oceans isn’t one either.
            You neglect conduction and turbulent mixing.
            Also, as I said. Land doesn’t allow penetration of any wavelength below the topmost molecules.
            Doesn’t stop that from warming more proportionally with raised TSI.
            And land doesn’t do mixing either.

            By cooling the surface relative to the waters below then there is a greater conductive flux to the surface … hence greater cooling to the atmosphere.

            https://judithcurry.com/2014/05/21/mechanisms-for-warming-of-the-oceans/

            “One thing we can assert however, is that warming of the oceans by an increase in downwelling infrared radiation to the surface is an efficient process, and the initial rate of heat loss by the mixed layer is only slightly less than the magnitude of the imposed forcing. The average surface forcing due to increased CO2 over the past 55 years was roughly 0.4 W/m2. It is therefore not unreasonable to expect that the upper mixed level of the ocean would have warmed by an input of roughly this amount over that time period. Experimental data on warming of the oceans indicate that over the past ~50 years, the average warming was due to a flux of about this magnitude.”

          • Toneb says:

            Also this from Roy….

            http://www.drroyspencer.com/2014/04/can-infrared-radiation-warm-a-water-body/

            “If you claim that any additional IR (say, due to increasing carbon dioxide) is immediately lost by the water body through evaporation, how exactly does that occur? The surface doesnt know why it has the temperature it does, it will evaporate water based (partly) on surface temperature, and it does not distinguish where the heat comes from (solar radiation from above, mixing from below, IR from above, sensible heat flux across the air/water interface). To claim that any energy gain from IR is immediately lost by evaporation is just an assertion.”

          • mpainter says:

            Please see below for my response.

          • Ball4 says:

            mp, “By these considerations we know that the incident energy of back radiation is not conveyed to depth.”

            Last summer Dr. Spencer’s thermometers picked up the temperature difference from added night time cirrus down welling LW several inches deep in a small tub of water free to evaporate mpainter. So once again your study in this field is so lacking you cannot understand and learn from basic experiments.

          • mpainter says:

            B4, you neglect to describe results concerning which you have given ample proof since the experiment that you incompletely grasp.

          • Ball4 says:

            I neglect something such as? Try to make your detail comment not so bizzare as usual mp.

      • Tim Folkerts says:

        mpainter says: SST is determined by insolation alone.

        This is rather bizarre! No temperature anywhere is determined by any one power input alone. Temperature is a response to ALL inputs as well as ALL outputs!

        Insolation alone as teh input (along with BB radiation for the output) would never get the SST above freezing!

        • Ball4 says:

          Much of what mpainter writes is bizarre Tim, clearly mp has not accomplished enough study in this field.

          • mpainter says:

            If you put a little vinegar and oil on your comment, and some salt, it won’t taste so much like sawdust.

        • mpainter says:

          Tim, thanks for your response. You say:

          “ALL inputs as well as ALL outputs!

          Insolation alone as teh input (along with BB radiation for the output) would never get the SST above freezing!”

          ##
          You fail to take into account the thermodynamics of the sea surface, meaning the upper 100 meters (the photic zone).
          Specifically, you neglect to consider that solar energy is _cumulative_ in the oceans, particularly so in the tropics. SST is due to this cumulative effect. Indeed, upwelling sea water (meridional ocean overturning) begins at a temperature of about 4 C, and can reach a temperature of 22 C _before_ it reaches the surface. This is entirely due to insolation (and the accumulation of its energy in the subsurface). Your theory assumes a radiative equilibrium at the sea surface. There is no such thing in the tropics or subtropics, where the ocean accumulates solar energy, nor in the higher latitudes where oceans discharge their accumulated energy and finally reach 0 C, or close to that. Your science applies principles of equilibrium where there is none.

          • Tim Folkerts says:

            “You fail to take into account the thermodynamics of the sea surface, meaning the upper 100 meters (the photic zone).”
            No, I don’t. Thermodynamics says objects warm and cool in response to ALL energy inputs and outputs

            “Specifically, you neglect to consider that solar energy is _cumulative_ in the oceans, particularly so in the tropics.”
            No, I didn’t neglect that. Energy is ALWAYS cumulative — conservation of energy requires that energies always be accounted for. You cumulatively add all inputs and cumulatively subtract all outputs.

            “This is entirely due to insolation (and the accumulation of its energy in the subsurface).”
            Still no.

            “Your theory assumes a radiative equilibrium at the sea surface. “
            No. I don’t assume that.

            “Your science applies principles of equilibrium where there is none.”
            No, your science is inventing energy that does not exist. In particular, with only insolation and without IR backradiation, there would be no 4 C upwelling ocean currents to begin with. If the downwelling IR was eliminated, the surface of the entire ocean would rapidly cool — way past the ability of insolation to keep anything warm. Other than perhaps small patches near the equator, the entire surface of oceans would freeze solid.

            Your myth of “insolation only” warming is ONLY effective with backradiation ALREADY helping hold the entire surface of the earth warm.

          • mpainter says:

            Tim, LWIR is not an input to SST. Even the denizens of RealClimate agree to that.

          • Ball4 says:

            mpainter asserts, “LWIR is not an input to SST”

            Wrong again, mpainter, those “denizens” claim LWIR IS an input to the SST:

            “When clouds are present, they emit more infrared energy towards the surface than does the clear sky.”

            “The difference between the two is the net infrared forcing of the skin layer. If we can establish a relationship between the temperature difference across the skin layer and the net infrared forcing, then we will have demonstrated the mechanisms for greenhouse gas heating the upper ocean.

            There is a clear dependence of the skin temperature difference on the net infrared forcing.”

            Which is what your “denizens” plotted, concluding Delta 0.1K SST for each delta 100 W/m^2 net LW.

            I will let mpainter discover which two items are differenced. It will be educational for him.

          • Ed Bo says:

            Yes, those are part of the real world, following the same laws of thermodynamics.

            And all you have is a fantasy world based on something you (mis)read on the internet years ago…

        • Ball4 says:

          I am glad you showed us you tried that bizarre mixture mp, now try to write comments in compliance with 1LOT, 2LOT that are not so thoroughly bizarre.

        • mpainter says:

          Tim, DWLWIR is not an input to SST, as water is opaque to LWIR. You seem to be having difficulty in grasping this principle.

          • Ball4 says:

            mpainter is having difficulty grasping results from test that water free to evaporate is not opaque to LWIR, data demonstrates an increase in T over water not absorbing the same LWIR. Both Dr. Spencer testing and in situ testing came to that same answer which mpainter ignores.

          • mpainter says:

            The opacity of water to LWIR is shown by data obtained from lab measurements. This data is available over the web.

          • Ball4 says:

            Ought to be easy for mpainter to find and show one then.

          • Ed Bo says:

            mpainter:

            Downwelling solar radiation is not an input to rock temperature as rocks are opaque to solar radiation.

            The energy in solar radiation absorbed in the top few microns of the rock is immediately lost to conduction/convection to the atmosphere, and to upwelling longwave infrared radiation from the rock.

            That’s where your argument leads you!

            Now I’m sure you will claim that water also has the ability to lose energy by evaporation as well, and that this transfer mechanism is at least as big as the other two combined.

            But you have simply assumed (and certainly never demonstrated) that this additional “cooling mechanism” is exactly big enough to dissipate all of the input power so that there is no warming.

            It’s easy to find videos of a LWIR laser boiling water, which you claim would be impossible.

          • mpainter says:

            Ed Bo, is the GHE a laser? Is water rocks?

          • mpainter says:

            Ed Bo, temperature gradient at sea surface means no heat is transmitted down from interface to subskin or below. You obviously lack any understanding of the physics of the sea surface, or you would not talk about rocks and lasers.

          • Ed Bo says:

            mpainter:

            You argued that “DWLWIR is not an input to SST, as water is opaque to LWIR.”

            I directly followed your logic using the directly analogous case of solar radiation and rocks, and found your conclusions did not hold in the real world.

            I anticipated your argument about the extra cooling mechanism of evaporation for water, so I brought up the case of directly adding only LWIR radiation to the water surface. This added radiation is absorbed in the top few microns of the water’s surface.

            Your analysis says this should just increase the rate of evaporation of the water, because “DWLWIR is not an input to SST”. (Your words, not mine!) But in the real world of actual experiment, this is not what happens at all!

            So it is your turn to figure out why your analysis is so wrong. And it is you who does not understand the physics of heat transfer in water.

            The skin layer is generally cooler than the water below, primarily because of the radiation and evaporation losses to the atmosphere. This causes thermal conduction (and sometimes convective overturning) from the warmer water below.

            But since water is highly opaque to LWIR, all of the radiation to the atmosphere occurs from the top few microns. If there were no “back radiation” to compensate for most of this, the skin layer would be a lot cooler, and there would be more upward transfer from deeper layers, cooling them as well.

            So the “back radiation” does lead to warmer water below, even if the net heat transfer is still largely from the deeper layers to the skin layer.

          • mpainter says:

            Ed Bo,
            No more talk about rocks and lasers from you, so maybe you saw the ineptness of your analogy. I note that you agree that water is opaque to back radiation. I note you agree that the incident energy of back radiation is rapidly returned to the atmosphere. You are coming along quite well, Congradulations. Keep thinking about it, and maybe the light will blink on for you: SST is determined by insolation. DWLWIR makes no contribution to SST. The temperature gradient at the sea surface does not allow it. Physics, it’s called.

          • Ed Bo says:

            I note that you never have any actual counterarguments.

            You just keep referring to something you happened to see on the interwebs a few years back but can’t really remember and can’t find (or I suspect you did find and realize it doesn’t back up what you’re saying now.) Pathetic.

          • Ed Bo says:

            I also note that you display absolutely no reading comprehension. I was going into more depth on my (completely appropriate) uses of the rock-in-sunlight and longwave-laser real-world examples, which absolutely demolish your silly theories.

          • mpainter says:

            “I note that you never have any actual counterarguments.”
            ###

            My view: heat does not transfer against the temperature gradient of the skin and subskin of the sea surface.

            You imagine to transfer heat to depth against that temperature gradient and warm the sea with back radiation.

            And then you say to me:

            “And it is you who does not understand the physics of heat transfer in water.”

            Until you forget about rocks and lasers, you will be lost in the thickets and tangles of your mind.

          • Ball4 says:

            “temperature gradient at sea surface means no heat is transmitted down from interface to subskin or below.”

            Physical testing shows added KE from added incident LWIR is transmitted down into water free to evaporate so reality is otherwise mpainter. Temperatures 5+cm or so below the sea surface increase with increased incident LWIR in data you have been shown. Ignoring the testing isn’t going to make it go away mp. Just makes mp comments look silly as Ed writes.

          • Ed Bo says:

            mpainter: You say:

            “My view: heat does not transfer against the temperature gradient of the skin and subskin of the sea surface.

            You imagine to transfer heat to depth against that temperature gradient and warm the sea with back radiation.”

            You missed the very basic point I made when I said:

            “If there were no back radiation to compensate for most of this, the skin layer would be a lot cooler, and there would be more upward transfer from deeper layers, cooling them as well.

            So the back radiation does lead to warmer water below, even if the net heat transfer is still largely from the deeper layers to the skin layer.”

            This is a very important distinction missed by a lot of people who have never formally studied heat transfer and had to crunch through the differential equations of transfer. That is why I use simple analogous situations, because you obviously haven’t done these.

            So another one:

            You have a long metal bar, with one end in a fire and the other end in ice water, well insulated along the length. You have a thermocouple on the midpoint of the bar to measure its temperature there.

            Now you put the cold end of the bar in boiling water instead of ice water. The boiling water is still far colder than the fire, of course.

            What happens to the temperature at the middle of the bar? Your analysis. Your analysis says that it could not “transfer heat against the temperature gradient” and so the temperature here would not increase. Do you really believe that?

          • mpainter says:

            So now Ed Bo shifts ground from rocks and lasers to demonstrate his point of view. Metal bars now will clinch the argument.

          • Ed Bo says:

            The laws of thermodynamics are universal. mpainter doesn’t have the sophistication to understand that, so he certainly couldn’t follow the (common) differential equations underlying the proper analysis.

          • mpainter says:

            I copy my original comment at the top of the thread, the basis for subsequent comments:

            mpainter says:
            September 4, 2016 at 3:29 PM
            Tim, DWLWIR is not an input to SST, as water is opaque to LWIR. You seem to be having difficulty in grasping this principle.

            ###

            I stand on that. The physics and thermodynamics of the sea surface supports that view. Rocks, lasers, and metal bars notwithstanding.

          • Ed Bo says:

            I have definitively refuted every single aspect of your argument in detail, with specific real-world examples.

            And you just go on believing that water follows different laws of thermodynamics than everything else.

            Wow…just, wow!

          • mpainter says:

            “Specific real world examples” says Ed Bo.

            I think this translates to rocks, lasers, and metal bars.

          • Ed Bo says:

            Yes, those are part of the real world, easily testable, following the same laws of thermodynamics.

            And all you have is a fantasy world based on something you (mis)read on the internet years ago

          • mpainter says:

            Well Ed Bo, since you have offered your opinion, I will offer mine. The rock, laser, and metal bar analogies are rather inappropriate and miss by a wide margin. I find it curious that you deem them apt. Yours is the fantasy, imo.

          • mpainter says:

            To clarify, your rock, laser, metal bar analogies were inapt in their application to the physics of the sea surface.

  36. phi says:

    Joel Shore,
    Your first message does not concern at all what I said.
    For the second:
    “So, to a first approximation it remains constant as GHGs increase.”
    It is perfectly bogus for low levels of greenhouse gases.
    The radiative effect (not water vapor feedback) of GHG on convection is not at all included in the theory and models.
    Have you any evidence that this approximation is eligible to properly assess the effect of increased CO2 levels on surface temperatures?

    • Joel Shore says:

      “The radiative effect (not water vapor feedback) of GHG on convection is not at all included in the theory and models.”

      Sure it is…An increase in the lapse rate above the adiabatic lapse rate sparks convection that reduces the lapse rate back down to the adiabatic lapse rate. Your claim is nonsense. You just don’t like the fact that they are not getting the answer your ideology demands they get.

      • Bart says:

        But, an increase in convection also decreases the ERL. Overall, it is a net negative feedback on temperature at the surface.

      • Joel Shore says:

        No…Why would it decrease the ERL? The increase in convection is what is necessary to prevent the lapse rate from becoming steeper … It doesn’t make the lapse rate less steep. (Except for the fact that the moist adiabatic lapse rate is a decreasing function of temperature and that negative feedback is already included in all the climate models.)

        • Bart says:

          Joel Shore @ September 3, 2016 at 2:44 PM

          “It doesnt make the lapse rate less steep.”

          Missing the point. The temperature at the surface is the temperature at ERL plus the integrated lapse rate from there. If the ERL is lower, then the temperature at the surface decreases, because you have less distance over which to integrate the lapse rate before reaching the surface.

          “Why would it decrease the ERL?”

          Because heat is transferred to altitudes where it is free to radiate to space without having to overcome the obstacle of absorbing elements in the atmosphere.

      • phi says:

        Joel Shore,
        “Sure it is”
        No :
        “As Ramanathan and Coakley pointed out in their 1978 paper, convection is what determines the temperature gradient of the atmosphere but solving the equations for convection is a significant problem so the radiative convective approach is to use the known temperature profile in the lower atmosphere to solve the radiative transfer equations.”
        [http://scienceofdoom.com/2010/04/]

      • Joel Shore says:

        phi: In other words, they assume that convection is infinitely fast and effective at reducing any super-adiabatic lapse rate down to the adiabatic lapse rate. Assuming convection is infinitely fast at producing the heat transfer is not the same as ignoring it. Far from it.

        • phi says:

          It is not a question of speed. On the other hand, in subsidence, the gradient is not adiabatic but a compound of compression and radiative losses. And then still observed are temperature inversions in the troposphere, they have little to do with the adiabatic gradient. The hypothesis is interesting but not verified.

        • Joel Shore says:

          They don’t enforce an adiabatic gradient everywhere. If the gradient is less than the adiabatic lapse rate it is stable and only if it is greater than the adiabatic lapse rate is it unstable to convection that then reduces the lapse rate down to the adiabatic lapse rate.

        • Joel Shore says:

          And, it’s not a hypothesis that the adiabatic lapse rate is the stability boundary (i.e, that an atmosphere with a lapse rate less steep than the adiabatic lapse rate is stable to convection and an atmosphere with a lapse rate steeper than the adiabatic lapse rate is unstable to convection). That is basically the definition of the adiabatic lapse rate.

          By the way, a quick look at the IPCC report suggests that my description of how convection is handled may be somewhat out of date. It sounds like it might be more sophisticated now, but I am not completely clear on the details.

          There is, of course, a somewhat interesting recent history of claims that the mishandling of convection is what allows the greenhouse effect to persevere in the models: This claim was made by Nikolov and Zeller (https://tallbloke.wordpress.com/2011/12/28/unified-theory-of-climate-nikolov-and-zeller/) in their misguided screed a few years ago where they showed how they could put convection into the basic radiative shell model and then the greenhouse effect went away.

          The embarrassing mistake that they made, however, was to put in convection such that it drove the temperature of the Earth and the “atmospheric shell” to the same temperature (by their own description!!!), hence creating an isothermal atmosphere. They were apparently unaware that it was well-known that the lapse rate is vital to the greenhouse effect (e.g., a fact emphasized in Ray Pierrehumbert’s text). In fact, they ought to have noticed that if they simply enforced the condition that convection produces whatever heat flow is necessary to equalize the temperatures of the shell and the Earth then they would also have destroyed the greenhouse effect.

          Hence, the survival of the greenhouse effect in the presence of convection occurs precisely because convection is only effective in driving the lapse rate down to the adiabatic lapse rate but no further, a fact that I think is somewhat under-appreciated when people start talking about convection and its importance.

          • phi says:

            I am not saying that the increase in CO2 levels has no effect on surface temperatures, but that we do not know how to calculate it.

            The adaibatic lapse rate is roughly a stability limit but what matters for the radiative equilibrium is the average profile which has a radiative component and thus is directly dependent on the level of GHG.

            The average gradient is not an invariant, any imbalance does not recover only by translation of the thermal profile, the notion of forcing is inadequate and the use of a parameterized convection on the basis of empirical values can not lead to satisfactory results.

          • Bart says:

            Moreover, the gradient is only part of the story. It’s like using your car’s speedometer to tell you where you are. It can only tell you how far you’ve driven. But, to determine where you are, you also need information on where you started.

            Joel is wandering into slayer territory. They posit that, since they can determine the adiabatic lapse rate based on gravity as g/cp, it is gravity that determines temperatures on the surface. But, that is not the whole story. It is a differential relationship that requires boundary conditions to be complete. And, the boundary conditions are dependent on atmospheric composition of radiating gases, as well as their dynamic convective state.

          • Joel Shore says:

            Bart,

            There is no convection into space. A hypothesis that convection magically saves you has testable consequences, including that the upper parts of the troposphere would have to warm much more rapidly than the surface.

            I don’t think there is any evidence of that. In fact, there is still considerable debate about whether or not the upper parts of the troposphere are warming more slowly than models predict or whether, within uncertainties, they are still consistent. I have seen noone claim that the warming is actually much greater than the models predict so that that lapse rate is decreasing considerably more than expected as the Earth warms.

          • Bart says:

            “There is no convection into space.”

            No sh%t?

            “A hypothesis that convection magically saves you has testable consequences, including that the upper parts of the troposphere would have to warm much more rapidly than the surface.”

            A hypothesis that convection has no impact could be said to magically doom us. Bringing up magic is an emotive mode of argument. It conveys no information beyond subjectively emphasizing your opinion.

            There does not have to be an abrupt transition. Every incremental increase in elevation of energy bearing molecules brings them that much farther past the atmospheric filter, incrementally reducing the impedance of their emission to space. The total decrease in impedance is the integrated result across the entire atmospheric column.

          • Kristian says:

            Bart says, September 5, 2016 at 1:49 PM:

            “Every incremental increase in elevation of energy bearing molecules brings them that much farther past the atmospheric filter, incrementally reducing the impedance of their emission to space. The total decrease in impedance is the integrated result across the entire atmospheric column.”

            Yes, that is what’s being ignored here. The energy being somehow radiatively prevented from being emitted to space from one particular spot or level doesn’t thereby stay in that spot or at that level, as if “trapped”, creating excess warming. It is simply transported along, inside the bulk air itself, by atmospheric circulation, to be emitted to space ELSEWHERE:

            Air convected to the top of the troposphere in the ITCZ has a very high potential temperature, due to latent heat release during ascent in hot towers. Air spreading out at higher levels also tends to have low relative humidity, because of moisture losses by precipitation. As this dry upper air drifts polewards, its potential temperature gradually falls due to longwave radiative losses to space (this is a diabatic process, involving exchanges of energy between the air mass and its environment). Decreasing potential temperature leads to an increase in density, upsetting the hydrostatic balance and initiating subsidence. The subsiding air warms (as pressure increases towards lower levels), further lowering the relative humidity and maintaining clear-sky conditions. However, although the subsiding air warms, it does not do so at the dry adiabatic lapse rate. Continuing losses of longwave radiation (radiative cooling) means that the air warms at less than the dry adiabatic lapse rate (i.e. some of the adiabatic warming is offset by diabatic cooling).

            http://www.st-andrews.ac.uk/~dib2/climate/tropics.html

            That’s how you get OLR charts like this:
            https://okulaer.files.wordpress.com/2016/04/olr-trop.png

            Much of that fiery red emission over the subtropics comes from energy brought there from the central tropics. So even if the OLR at the ToA is much lower around the equator, that doesn’t mean the energy is “trapped”. And it certainly doesn’t mean that it therefore causes the surface T_avg below to be higher in any way. In fact, the opposite is true. More H2O in the tropospheric column tends to make the surface T_avg below lower. For many reasons.

  37. alphagruis says:

    phi said

    The problem is that this notion of forcing makes sense only with the assumption of the invariance of the thermal gradient.

    Nope, the notion of forcing has nothing to do with an “invariance of the thermal gradient”. Forcing is just a matter of imbalance in the TOA energy fluxes. It makes sense whatever the temperature gradient might ever do.

    The so-called “invariance of the thermal gradient”, on the other hand, is the consequence of convective equilibrium always established in an atmosphere with enough GHG’s to make the troposphere unstable with respect to convection.

    Now you’ve been told repeatedly before how silly your “physics” actually is in a French forum.

    Good luck in your attempt to draw further attention to your fancy views about atmospheric thermodynamics.

    • phi says:

      Mon cher Tsih,
      The notion of forcing is unknown in thermodynamics simply because a change in structure is not the same as energy input. You mingle heating and insulation. This confusion is of no consequence here only if we assume that the temperature gradient is an invariant.

  38. mpainter says:

    Toneb,you say: “Nope.”
    But you link below says “Yep”.
    Perhaps you should study your links. Yours link below accepts the verifies my statement quoted by you.
    It does this in order to present its arguments.
    ####

    You say: “You neglect conduction and turbulent mixing.”

    Nope. You neglected to read your link which made it explicit that heat was conducted from depth to the interface, _not_the_other_ way_.
    Concerning turbulent mixing. Whether the turbulence? Wind? That cools. Wave? The ocean swell does not break the coherence of the “skin”. Unless there is a storm…but that’s wind.

    Your claim of turbulent mixing is nothing but arm waving, I’m afraid.

  39. mpainter says:

    Toneb, you speak of land surface in rebuttal of my statement that addresses the the physics of the sea surface. Why the confusion? The two are different, with different thermodynamics.

    Your link to the Curry blog is a chain of bald assertions. I’m not impressed.

    You need to explain how your second link and quote answers my comment, above. I see nothing in it that’s contrary to my statement.

  40. mpainter says:

    Toneb, your link to RealClimate purports to show a temperature difference between the “skin” and the ocean “bulk” (5 cm depth) under IR forcing, showing a scatterplot from which they derive a result of 0.00002 per W/sq m temperature difference.

    Not very impressive to a man of science. Particularly since the study gave no temperature data, no measurements data, did not described their procedures or methods (except inadequately), gave no rationale for their selection of 5 cm depth as “bulk”. Lots of unanswered questions.

    Most notably, they gave no indication as to whether the skin had the higher temperature, or the “bulk”.
    Also, the measurements were taken under cloudy conditions, which would have altered insolation and they wholly neglected to address this important effect, as they took both nighttime and daytime measurements (seemingly around the clock).
    This is not what I call good science. Plus the result of 0.00002 C per W/sq m had no error bounds.

    • Ball4 says:

      The graph says about 0.1K difference SST skin plotted against LW clear and cloudy sky mpainter at different local times, just the same as Dr. Spencer confirmed with his test at night with clear and added cirrus, maybe a little deeper calm water depth. Their M-AERI equipment is thoroughly described in published papers contrary to mp comment.

      You are misconstruing facts to line up with your political views mpainter, there are other sites for politics. This site proprietor does real custom testing and basic analysis like top post.

      • mpainter says:

        B4, text clearly states that water temperature measurements were made under cloudy conditions for the IR effect. You need to strive for better comprehension. You missed by a mile.The scatterplot was derived from all cloudy measurements.

        B4,:
        “You are misconstruing facts to line up with your political views mpainter, there are other sites for politics. This site proprietor does real custom testing and basic analysis like top post.”

        This accusation has no basis.You should retract that and apologize. I don’t think Dr. Spencer would appreciate your involving him in such a scurrility.

      • Ball4 says:

        “The net forcing is negative as the effective temperature of the clear and cloudy sky is less than the ocean skin temperature, and it approaches values closer to zero when the sky is cloudy.”

        Reading is good for you mp. And not scurrilous at all, Tim F. also points to mpainter bizarre repeated comments about SST from insolation only:

        “No temperature anywhere is determined by any one power input alone. Temperature is a response to ALL inputs as well as ALL outputs!”

        Read up on the 1LOT,2LOT atm. radiation science mpainter, as Dr. Spencer would advise, try to add to the discussion, not subtract as usual.

        • mpainter says:

          The net forcing is negative as the effective temperature of the clear and cloudy sky is less than the ocean skin temperature, and it approaches values closer to zero when the sky is cloudy.

          You misunderstand once more B4. That refers only to the gradient between skin temperature and air temperature, under clouds and clear sky both.
          Good grief.

          Once again, the study clearly states that water temperature measurements were made under cloudy conditions. That was the whole purpose.
          Good grief.

          I really doubt that you have comprehended any of the study.
          Good grief.

          You are a scurrilous OOO. Adios. Please don’t respond to any of my comments in the future.

          • mpainter says:

            The first paragraph of my comment was in quotes, which dropped off.

          • Ball4 says:

            Nope mp, it refers as “The figure below shows just the signal we are seeking.” Good grief. Try to quote the authors.

            Nope mp, the study says: “we use the natural variations in clouds to modulate the incident infrared radiation at the sea surface” Good grief. Try to quote the authors not use your wording to suit your own purposes.

            No doubt that mpainter has comprehended little of the study. Try to comprehend what Tim F. is trying to tell you also.

            “Please dont respond..”

            Making up your own rules won’t win on a blog, only in a knife fight mp, experimental science wins on this site. Don’t bring a knife to a gunfight.

          • mpainter says:

            Ball4 says:
            “Making up your own rules wont win on a blog, only in a knife fight mp, experimental science wins on this site. Dont bring a knife to a gunfight.”

            He really did say that.

          • Ball4 says:

            For once, mp pays attention. Keep it up!

        • mpainter says:

          You are verminous trash.

    • Toneb says:

      “Toneb, your link to RealClimate purports to show a temperature difference between the skin and the ocean bulk (5 cm depth) under IR forcing, showing a scatterplot from which they derive a result of 0.00002 per W/sq m temperature difference.”

      No it doesn’t purport anything.

      It shows an actual set of observations done by the NZ research vessel Tangarova.

      Here is the full paper for you to *study*.

      BTW: Due to your latest nasty comment to Ball4 on the (current) bottom of this thread, I will no longer soil myself by conversing with you.
      TaTa

  41. Bart says:

    FTA: “If the model had dozens of atmospheric layers all interacting, it would produce much higher surface temperatures, and much lower temperatures in the upper atmosphere, producing a strongly super-adiabatic temperature profile (Manabe and Strickler, 1964). This is what causes atmospheric convection, which provides a net transport of heat from the surface to the middle and upper troposphere (not contained in this radiation-only model).”

    Well, that’s the key, isn’t it? It indicates that convection provides a negative feedback. So, once you’ve gotten the atmosphere stirred up, it is no longer assured that an incremental increase in concentration of a particular greenhouse gas will produce a corresponding incremental increase in surface temperature of any significance.

    • Bart says:

      And, that’s without even considering the negative feedback of cloud formation.

    • Joel Shore says:

      Bart,

      The fact that convection reduces the greenhouse effect is old news…and, of course, you have to go back to 1964 (or probably earlier) to find radiation-only models used for anything other than basic illustrative purposes.

      • Bart says:

        Yes, it is old news. But, that does not mean that it is well understood. It may be thought it is well understood, but the experiment is still running, and the results appear to indicate that the understanding was off.

        Moreover, it is important because it is claimed that AGW is a simple application of well understood physics, and therefore assured. But, it is not assured, because there are viable physical avenues by which the radiative dynamics can be short circuited, and the overall effect becomes negligible.

      • Joel Shore says:

        “but the experiment is still running, and the results appear to indicate that the understanding was off.”

        Not really… http://www.realclimate.org/index.php/archives/2016/08/unforced-variations-aug-2016/

        “But, it is not assured, because there are viable physical avenues by which the radiative dynamics can be short circuited, and the overall effect becomes negligible.”

        Nothing is assured in science. It is not assured that the next time I throw a ball up in the air, it will come back down. What is assured is that most of those who deny AGW do so for ideological reasons and will continue to do so and pretend that it is really for valid scientific reasons, which is why there will be continue to be a large gulf between what these people conclude from the evidence and what the respected scientific authorities (NAS, AAAS, AMS, AGU, …) conclude from the evidence.

        • mpainter says:

          Ah, so! The science is settled. Joel Shore proclaims it so and produces a list of approved “respected scientific authorities” which proves it.
          Joel, I have learned that in climate science, its best not to believe in “infallibility”. That’s for religion, not science.

        • Kristian says:

          Well, Joel, the observational evidence from the real Earth system unequivocally shows us that the radiative imbalance at the ToA is not a result of an “enhanced GHE”, but by an increase in solar input (ASR). The warming is natural. An increase in atmospheric CO2 does not and cannot warm the real world. Source: The real world.

        • Bart says:

          “It is not assured that the next time I throw a ball up in the air, it will come back down.”

          The standard obfuscatory technique of comparing a tenuous hypothesis to a well established body of theory and observation to make it appear more credible. Meh.

          I would bet my life that a ball thrown in the air, in the absence of any artificial intervention, will come back down. Would you bet your life that AGW will survive the test of time? If so, I would question your sanity.

          “Not really”

          Really. This link is blatant fudging. All it is, is shifting the baseline after the fact to appear to make the curves line up with one another somewhat. You can always do that. It is meaningless.

          And, of course, the blatant fudging in GISTEMP that disappeared the “pause”.

          Joel, I respect your intellect. We have interacted many times over the years, and you have often provided cogent insights. You sold me on the theory and reality of the “Ozone Hole”, and for that, I thank you.

          But, anyone who buys these blatant misrepresentations is either a fool, a knave, or has sold out for what they believe is a greater good. I give you the benefit of the doubt and assume you are the latter.

          I would only ask that you engage in some introspection, and ask yourself if this debasement of science is really going to serve the greater good in the long run, and whether you should be supporting it? Faustian bargains never work out as you expect or hope.

    • mpainter says:

      Bart, interesting study from U of Bremen (December, 2015, Schmithusen, et al) of the negative (!) GHE of central Antarctica. You might be familiar with it. This region is high, 4 km, more or less, and the driest place on earth. Apparently the surface radiates to space. There is little or no lapse rate, and this can even be negative, that is, the surface colder than the stratosphere.

      I learned from this study that CO2 emits to space from the stratosphere. This fact challenges the meme that CO2 has a cooling role in the upper troposphere (the so-called “effective radiating level”).

      The study supports my view that cooling in the upper troposphere (emittance to space) is achieved by atmospheric water, not CO2. By this, the role of CO2 in the GHE is diminished to insignificance.

  42. Norman says:

    Postma’s at it again

    http://principia-scientific.org/simple-time-dependent-model-refutes-atmospheric-greenhouse-effect/

    I guess this is were most the bad science we read on Roy’s blog come from.

    Postma: “Its not logically self-consistent, and it cant be self consistent because it isnt logically consistent with existence in the first place with the entry postulate that backradiation can cause further temperature increase than what the source initially provided.”

    That is why his reasoning is flawed and he leads so many away from solid physics.

    The source does not just one time provide energy. It is continuously adding energy and if you slow down the loss of energy the surface will reach a warmer equilibrium state than in a case without such a slow down of energy loss.

    Really sad that Postma studied physics at some University and he is so poorly educated in understanding radiation transfer process.

    • gbaikie says:

      — Norman says:
      September 3, 2016 at 4:52 PM

      Postmas at it again

      http://principia-scientific.org/simple-time-dependent-model-refutes-atmospheric-greenhouse-effect/

      I guess this is were most the bad science we read on Roys blog come from. —
      I doubt many read Postma. He says:
      “If you modify the Excel sheet to match that, then the final surface temperature is -47C! Why so cold?”
      Which made me think, if answer was say 15 C, then is that what
      is wanted?
      Or what is the number one might be looking for if you imagining that some point in the atmosphere is your surface.
      Not that these are important questions. Then I wondered, do they model for the actual surface of Earth.
      Which reminded me, I once wondered what is the actual temperature of the surface of Earth. We do the surface air temperature [5 feet in the shade] but what about the surface temperature?

      There is problems measuring the actual surface of Earth. One problem is, no one cares what it is. And other problem is the Earth ocean and land. The ocean surface temperature is roughly the same as 5 feet up in the shade of white box and the land is not. And one say that in terms of Earth average temperature of 15 C- the land temperature- whether surface air or ground, it’s is not important. All need for 15 C average global temperature is fact that tropical oceans are large portion of Earth and they are fairly warm all the time. Or ocean surface temperature is the 15 C average temperature for global surface air temperature- whatever difference that was the temperature of the ground on the land- it doesn’t change the number- or it’s still around 15 C.

      Of course if you thought that GHE effect actually increases surface of the land [not surface land air temperature] you might want the actual surface of the land to be measured
      Or if you were a farmer, the actual surface of the land, would be important- as determine when to plant crops in the early spring.

      But if land temperatures made any difference- surface air or ground surface- then one could look at difference of surface air temperature vs ground surface temperatures. Or if Earth was covered by 70% land and 30% ocean, it would matter in terms of an average global temperature.
      Or we have vast area of land which is 70 C [158 F] daytime high, and don’t have vast area of land with air temperature routinely reaching as high as 50 C [122 C].
      Or again without getting complicated- ground is about 20 C warmer
      than air surface temperature. So night and day average global increase of +10 C if measuring the ground vs air.
      Of course if getting a bit complicated, a planet with 70% land is a world with a huge amount of deserts- and deserts are much cooler at night. Or Earth has somewhere around 30 million square km of desert- and 70% land world would easily have more than 150 million square km of desert. So 70% land world would not have average global temperature of 15 C + 10 C, instead it’s average global temperature would be about 0 C [or less]. Or if measured ground temperature it would be global average of about 5 C.
      Or 70% land world would have most of tropics as land area- let’s say 70%. And so 70% of tropic would have 70C daytime temperature
      ground temperature and 50 C surface air and more drier air than normal deserts on Earth. Night time average about 0 C- ground getting bit cooler than air- average air: 25 C, and average ground about 30 C.
      And lacking an ocean which can warm rest of world, outside tropics has much lower average temperature.

    • Kristian says:

      Norman says, September 3, 2016 at 4:52 PM:

      The source does not just one time provide energy. It is continuously adding energy and if you slow down the loss of energy the surface will reach a warmer equilibrium state than in a case without such a slow down of energy loss.

      But Norman, you are specifically NOT reducing the energy loss from the surface in the ridiculous and counter-physical “back radiation” explanation of the so-called “GHE”. You are rather increasing the energy INPUT to the surface to directly raise its temperature, upon which the surface responds by increasing its energy loss.

      That is a perfect description of an absolute temperature rise by “extra heating”, not by “insulation”.

      You still don’t see this. It is not the EFFECT itself that violates the 2nd Law of Thermodynamics, only this particular attempt at EXPLAINING it … It clearly says that it is not the reduction in net LW that causes the temperature to rise. The net LW is only reduced as an effect of the ‘real’ cause, namely the increase in DWLWIR, just as the sfc temp increase (and consequently the increase in the sfc energy loss) is simply an effect of the ‘real’ cause, the increase in DWLWIR.

      This is nonsense physics! The DWLWIR EXPLANATION of surface temperature rise is, not the insulation effect itself.

      • mpainter says:

        Kristian makes a telling point. Energy budget diagrams show “back radiation absorbed by the surface”. Thus enhanced GHE adds more “back radiation absorbed by the surface”. As Kristian says, this means heating through radiative energy added, not through heat retained through insulation or retarding effect.

        • Ball4 says:

          Kristian to Norman, “you are specifically NOT reducing the energy loss from the surface in the ridiculous and counter-physical back radiation explanation of the so-called “GHE”.”

          Yet that is exactly what Kristian does with his (- T_atm^4) term, specifically reducing the energy loss from the surface. This term is the all sky emission to surface which isn’t heating the surface (- T_atm^4 is emitted from a lower T) just adding energy (day and night). Only the sun is heating the surface (day).

          —–

          mpainter, “”back radiation absorbed by the surface”. As Kristian says, this means heating through radiative energy added”

          No heating from – T_atm^4 term mpainter, this term just means radiative energy added (night and day) reducing sfc energy loss, only the sun is heating the surface (day).

          • mpainter says:

            Depart, verminous one

          • Norman says:

            Ball4

            Yes that is what is going on. DWIR adds energy to the Earth’s surface but not heat. They can’t see the two way flux so they attack it. It is the same effect but two way flux makes atomic sense since vibrating molecules on the Earth’s surface do not slow down or reduce their IR output (this would only occur if the temperature of the surface dropped). Heat flows one way, energy flows in many ways and it adds and subtracts from the total.

            I read Postma’s page and none of them understand the process and I am amazed that Postma gets it wrong. I can understand the people who post on his blog that have no physics background and are thinking based on what they think is correct but Postma is a strange fellow.

          • Ed Bo says:

            Ball4:

            You are debating with people who don’t know the difference between gross and net flows (exchanges) to be able to make change properly as a cashier.

          • Kristian says:

            Ed Bo says, September 4, 2016 at 11:48 AM:

            You are debating with people who dont know the difference between gross and net flows (…)

            No, apparently he’s debating people who DO know the difference between gross and net flows. That’s exactly why we point out how you should treat them and how you can’t treat them, thermodynamically. But it just won’t sink in, will it?

          • Ball4 says:

            Kristian, the gross LW energy up from the surface is net of the LW energy down to the surface as your own eqn. shows. Very simple when incoherent photons do not interact. No heating, the sun does that. Now consider what happens Tmedian as the atm. opacity increases or decreases as shown in Dr. Spencer spreadsheet that does the net for you.

          • mpainter says:

            Norman, there you go again : “DWIR adds energy to the Earths surface but not heat.”
            ###
            You imagine that back radiation can be absorbed without being absorbed. You are a wonder. Also, in the sense that heat is a property of matter, heat and kinetic energy are equivalent. Please, no stupid semantic arguments. I know some will define heat as a transfer process, but please do not quibble over semantics.

          • Ball4 says:

            Norman, “I am amazed that Postma gets it wrong.”

            I am not as he does no confirming experiments as does Dr. Spencer.

            Also it is mpainter that imagines that back radiation can be absorbed without being absorbed not Norman.

            —–

            mpainter, “heat is a property of matter”

            mp, per Clausius KE is a property of the constituents of matter, heat is the measure of their KE.

            —–

            John O’Sullivan: “Sophists are those who conjure with words to deceive and throw insults when out of their depth.”

            Postma: “If you and your cohorts know these things, then you’re frauds and clandestine operatives. If you don’t know them, then you’re incompetent.”

            Also note the title of Postma’s blog, “Climate of Sophistry”, you can get plenty of sophistry over there as they do no testing, just conjure with words.

          • Norman says:

            test?

          • Norman says:

            mpainter

            Example:

            A surface molecule absorbs a DWIR and starts to jiggle more which is picked up by the surrounding molecules and it would seem that the overall kinetic energy would go up so why doesn’t it? That is because for this one molecule that absorbed an IR photon and increased its local kinetic energy there were two molecules elsewhere on the surface that emitting photons (same time) and lost more kinetic energy than the surface gained by the one absorbed photon. With this process going on trillions plus times per second you have a net loss of energy to the cooler atmosphere.

            But the more DWIR you have the less energy you lose from your surface. If the DWIR is equal to the surface UPIR (cloudy night) you lose no energy as the photon exchange is equal.

            The point that Postma and his group never understand is the surface has another source of incoming energy.

            So now you have a situation where you have the same one DWIR photon being absorbed by the surface and the two UPIR photons of similar energy. But now we add the solar input say of two additional DWSW photons to the surface (Photon energies would be all over the spectrum, I do know this but I am trying to get you to understand the basic concept). The surface is now receiving 3 photons worth of energy but only giving up two. What happens now, the kinetic energy of the surface overall is now warming since it is receiving more energy than it is losing. It will continue to warm until it emits the same number of photons it is receiving. It will warm to a temperature where the kinetic energy is high enough to emit 3 photons at the same rate it is receiving 3 photons. Equilibrium.

            Not sure if you read this long post or understood my point. Hopefully you do and did. Take care.

          • mpainter says:

            Norman, I think I understand. You trying to say that back radiation is absorbed by the surface, but it doesn’t warm the surface.
            Is this what you mean?

          • Kristian says:

            mpainter says, September 4, 2016 at 2:38 PM:

            Norman, I think I understand. You trying to say that back radiation is absorbed by the surface, but it doesnt warm the surface.

            Yes. It is absorbed by the surface and thus makes the surface temperature higher. But it doesn’t warm the surface. LOL!

          • Ball4 says:

            Kristian, your ( T_atm^4) is absorbed all night yet doesn’t make the surface warmer. Surface cools all night. Shown by NOAA ESRL.

          • Norman says:

            mpainter

            YOU: “Norman, I think I understand. You trying to say that back radiation is absorbed by the surface, but it doesnt warm the surface.
            Is this what you mean?”

            Yes to your question. It only makes sense with simultaneous activity. If you look only one way it will be most absurd.

            http://www.esrl.noaa.gov/gmd/webdata/tmp/surfrad_57cc97de65d7d.png

            The link is the support of my position. The DWIR is less than the UWIR. The DWIR is reaching the surface and picked up by detector so it is a real energy flux. But it is less than the energy the surface is continuously emitting, so the surface will cool. It is losing joules at a greater rate than gaining them. The backradiation will allow less joules from leaving so if you have two cases, one with DWIR and one without, the one with DWIR will be be at a warmer equilibrium temperature (both are receiving the same input energy from an external source…with Earth that would be solar energy).

            The CERES site also confirms all this information. The only way around is to make up a belief that the instruments are not detecting photons from the sky.

          • Kristian says:

            Norman says, September 4, 2016 at 3:58 PM:

            Yes to your question. It only makes sense with simultaneous activity. If you look only one way it will be most absurd.

            Yes, you’re right. It would be most absurd. And yet, this is exactly how the IPCC portray the process. By pretending the two streams are independent and separate ‘fluxes’, thus placing the one on the INPUT side and the other on the OUTPUT side of the surface budget equation. As if the one caused the other one.

            Again, it’s the “back radiation” EXPLANATION that is absurd, not the insulation EFFECT.

          • Norman says:

            Kristian

            I think one problem with our ongoing debate is your mind is not able to process multiple processes taking place at the same time. You have a serial mind with one event taking place at any time.

            YOU wrote: “Yes. It is absorbed by the surface and thus makes the surface temperature higher. But it doesnt warm the surface. LOL!”

            You are unable to grasp that the DWIR can be absorbed by the surface (primarily depends upon the surface material, some absorb IR better than others) yet not make the surface temperature higher or warm the surface. As the surface is gaining energy from the DWIR it is losing energy at a greater rate in the UPIR. Look at CERES data and it will show this to you at whatever location you want to examine. Dry Sahara, wet Congo, cold Antartica.

          • Norman says:

            Kristian

            YOU: “Yes, youre right. It would be most absurd. And yet, this is exactly how the IPCC portray the process. By pretending the two streams are independent and separate fluxes, thus placing the one on the INPUT side and the other on the OUTPUT side of the surface budget equation. As if the one caused the other one.

            Again, its the back radiation EXPLANATION that is absurd, not the insulation EFFECT.”

            Not absurd at all. I think you are way to hung up on Cause/Effect semantics. Roy Spencer clearly shows how both are cause and effect.

            He starts his simulation at zero and lets it go from there. The Solar input warms the surface and it starts to weakly emit (in Roy’s simulation). The emission is absorbed by the GHG in the atmosphere which slowly warm (he has taken out all other mechanisms like convection, evaporation, conduction, clouds etc just to simplify the concept and show how the process can work…have you clicked on his simulation and played around with the variables?).

            So now they are both causes of the temperature and effects. The warmed atmosphere weakly emits to the surface of which this small amount of energy is then absorbed by the surface. The surface always being warmer than the atmosphere will always be emitting radiant energy at a greater rate than it receives from warming atmosphere. But nothing is cooling at this early stage because the surface is receiving a continuous input of solar energy (240 W/m^2 is what Roy used).

          • mpainter says:

            Norman, my response below.

          • Kristian says:

            Norman says, September 4, 2016 at 4:17 PM:

            I think one problem with our ongoing debate is your mind is not able to process multiple processes taking place at the same time. You have a serial mind with one event taking place at any time.

            So now you’re my mental coach? Don’t worry about me, Norman. I know perfectly well what you’re trying to say. I understand perfectly the “back radiation” argument. You should rather worry about yourself and your stubborn unwillingness to listen and read.

            This is very simple: The “back radiation” EXPLANATION violates the 2nd Law, NOT the insulation effect it tries to explain.

            How many times does this have to be repeated? This is nonsense physics:
            https://okulaer.files.wordpress.com/2014/10/drivhuseffekten.png

            You say the “back radiation” doesn’t make the surface any warmer, because there is more radiation going out of the surface. Then how, looking at the diagram above, do we get from 232 to 289 K? From a pure solar equilibrium temp to the observed global mean …?

            The surface temperature rises in absolute terms, and it stays there, Norman, as the “back radiation” is absorbed. In your scenario the “DWLWIR” DOES heat the surface some more. Directly and all by itself. You cannot escape this fact, no matter how much you try to redirect the discussion. Just look at the diagram. THAT’S exactly what happens.

            You simply cannot EXPLAIN insulation as an extra INPUT of energy to the heated object. Because there is no extra input of energy.

            I quite recently explained this to you on another thread. But you keep ‘forgetting’ and keep asking the same questions over and over again:
            http://www.drroyspencer.com/2016/08/simple-experimental-demonstration-that-cool-objects-can-make-warm-objects-warmer-still/#comment-222099

            The way you can think of this is through Clausius’ principle of “compensation”: Whenever some photons originating from a cooler body are absorbed by a warmer body, there are ALWAYS more photons simultaneously being emitted by the warmer body to the cooler. This ensures that you will NEVER be able to DETECT a rise in “internal energy” [U] in the warmer body. Simply because it never happens. And since it NEVER occurs that the “internal energy” of a warmer body faced with a cooler one at any point in time increases even incrementally, even for a split second, then it makes fundamental sense to say that macroscopically/thermodynamically the cooler body never actually TRANSFERS energy to the warmer body. The exchange of energy between the two is synchronous and instantaneous [, throughout the radiation field], and the probabilistic average of all the microscopic absorp tion and emission events, that individually cannot be accounted for, is a macroscopic NET MOVEMENT, a TRANSFER, of energy from the warmer to the cooler body only.

            Yes, in a way it’s all semantics, but it’s still important to make the point, because what you do when you place the “DWLWIR” on the INPUT side of the energy budget of the surface, is that you make it seem, and hence confuse yourself into believing, that the energy from the cooler atmosphere to the warmer surface actually DOES in fact increase the surface “internal energy”, that the surface absorp tion of the “DWLWIR” constitutes a separate and independent process or event from the surface emission of the “UWLWIR”, and that the former process/event actually contributes to CAUSING the latter to happen, because seemingly, according to your logic, the surface needs to absorb both the solar flux (165 W/m^2) and the “DWLWIR” (345 W/m^2) in order to be ABLE to emit 398 W/m^2 worth of “UWLWIR”, from a temperature of 289K, rather than one of a mere 232K (from 165 W/m^2 alone).

            Which is turning an actual temperature EFFECT the “DWLWIR” into what might be perceived as a temperature CAUSE. Causality is turned right on its head …

  43. Kristian says:

    From the original exchange upthread:
    http://www.drroyspencer.com/2016/08/simple-time-dependent-model-of-the-atmospheric-greenhouse-effect/#comment-222431

    Norman,

    Let’s go back to our hypothetical planet. In the initial steady state, there is no atmosphere on top of its global surface, and so the global surface has simply equilibrated with the average radiant heat input (ASR, net SW) from the planet’s mother star, meaning, its average radiant heat output (OLR, net LW) is equal to it. In our particular case, let’s now say that the average ASR value is 296 W/m^2, and so, in the steady state, this is also the average OLR value: 296 W/m^2 IN = 296 W/m^2 OUT. This state is ideally attained at the point where the planet’s global surface T_avg has reached ~269 K. (This, of course, relies specifically on two purely hypothetical conditions to abide: The surface is 1) a blackbody, and 2) isothermal.)

    We now place a massive – and very much radiatively active! – atmosphere on top of this equilibrated planetary surface. Now, just as the surface before it could be considered to start its original journey toward its original steady state temperature (269 K) from a hypothetical initial temperature around absolute zero (or 2.7 K, rather), so could the atmosphere. We simply want to see what happens as energy accumulates inside this massive atmosphere, gradually warming it.

    So to begin with, before any energy has managed to be transferred as heat from the surface to this new atmosphere, the surface radiant heat loss is: q/A = σ (T_sfc^4 – T_atm^4) -> σ (269^4 – 2.7^4) -> 296 W/m^2, IOW just what it was before the atmosphere was placed on top of it. And remember now, this is a fully radiatively active atmosphere – it is able to absorb and emit EM radiation. The only problem is that it’s still too cold.

    But what happens as this atmosphere now absorbs more and more energy from the surface (and from the local sun), thermalising it and gradually warming from it? Its temperature rises beyond that of space itself. And as a simple consequence of this, the atmosphere is now turned into an insulative layer, basically interposing a thermal barrier between the solar-heated surface and the absolute coldness of space.

    As the atmosphere warms, its apparent DWLWIR to the surface increases. But the atmosphere warms, not from apparent, thermally generated ‘radiant fluxes’, but from the absorbed and thermalised energy transferred to it as heat from the surface and directly from the sun. And what happens when the surrounding temperature of a constantly heated object/surface all of a sudden increases? The temperature difference between the object/surface and its surroundings is reduced. And what does this lead to? It leads to a reduction in the rate of “heat loss” from the object/surface in question. This is true whether the mode of heat transfer happens to be ‘radiative’, ‘conductive’ or ‘convective’. And so, if we assume that the rate of incoming heat to the object/surface remains unchanged, then energy will accumulate (because Q_in > Q_out) and the object/surface will necessarily warm as a result, until its heat balance is restored (Q_in = Q_out). And what happens when the surface temperature rises as a consequence of this process? Its apparent UWLWIR increases. Which makes the (radiant) heat loss rate (DWLWIR minus UWLWIR) of the surface go up again. And so the surface radiant heat loss rate itself could theoretically stay constant during the entire warming process towards the new steady state temperature. It would be 296 W/m^2 in the initial state (296W/m^2 – 0W/m^2, 269 vs. 2.7 K), and it could be 296 W/m^2 in the final (steady) state also (say 398W/m^2 – 102W/m^2, 289 vs. 206 K).

    The thing, though, is that once the massive atmosphere is put on top of the solar-heated surface, then the whole surface situation changes. Moving towards the new steady state, the atmosphere will 1) make the planet’s albedo increase substantially, and 2) absorb for itself a significant portion of the incoming heat input from the sun, so that it never manages to reach the actual surface at the bottom. This will reduce the average ASR at the surface from the original no-atmo value of 296 W/m^2 to a final +atmo value of a mere 165 W/m^2. Which would mean that about 44 % (!) of the original solar heat absorbed by the global surface is now somehow made unavailable to it by the very presence of the radiatively active atmosphere resting on top of it, either reflecting it back out to space or absorbing it for itself, before it could ever reach the surface.

    This situation alone would reduce the potential steady-state surface radiant heat loss from 296 to a maximum of 165 W/m^2. But would it thereby necessarily change the surface T_avg also? No. Because the heat loss (the net LW) is only constrained by the temperature DIFFERENCE between the surface and the atmosphere, not by the individual temperature of the surface and/or the atmosphere. So we could go from 296 minus 0 W/m^2 (net LW: 296 W/m^2) in the initial steady state (at 269 vs. 2.7 K), to 296 minus a potential 131 W/m^2 (net LW: 165 W/m^2) in the final steady state (at 269 vs. 219 K).

    However, it doesn’t end there. You see, more things change at the surface once the massive atmosphere is put on top of the solar-heated surface than just the total heat balance (moving from Q_in(296W/m^2)=Q_out(296W/m^2) to Q_in(165W/m^2)=Q_out(165W/m^2)). The surface heat budget after all also stops being a purely radiative one. And this fact is an extremely important fact to appreciate, because it has obvious implications for the surface radiant heat loss, which used to make up 100 % of the total. It won’t anymore. It will naturally have to make room for other losses, NON-radiative ones. IOW: It will, by physical necessity, become significantly smaller.

    And so this really changes the whole narrative. The surface radiant heat loss ISN’T reduced because of a rise in the effective atmospheric temperature leading to a rise in apparent atmospheric DWLWIR to the surface. It is simply reduced because there is less ‘need’ for it, because other heat loss mechanisms than the radiative one are also now contributing to the total.

    So you see, the absolute magnitude of the surface radiant heat loss is thoroughly constrained first by the heat INPUT to the surface (the ASR), then by the (effectiveness of the) other heat loss mechanisms at work. It can’t be determined simply according to some perceived atmospheric level of IR opacity.

    In the end, we might have a situation where, after having emplaced a massive atmosphere around our hypothetical planet, the surface steady state corresponds to a Q_in = Q_out of only 165=165 W/m^2 (rather than one of 296=296 W/m^2), where the radiant part is further reduced to, say, 53 W/m^2. And so we’re left with the following inescapable apparent DWLWIR-UWLWIR relationship: 398 – 345 W/m^2 (net LW: 53 W/m^2), at 289 (T_sfc) vs. 279 K (T_atm). But this doesn’t tell us anything about how we got from a surface T_avg of 269 to one of 289 K. All it tells us is that 1) the surface steady-state T_avg happens to be 289 K, and that 2) the surface steady-state radiant heat loss happens to be 53 W/m^2. 289 K -> 398 W/m^2, and 398 – 53 = 345 W/m^2. This is exactly how a pyrgeometer would compute the apparent “sky radiation” (DWLWIR). But it tells us nothing about why the radiant heat loss happens to be 53 W/m^w, nor why the surface T_avg happens to be 289 K. Remember how, when we first placed the atmosphere on top of the solar-heated surface, the heat input to the surface from the sun was 296 W/m^2, and there were no other heat loss mechanisms in operation except the radiative one. Moreover, the no-(or pre-)atmo steady state surface temperature was 269 K (296W/m^2 IN = 296W/m^2 OUT), and this was also the initial situation as the massive, radiatively active atmosphere (at 2.7 K) was placed around the planet – the DWLWIR was practically zero, because the atmosphere was too cold. Then several things happened: i) the atmosphere started warming (from absorbing energy transferred to it as heat), ii) the heat input to the surface was reduced, and iii) other heat loss mechanisms besides the radiative one became available and operative as the system still grew warmer (a result of the warming atmosphere). And so, from the initial to the final steady state, we went from a surface net LW (radiant heat loss) shedding 296 W/m^2 to one at a mere 53 W/m^2. Meaning, we went from a temperature difference of [269-2.7=] ~266 K between the surface and the APPARENT “effective atmospheric level of downward radiation” to one of [289-279=] 10 degrees. Does this mean that the atmosphere somehow got immensely more opaque to surface IR, lowering the effective level of “sky radiation” to the surface by this huge amount, during the journey from t_i to t_f? No, of course it doesn’t. Here’s what happened in between: i) the atmosphere got warmer, meaning, the temperature difference between the surface and the layers of air above it grew steadily smaller; ii) the overall heat input to the surface from the sun, the ASR (net SW), grew steadily smaller, and so the surface target output value naturally decreased with it; iii) the radiative share of the total surface heat output dropped from its initial 100 %, because of NON-radiative heat loss mechanisms growing to prominence …

    To conclude:
    1)
    The increase in DWLWIR is an apparent radiative effect of the atmosphere warming. When the atmosphere warms beyond space, from absorbing, thermalising and storing up energy transferred to it as heat from the surface and directly from the sun, the temperature difference between the surface and its surroundings decreases, and so its total heat loss is naturally reduced. This forces the surface to warm so as to restore its heat balance (the heat input is assumed constant). Yes, if the atmosphere in question is completely radiatively inert, it (or the main portion of it) will eventually be thermodynamically disconnected from the surface, and so space can still be considered the surface’s only significant cold reservoir. But as soon as you make the atmosphere radiatively active, you will connect it thermally with the surface, and so now the atmospheric temperature will directly affect the total heat loss from the surface, which will be greatly reduced at any given surface T_avg relative to the former situation, once the atmospheric T_avg rises above the ‘temperature’ of space. You might fool yourself into thinking that it is in fact the increase in the apparent atmospheric DWLWIR itself that forces the surface temperature to rise in this situation. But it’s not. The absolute TEMPERATURE rise is the CAUSE. The DWLWIR is but a TOOL enabling the atmospheric temperature to connect with the surface temperature in the first place. When the atmospheric temperature rises in this situation, it will then simply be able to affect the surface temperature, because now the atmosphere is thermodynamically connected with the surface, and so it effectively replaces space as the surface’s thermal surroundings, meaning that, as it warms, the temperature difference between the surface and its surroundings goes down (this couldn’t happen with only the vacuum of space around, since a vacuum cannot warm). And as the temperature difference between the surface and its surroundings goes down, there will be a decrease in both radiant, conductive and evaporative/convective heat loss from the surface. Forcing a surface temperature rise.

    The increase in DWLWIR is simply one expression of this decrease in temperature difference between the surface and its surroundings, thus of the reduction in surface heat loss.

    2)
    The steady-state magnitude of the surface radiant heat loss is to a certain extent simply a function of the steady-state magnitude of the non-radiant heat losses (conduction and evaporation/convection), which will inevitably start ‘eating into’ the radiant portion of the total as soon as a massive atmosphere is in place on top of a solar-heated planetary surface. At some point along the continuum – from 100 to 0 % – a balance will be struck. It simply depends on how effective the radiant vs. the non-radiant losses are at ridding the surface of energy at some particular temperature. This will vary from place to place on the same planet, as it will from planet to planet. The relationship between this steady-state ratio of heat losses and the steady-state surface temperature, though, is apparently not a straightforward (linear) one. The total heat loss from the surface in the Sahara-Sahel and in the Congo are about the same, but the radiant portion of the total is much larger in the Sahara-Sahel (~100 out of 175 W/m^2) than it is in the Congo (~50 out of 175 W/m^2). This circumstance, however, doesn’t translate at all into a lower surface T_avg in the former region. You can’t just say that a region with a more effective (larger) surface radiant heat loss will necessarily end up having a lower T_avg than one where the radiant heat loss is much smaller. In fact, the surface T_avg is higher in the Sahara-Sahel than in the Congo by a significant amount.

    So how come the radiant heat loss in the Congo is only ~50 W/m^2 and around twice that in the Sahara-Sahel? Atmospheric humidity and clouds. Yes. But also evaporative heat loss and deep moist convection. The two are intimately connected.

    • Norman says:

      Kristian

      I read through your long post. Sounds a lot like what your saying I do agree with.

      You do know that your CERES information confirms that the amount of GHG in the atmosphere has a significant effect on the DWIR. It is around 405 W/m^2 in Congo region and 375 W/m^2 in Sahara. So that should indicate to you an increase in GHG concentration will indeed lead to a higher DWIR which will then reduce the amount of IR leaving the surface (as you pointed out…50 W/m^2 for Congo and 100 W/m^2 for Sahara). One thing we both agree on is that DWIR does reduce surface IR loss.

      I totally agree there are other mechanisms that reduce the surface temperature but they are all considered and accounted for in the global energy budgets. They did not ignore or reject them. Your Congo Vs Sahara is just an example of a region where other effects are dominant above radiation when compared to each other. But GHE is maintaining a higher equilibrium surface temperature in both locations and the difference is only 30 W/m^2 out of a contribution of hundreds of Watt/m^2 at both locations.

      • Norman says:

        Kristian

        Here is a point to make that will allow you to see the significance of GHG. Water is a difficult GHG to model because of all its complex modes of both warming and cooling.

        But let us go to your Sahara location with its 375 W/m^2 DWIR.

        What if you were to introduce tons of Fluoroform into the Sahara atmosphere?

        Here are its properties:
        https://en.wikipedia.org/wiki/Fluoroform

        Several times the GHG of Carbon Dioxide. (11,700 times).

        Now this gas would not have the cooling aspect of water vapor with its clouds and evaporation. Add an equivalent amount of this gas so that Sahara DWIR is now 425 W/m^2. If nothing else changed (like wind patterns) you don’t see how this would nor end up heating the Sahara temperature much above what they are at currently? You have 100 W/m^2 Net IR leaving the Sahara under its current conditions. If you add the gas and increase the DWIR and only get 50 W/m^2 leaving but the same amount of solar input what will happen? You described it. The surface will heat up until it reaches a new equilibrium either via radiation, convection (very little evaporation here) or change in advection.

    • Kristian says:

      Norman says, September 4, 2016 at 5:17 PM:

      You do know that your CERES information confirms that the amount of GHG in the atmosphere has a significant effect on the DWIR. It is around 405 W/m^2 in Congo region and 375 W/m^2 in Sahara. So that should indicate to you an increase in GHG concentration will indeed lead to a higher DWIR which will then reduce the amount of IR leaving the surface (as you pointed out50 W/m^2 for Congo and 100 W/m^2 for Sahara). One thing we both agree on is that DWIR does reduce surface IR loss.

      Er, yes. What did that last paragraph of mine say?

      “So how come the radiant heat loss in the Congo is only ~50 W/m^2 and around twice that in the Sahara-Sahel? Atmospheric humidity and clouds. Yes. But also evaporative heat loss and deep moist convection. The two are intimately connected.”

      I totally agree there are other mechanisms that reduce the surface temperature but they are all considered and accounted for in the global energy budgets. They did not ignore or reject them. Your Congo Vs Sahara is just an example of a region where other effects are dominant above radiation when compared to each other. But GHE is maintaining a higher equilibrium surface temperature in both locations and the difference is only 30 W/m^2 out of a contribution of hundreds of Watt/m^2 at both locations.

      And as I explained in the long comment above, the atmosphere limits the surface heat loss first and foremost by being much warmer than space. The “DWLWIR” is primarily an expression of this simple fact. That you still cannot fathom it really boggles my mind.

      The one thing radiative properties of a massive atmosphere do is connect the atmosphere (in the steady state) thermodynamically to its surroundings. And that’s it. They connect it with the surface underneath, and they connect it with the vacuum of space outside. They allow steady-state heat transfers INTO and OUT OF the atmosphere.

      This is what they do. They’re a “connecting tool”, basically. What causes the surface temperature to actually rise, once the surface is thermodynamically connected to the atmosphere above, is the fact that the warming atmosphere is now able to affect the surface thermally. But it is the warming of the atmosphere that forces the surface to warm also, not the “DWLWIR”.

      You still don’t get this, I know. But there it is …

      • Norman says:

        Kristian

        You say: “And as I explained in the long comment above, the atmosphere limits the surface heat loss first and foremost by being much warmer than space. The DWLWIR is primarily an expression of this simple fact. That you still cannot fathom it really boggles my mind.”

        I feel the same way but in the opposite direction. It boggles my mind you can’t see the direct effect of DWIR on the surface energy budget. A warmer atmosphere without a lot of GHG will not produce a DWIR so the surface radiation will transmit directly to space as if there were no atmosphere. That is why your posts boggle me.

        Here is a thought experiment for you that will demonstrate why my current understanding is the correct one and yours is just plain distorted. It is close to the correct view but just a bit twisted in some weird tangent that just does not make a straight line.

        Take the Earth. Now remove all the lower atmosphere and make it a vacuum so there is not air in contact with the surface. Above have a dome so the air cannot contact the surface, a vacuum (not sure it could ever be constructed in reality since the material would probably not hole). Above the dome is a normal Earth atmosphere, nitrogen, oxygen water vapor, and carbon dioxide. The Earth’s gravity keeps this atmosphere in place like normal it just is not in contact with the surface. The dome allows all radiation in and out (both visible and IR). The solar flux warms the surface like normal, it heats up and starts to emit IR. This IR goes through the dome and is absorbed by GHG’s present (Roy gives the number at 0.8, I have seen higher values). The energy absorbed will start heating and warming this atmosphere. As it warms it will now start radiating in all directions. Some of this atmospheric IR will head to the surface where it is absorbed by the Earth’s surface along with the incoming solar flux. This new DWIR flux will reduce the amount of NET IR the surface will emit so now the energy balance with the incoming flux has changed. Same energy in, less going out. This means the surface will warm until a new equilibrium condition exists. Now the same warming will take place at the surface with zero contact with the atmosphere.

        It is really odd to keep posting to you about this. You can totally understand insulation when it slows conduction but you can’t understand it at all when radiation is involved. The more GHG you add the thicker the insulation, the more DWIR you have and the less energy the surface can lose.

        If you take an oven with a steady heat source and no insulation and it reaches an equilibrium temperature of 250 C but now add a thick layer of insulation to drastically cut back on conduction losses will your oven temperature go down? Stay the same? Or go up?

        Not much different than that no matter how you want to think it is.

        • gbaikie says:

          “I feel the same way but in the opposite direction. It boggles my mind you cant see the direct effect of DWIR on the surface energy budget. A warmer atmosphere without a lot of GHG will not produce a DWIR so the surface radiation will transmit directly to space as if there were no atmosphere. That is why your posts boggle me. ”

          Well because water vapor forms into water droplets and becomes
          clouds, are clouds GHG?

          It the process of UV light converting O2 into O3 [and chemical reaction which causes oxygen atoms and molecule to emit radiant energy] which occurs in ozone layer. Called GHG. Or to be clear, we can’t have oxygen and our sun and not have ozone.
          Or can’t have just N2 and O2 atmosphere and not have ozone.
          So a N2 only atmosphere is what you mean?

          So I think 1 atm pressure of just N2, would reflect almost as much
          of the Sunlight as our world with it’s mixture of gases which are mostly N2.
          So roughly it’s 240 to surface and 240 watts radiated back into space with 100 reflected.
          Suppose one wants to simplify and have some white sphere in space
          which absorbed 240 and reflected 100 watts.
          So if mixed paint and painted a surface and tested it, and found
          some color of paint [paint can be made of anything which one can grind up and which can be glued together, somehow- that is what paint is- definitionally]. So got something painted and it reflects 100 watt of 340 watts of sunlight [and absorbs and emits
          240 watts.
          A couple ways of getting 340 watts of sunlight. One way is to be at distance from the sun in which the sunlight was 340 watts. That would be a bit further than Mars orbital distance.
          Another simpler way [not as accurate] is to be on the earth surface and have sunlight hit surface at an angle or be at location where sun is at around 20 degree above the horizon. So
          if sunrise is at 6 am, around or before 7:30 am with clear skies.
          Another way is 4 times 340 is 1360 watts, so go to orbit
          and it should reflect 400 watts and absorb 1040 watts.

          But this is all wrong, because the N2 atmosphere planet is sphere, but let’s try it anyhow. Or making mistakes is only way anyone learns anything.
          So get sphere and paint it with something that reflects 100 and absorbs and emits 240 from 340 watts per square meter.
          Make 100 meter in radius sphere out of copper and put in solar orbit at Earth distance from the Sun, and have solid cooper sphere painted [wrongly as above].

          So in sunlight this is going to be blindly bright- even if color is painted with flat black paint, or so, is the Moon. Depending on the paint it could blind a human without some kind “sun glasses” or it matters how the paint deals with UV, X-rays, etc, plus just the intensity of sunlight. And matters what angle the sun is related to the sphere and how far you are from it.
          And looking at the sun itself is like watching welding at night- worst than looking at the sun on earth surface.
          So painted sphere will get the most amount sunlight per square meter of the surface, within 45 degree latitude and longitude
          away from the point of zenith of sun upon the sphere.
          Or sunlit side of sphere [it’s hemisphere] is 90 degree from the sun zenith on the sphere [of 360 degrees of entire sphere].

          So the smaller circle in middle of sunlit hemisphere, will reflect 400 watts per square meter [cause I did it wrong] and rest of sphere will reflect less the 400 watts per square meter- it receives less than 1350 watts per square meter.

          So how much area is in this smaller circle.
          100 meter radius is 100 times 2 times Pi in terms of circumference of sphere: 628.318 meters.
          Each degree: 628.318 / 360 is 1.745 meters.
          times 45 is 78.539 meter
          Small circle is 78.539 in radius
          Sunlit hemisphere is 100 meters in radius- but since dealing
          with a curved surface it might be misleading.
          Or if flat instead of curved the difference in area is:
          Smaller circle: 19378.87 square meter
          Larger circle including small circle: 31415.9 square meter
          [Pi x 10^4, btw], And 31415.9 – 19378.87 is
          12,037.03. Or if flat it’s small area than small circle- but
          instead with curved sphere is quite the opposite.
          Area of sphere, 100 meter radius, google: 1.2610^5
          or hemisphere: .63 x 10^5 or 63,000.
          And quite roughly 63,000 – 19378.87 is 43621 square meter.
          I can roughly use flat small circle because it’s not curved
          much.
          Small circle: about 19379 square meters
          Larger circle, donut: about 43621 square
          Or donut is more than twice area as donut hole
          So energy of the sun is disk area: Pi x 10^4: 31415.9
          The donut hole get 19379 of the 31415.9
          and donut gets 12,037.03 of 31415.9 but spread
          over more than twice the area as compare to hole.

          Now with atmosphere of N2 reflecting sunlight,
          the donut hole area does not have much sunlight
          reflecting from it, and the donut has bit more
          but it’s twice the area.
          Or from a 2 dimensional kind of way the donut reflects
          a lot more of the sunlight than donut hole, but not
          huge amount more per square meter, as it receive less
          sunlight per square square. As percentage of the
          sunlight it receive it’s a much larger percentage.
          Now the painted sphere does something similar, a main
          difference is the paint “designed” to reflect the right
          amount of sunlight, would reflect a lot more sunlight
          in the donut hole, than it should. And atmosphere would
          reflect more of percentage of sunlight as compare to
          painted sphere.
          Or to try to match it with paint requires using different
          paint in different parts of sphere- or, some kind of magical
          paint.
          Anyways, all that and I didn’t really get to a point of
          getting to my point.

          • Norman says:

            gbaikie

            too bad you were unable to make your point.

          • gbaikie says:

            “too bad you were unable to make your point.”
            Getting to it.
            So 100 meter radius solid copper which is painted sphere- correctly painted- should, in vacuum have equator average temperature
            of 30 C. With colder temperatures pole ward

            And it approximates a planet with N2 atmosphere.
            A small solid sphere in vacuum can conduct some heat poleward,
            and likewise a N2 atmosphere could also transport some heat
            poleward. [[Small solid sphere has little variation in night and day temperature at equator- assume one day rotation- unlike a planet with N2 atmosphere.]]
            But not as much poleward heat transport as water planet can do. Of course the configuration of land masses in relation to ocean would vary how well a water planet can do this.
            So roughly colder polar regions or above 50 degree latitudes
            and desert like conditions in tropics- hot days and cold nights.
            Should be no shortage of wind, and probably large chunks of air masses moving- and/or stronger winds.
            Though not allowing for unknowable weather patterns- one could get something like global wind near equatorial regions, with could make it warmer and more uniform day and night temperatures.
            Or something vaguely like El Nino without involving an ocean. Plus polar vortexes similar, stronger or weaker as compared to Earth.

  44. Kristian says:

    Global OLR anomaly at the ToA vs. global TLT anomaly, Mar 2000 – May 2016:
    https://okulaer.files.wordpress.com/2016/09/tlt-vs-olr.png

    Still absolutely no “enhanced GHE” in sight, only natural radiative effects to tropospheric temps plus ENSO-induced cloud anomalies.

    Global “DWLWIR” anomaly at the surface vs. global TLT anomaly, Mar 2000 – Feb/May 2016:
    https://okulaer.files.wordpress.com/2016/09/tlt-vs-dlr.png

    No “enhanced GHE” to be seen here either …

    • Ball4 says:

      Kristian, still no work behind the graphs to be seen either, show all your work behind those graphs like Dr. Spencer and the CERES team publishes.

      • Kristian says:

        My work? I downloaded the data and plotted it, what more do you need? You know where to find it. You’re just a pest, Ball4.

        • Ball4 says:

          Downloading CERES data is only a beginning Kristian, as the CERES team informs you – the instrument has to be calibrated to compare to temperature like any mercury thermometer.

          • Kristian says:

            Ball4,

            Can you stop it with this nonsense!? Please provide the full quote (with link to specified source) stating that the CERES EBAF Ed2.8 TOA OLR data is not not usable in its presented form, because it is not ‘calibrated’ and ‘validated’, and hence does not in fact have the precision level claimed by the CERES team.

          • Kristian says:

            Also, I’ve already shown you this several times, Ed2.8 (black) vs. Ed4 (blue & red) OLR:
            https://okulaer.files.wordpress.com/2016/08/olr-30-30-paper.png

            Why do you keep ignoring it?

          • mpainter says:

            Kristian
            B4 engages in these tactics deliberately, a sort of malice. Best not let him bait you.

          • Ball4 says:

            Kristian could answer the question by replicating Loeb 2016 but won’t do so even though it has a different conclusion than Kristian. Kristian provides no confidence intervals. Kristian doesn’t even tell us which data set Level he downloaded.

            Calibration issues are not discussed by Kristian who just downloaded the instrument data. For example,

            “for monthly mean clear-sky TOA fluxes, (CERES team) do not explicitly account for changes in the physical properties of the scene (e.g., aerosols, surface properties) during the course of the day.”

            Kristian did not account for this. Does UAH?

            CERES data “Users should be aware that in these cases, albedos (derived from the ratio of outgoing SW to incoming solar radiation) exceed unity.”

            Did Kristian account for that? No. Did UAH?

            “EBAF uses geodetic weighting to compute global means.”

            This assumes a spheroid when Earth is oblate spheroid. Kristian made no correction, did UAH?

            “Sohn et al. (2006) note that differences in how clear-sky is defined in model output and observations can lead to regional LW TOA flux differences of up to 12 W m-2.”

            What did UAH do? Anything?

            “To account for in-orbit changes in SW spectral response function (SRF), direct nadir radiance comparisons between instrument pairs on the same satellite are made and an improved wavelength dependent degradation model is used to adjust the SRF of the instrument operating in a rotating azimuth plane scan mode.”

            What did Kristian do about that? Anything needed?

            “This paper focuses on how the CERES team corrects for on-orbit changes in instrument calibration.”

            Kristian always follow what they did? Dunno, he just downloaded a dataset. there are many more issues Krisitan needs to address, this only a sample.

            Kristian just downloaded the data, were all of them properly included to compare to what UAH did for their Tmedian? Dunno.

          • Kristian says:

            Ball4 says, September 4, 2016 at 2:48 PM:

            Kristian could answer the question by replicating Loeb 2016 but wont do so even though it has a different conclusion than Kristian.

            It doesn’t have a different conclusion than me. The OLR at the ToA data are basically unchanged from Ed2.8 to Ed4. As I’ve shown you. Again and again. Why do you continue to ignore this?

          • Kristian says:

            Still waiting for that quote and source, BTW.

          • Ball4 says:

            Kristian, calibration unknown, “It doesnt have a different conclusion than me…Still absolutely no enhanced GHE in sight”

            CERES team, Loeb 2016 with published calibration techniques: CERES/Terra OLR -0.89 +/-0.4 W/m^2/decade

            Feldman 2015: “Observational determination of surface radiative forcing by CO2 from 2000 to 2010”

          • Kristian says:

            Ball4 says, September 4, 2016 at 3:07 PM:

            CERES team, Loeb 2016 with published calibration techniques: CERES/Terra OLR -0.89 +/-0.4 W/m^2/decade

            And as you know full well (because I’ve shown you), Ed2.8 would give pretty much the same trend. For the particular time segment in question. However, my “conclusion” doesn’t rest on the 2003-2013 interval.

            The quote and its source, please.

          • Ball4 says:

            Very easy for even Kristian to find CERES principle investigator by google string: Norman Loeb website

            In case Kristian still has trouble

            http://ceres.larc.nasa.gov/science-team-members.php?person=Loeb

            Enjoy the 2nd item at this writing. Show us your work to find replication is “pretty much the same trend” as CERES/Terra OLR -0.89 +/-0.4 W/m^2/decade.

          • Kristian says:

            Ball4 says, September 4, 2016 at 5:27 PM:

            Enjoy the 2nd item at this writing. Show us your work to find replication is pretty much the same trend as CERES/Terra OLR -0.89 +/-0.4 W/m^2/decade.

            I already did:
            https://okulaer.files.wordpress.com/2016/08/olr-30-30-paper.png
            Why do you keep ignoring it?

            Now show us the quote (with linked source), Ball4, where it says that the CERES EBAF Ed2.8 ToA OLR data is not usable in its presented form, because it is not ‘corrected’, ‘calibrated’ and/or ‘validated’, and hence does not have the precision claimed by the CERES team.

            Stop acting like a child.

          • Ball4 says:

            There is no CERES/Terra OLR -0.89 +/-0.4 W/m^2/decade shown in that link Kristian unless you used double secret invisible font. No confidence interval shown at all. You need to accomplish some grown up work to replicate the published paper in full and provide confidence in your curves. The reader is not going to do that work, it is responsibility of the commenter.

            “Now show us the quote (with linked source…”

            Still having trouble using google? I did not use double secret invisible font Kristian, the latest link has been provided to you “Accounting for in-Orbit Changes in Instrument Calibration” which I looked for cold and found in a few minutes.

            “it is necessary to periodically reprocess the data in order to incorporate the latest calibration changes and algorithm improvements”

            Up to the commenter (Kristian) to show his work and the data Level downloaded with its formal description. Also, research on UAH compared to these calibrations is necessary. Are they apples and apples like a thermometer reading 32F in ice water? Or not, if so, where,why? The reader can’t tell from Kristian, all we get is some curves. I had to look up the published papers to find out.

          • Kristian says:

            Ball4 says, September 5, 2016 at 6:37 AM:

            There is no CERES/Terra OLR -0.89 +/-0.4 W/m^2/decade shown in that link Kristian unless you used double secret invisible font.

            It’s not just a ‘link’, Ball4. It’s a time series diagram of plotted 30N-30S OLR data from Jan’03-Dec’13, with the black CERES EBAF Ed2.8 curve superimposed on the red and blue CERES EBAF Ed4 curves. It’s for all to see – the three curves basically follow the same course.

            No confidence interval shown at all. You need to accomplish some grown up work to replicate the published paper in full and provide confidence in your curves. The reader is not going to do that work, it is responsibility of the commenter.

            I use CERES data, Ball4. So the confidence interval would be the confidence interval of the CERES data. Ask the CERES team. Or just look it up.

            I did not use double secret invisible font Kristian, the latest link has been provided to you “Accounting for in-Orbit Changes in Instrument Calibration” which I looked for cold and found in a few minutes.

            Yes, thanks, I’ve read that. And where exactly does it say that the Ed2.8 version of the CERES EBAF TOA OLR data is useless in its presented form because it is not ‘corrected’, ‘calibrated’ and ‘validated’, and thus does not come with the precision level claimed by the CERES team?

            Quote, please. I didn’t ask for a link to a paper. I asked for a direct QUOTE with a link to the source of that quote. You do understand this, don’t you, Ball4?

            You’re only evading, because you know you don’t have such a quote …

            “it is necessary to periodically reprocess the data in order to incorporate the latest calibration changes and algorithm improvements”

            Yes, they are presenting an upgrade of their data, Ball4. That’s pretty common. But it doesn’t mean that 1) the ‘old’ data was uncorrected, uncalibrated and/or unvalidated, or that 2) the ‘new’ data is necessarily completely different from the ‘old’ data. In fact, when it comes to the all-sky OLR at the ToA parameter (which is what we’re looking at here), the data is basically unchanged from Ed2.8 to Ed4. As the figure above clearly shows …

            You have no case here, Ball4.

            Up to the commenter (Kristian) to show his work and the data Level downloaded with its formal description.

            No, it’s up to you (Ball4) to provide the direct quote (with link to source) that supports your claim that there is an issue here at all … That’s it’s not just you being an imbecile.

          • Ball4 says:

            “Its a time series diagram of plotted 30N-30S OLR data..”

            Correct, what it is NOT is showing us Kristian’s download replicates this calculation: CERES/Terra OLR -0.89 +/-0.4 W/m^2/decade & until Kristian performs that work there is no confidence that Kristian’s other work is any good, at all. Apparently Kristian just ignores this important result so no one is impressed with Kristian’s simple graphs of readily available data.

            The meaning behind the curves & CI is essential to discuss Kristian. Until you get that there is no confidence in your other work especially when it draws different conclusions than CERES Team, you need a mountain of explaining work for that. Not a simple curve download.

            “And where exactly does it say that the Ed2.8 version of the CERES EBAF TOA OLR data is useless..?”

            Nowhere that I have found, that is Kristian’s misdirection. What is useless is Kristian’s simple download plots without showing us how he can get this out of it: CERES/Terra OLR -0.89 +/-0.4 W/m^2/decade.

            I’ve lost track of which quote Kristian can’t seem to find even though I found it cold in a few minutes. Anyone expecting to confidently extend CERES team work should have ALL their publications nailed.

          • Ball4 says:

            “So the confidence interval would be the confidence interval of the CERES data.”

            Says nothing about Kristian’s downloads CI(s) outside CERES team interval; Kristian has to show he can replicate the CI of the CERES team using their time interval then apply the same method to series outside their time interval.

            This is so readily apparent one has to conclude Kristian is not up to the task so there is no confidence, at all, in Kristian’s other interval work being meaningful.

          • Kristian says:

            Ball4 says, September 6, 2016 at 6:55 AM:

            “Its a time series diagram of plotted 30N-30S OLR data..”

            Correct, what it is NOT is showing us Kristians download replicates this calculation: CERES/Terra OLR -0.89 +/-0.4 W/m^2/decade & until Kristian performs that work there is no confidence that Kristians other work is any good, at all. Apparently Kristian just ignores this important result so no one is impressed with Kristians simple graphs of readily available data.

            Who are you talking to here, Ball4? Me or some imaginary third person. More and more I get the impression that you suffer from certain mental issues …

            (…) especially when it draws different conclusions than CERES Team (…)

            Again, my conclusion ISN’T different from that of the CERES team. The only one here ‘concluding’ that since the OLR went down between Jan’03 and Dec’13, this is PROOF of an “enhanced GHE” in operation, is you, Ball4.

            “And where exactly does it say that the Ed2.8 version of the CERES EBAF TOA OLR data is useless..?”

            (…)

            Ive lost track of which quote Kristian can’t seem to find (…)

            I’m not the one who cannot find it. YOU are, Ball4. I know it doesn’t exist. You continue to argue AS IF it does.

            (…) even though I found it cold in a few minutes.

            No, you didn’t. You found a paper detailing a CERES data upgrade, that I have shown you does not significantly differ from the ‘older’ version when it comes to all-sky ToA OLR.

            You’re deliberately misdirecting to evade, Ball4. I know it and you know it.

            Kristian has to show he can replicate the CI of the CERES team using their time interval then apply the same method to series outside their time interval.

            Haha! So I reckon this goes for anyone who dares posting simple plots of official data on blogs like this …

            I’ve posted official data, Ball4. If you think the data is somehow wrong, the onus is on you to back that idea up. You haven’t. What you have shown instead is the exact opposite, that the data is more or less spot on. According to the most recent version.

            You have nothing, Ball4.

          • Ball4 says:

            Kristian continues twisting things into a pretzel when all he has to do is replicate the calculation of the CERES team for CERES/Terra OLR -0.89 +/-0.4 W/m^2/decade from the data he downloaded.

            “Who are you talking to here, Ball4?”

            The commenter whose written words I clipped for anyone that can plainly see, well except for Kristian.

            “Again, my conclusion ISNT different from that of the CERES team.”

            Incorrect Kristian, since you write this: “the radiative imbalance at the ToA is not a result of an enhanced GHE, but by an increase in solar input (ASR).”

            Then you are forced to agree the OLR went down per CERES team when called out.

            “The only one here concluding that since the OLR went down between Jan03 and Dec13, this is PROOF of an enhanced GHE in operation, is you, Ball4.”

            Nope, never wrote that, quote my words Kristian’s won’t do. Feldman 2015 did provide proof of eGHE though.

            “I know it doesnt exist.”

            Again, which quote Kristian? Your bag of pretzels is opaque on that.

            “No, you didnt.”

            Yes. I did. I’ll find the quote in a few minutes again if Kristian will tell me which quote.

            ” Youre deliberately misdirecting to evade, Ball4.”

            No misdirection Kristian, it is you directing attention away from your inability to calculate CERES/Terra OLR -0.89 +/-0.4 W/m^2/decade from the data you dowloaded.

            “So I reckon this goes for anyone who dares posting simple plots of official data on blogs like this “

            Nope, only those that do so and then can’t calculate CERES/Terra OLR -0.89 +/-0.4 W/m^2/decade from the data.

            Kristian downloaded some official data which anyone can do, then Kristian fails repeatedly to show us the data calculates to CERES/Terra OLR -0.89 +/-0.4 W/m^2/decade. Kristian is unable to show us the data is spot on. I have the CERES team results Kristian, all Kristian has is some downloaded data he doesn’t quite know what to do with.

            It is easy Kristian, untwist the pretzel you find yourself in when called out:

            1) tell me which quote you want a link to,
            2) calculate CERES/Terra OLR -0.89 +/-0.4 W/m^2/decade from the data you downloaded.

            The results might even be interesting to discuss & not twist pretzels.

  45. mpainter says:

    And if the oil and vinegar doesn’t help, you might try some ketchup.

  46. mpainter says:

    So, Norman, let’s see if I understand you correctly. Let’s take the hypothetical case of tropical ocean with a clear sky insolation of 1,000 W/sq m and a back radiation of 400 W/sq m, both of these radiative fluxes “absorbed by the surface” for total of 1,400 W/sq m “absorbed by the surface”.
    So, if I understand correctly, the surface would not warm according to the total of 1,400 W, but only according to the 1,000 W of insolation. Is this correct?
    And the reason that the 400 W/sq m caused no warming is because the surface radiated that before it could warm anything. Right?

    • Norman says:

      mpainter

      I would say that is close but the point I would change was the “radiated that before it could warm anything”

      There is no before or after it is on a continuous basis, the surface is not radiating away the energy before the DWIR arrives, it is emitting the energy constantly as it is also absorbing energy constantly. The exchange is simultaneously going on with different parts of the surface, some parts are warming some cooling. The effect is a total of the individual surface effects.

    • mpainter says:

      But the surface warms from insolation, not from back radiation, right? From 1,000 W/sq m, not 1,400 W/sq m, right?

      • David Appell says:

        THe surface warms from all radiation it receives. Including back radiation (which is just radiation from the atmosphere).

        • mpainter says:

          But Norman says it does not warm from back radiation. Now I don’t know what to believe.

          • Norman says:

            mpainter

            David Appell is only partially right. The surface warming is not just a basis of how much radiation it receives. Roy Spencer has stated it numerous times. It is not just how much radiation that is received but also how much energy is emitted away.

            What do you say? Is an object receiving 2000 W/m^2 warming?

            Is it warming faster than one receiving 10 W/m^2?

            You can’t really tell. The object receiving 10 W/m^2 may be warming if it is only losing 5 W/m^2 at the time.

            The object receiving 2000 W/m^2 could be cooling if it radiating energy away at 3000 W/m^2.

            But it is certain that if an object is emitting away radiation at 3000 W/m^2 but receiving energy at the rate of 2000 W/m^2 it will cool slower than an object radiating away 3000 W/m^2 but not receiving any radiant energy.

          • mpainter says:

            If David is “partially right” then you are partially wrong. Because you said back radiation does not warm the surface. David says it does. Someone is confused. Is it you or David. Or do you mean that David is all wrong, and you you were just being nice about it?

        • Toneb says:

          Let me clarify this David:

          Solar TSI and the GHE warm the surface.
          However only with solar TSI is there a net flux of heat to the surface of Earth
          In the case of the GHE, the flux is from surface to atmosphere.
          As certain people know (as they have been told multiple times), the GHE is not a flux of heat back to Earth and therefore is perfectly in accord with the 2nd LoT.
          It surely has to be game with *them*.

          • gbaikie says:

            ” Toneb says:
            September 5, 2016 at 1:09 AM

            Let me clarify this David:

            Solar TSI and the GHE warm the surface.
            However only with solar TSI is there a net flux of heat to the surface of Earth
            In the case of the GHE, the flux is from surface to atmosphere.”

            So atmosphere is warmed by radiant Longwave IR from the surface?
            That seems like new version to me.
            So is this like Norman’s view that gases re-radiant and don’t really get warmer. Or said differently is it heating the CO2 and Co2 cools at same time so that don’t get warmer?

            Or is this going to another argument about lapse rate?

            Why don’t we go to Venus.
            Is land surface of Venus heated by sunlight and so does something like it does on Earth?
            Or there something different going on with Venus?
            And flipping over to Mars where there is 28 times more CO2 above the surface as compared to Earth- and Mars’ thin atmosphere also has 210 ppm of water vapor.
            Is there GHE on Mars? And if so, about how much?

          • mpainter says:

            But GHE = back radiation and Norman says that back radiation does not warm the surface. Whom to believe? Toneb says it does, David says it does, Norman says it doesn’t, but he doesn’t sound so sure, now.

          • gbaikie says:

            –mpainter says:
            September 5, 2016 at 9:27 AM

            But GHE = back radiation and Norman says that back radiation does not warm the surface. Whom to believe? Toneb says it does, David says it does, Norman says it doesnt, but he doesnt sound so sure, now.–

            Yes, but to be fair, the deniers don’t agree either.
            Of course, we prefer to be called skeptics- though
            skepticism is basis of science, so is that really fair?

            In context of the unsettled science [sometimes I like to be
            generous and overly optimistic to include the GHE theory as
            science rather some nutty pseudo science badly compiled by
            a nameless and feckless committee] I am agreeable with being labeled as a lukewarmer. Example of what believers say:
            “Dana Says:
            June 8, 2011 at 20:40

            Its difficult to define exactly what lukewarmer means, because all deniers want to be considered lukewarmers, because the term makes them sound reasonable. So you get people across the spectrum trying to invoke the label, and it becomes impossible to define.”
            https://ourchangingclimate.wordpress.com/2011/06/08/definition-lukewarmer-co2-not-irrelevant-despite-low-concentration/

            To make it clear, I am lukewarmer who knows that over last hundred years or so, there has been what could be called global warming which has resulted in increase in global temperature by about 1 C and fairly certain that in next hundred years or so, global temperature will not increase by more than 1 C.
            And roughly I think a 2 C increase in global temperature from the coolest depths of the Little Ice Age is a very good thing.
            Or is more more fortunate than anything done by any and all governments in the last hundred years.
            Or something perhaps more undeniable, it can’t be worst than what governments have been doing over last hundred years.
            And find interesting that witch burning was related to the idea
            witches were doing many bad things- which include effecting the weather.
            Governments burned witches, and gassed Jews and other “undesirables” [including homosexuals- and other people are simply disagreeable, politically]. And politician always blame something other than the government which constantly creating the mayhem and always dividing it’s citizens.
            And as general issue giving more power to the government- never ended well, and can not end well. And the desire for totalitarian government, trumps reason.
            Or soviet and Chinese government killed tens of millions of it’s citizen- because politicians are politicians whenever they have
            too much power.

  47. Norman says:

    mpainter

    Maybe I can use your tropical ocean example to show the way GHE will lead to a higher equilibrium temperature.

    In your example we now have a non rotating Earth so the 1000 W/m^2 insolation is continous. We want to eliminate the strong effect of evaporation to show just a GHE so we cover the water in a transparent plastic that allows solar and IR to move freely through it but prevents evaporation.

    Without the 400 W/m^2 DWIR the water would reach an approximate temperature of 91 C at which point it will be radiating IR at the rate of 1000 W/m^2. It will be an equilibrium condition. Now add the GHG atmosphere with its 400 W/m^2 DWIR.

    Your downwelling flux has some options when it hits the surface.
    1) transmitted…you have already ruled this out as IR does not penetrate water
    2) reflected…water has a high emissivity so should absorb almost all incoming IR.
    3) absorbed…this one seems the most likely based upon the properties of water and its ability to absorb IR.

    If you think in Kristian’s methodology you could say that with the 400 W/m^2 DWIR combined with the 1000 W/m^2 UPIR the surface is losing only 600 W/m^2. You can do it either way. The surface still emits 1000 W/m^2 and the atmosphere emits 400 and they subtract with a surface losing 600 W/m^2. You still have the solar insolation of 1000 W/m^2 but only losing 600 W/m^2.

    What happens now? If the surface has an input of 1000 and an output of 600 it will start to warm until the output is at 1000.

    So with the DWIR of 400 the surface (in my way) would have to rise in temperature until it is emitting 1400 W/m^2 that is the only way it will lose 1000 W/m^2.

    So the new equilibrium temperature reached would be 123.25 C.

    • mpainter says:

      Yes, but Norman, the surface warms from insolation, not from back radiation,right? That is what you said up thread. Or do you think differently now?

  48. Norman says:

    mpainter

    I did not change my view. The backradiation does not warm the surface. The surface is generally radiating away energy at a higher rate than the backradiation so it would be a cooling effect on its own.

    What happens is the equilibrium temperature that can be reached by a constant flux changes if you add backradiation and will go up.

    The energy comes from the constant influx of energy.

    On Postma’s blog they talk about a tea cup.
    https://climateofsophistry.com/2016/08/31/cup-calling-the-kettle-for-back-tea/

    Postma does not understand GHE as is obvious from his example yet he goes off on a rant about people who do understand the process.

    His words: “Both concepts are obvious, gross, flagrant, childish, and idiotic claims violating the most basic thermodynamics, and childhood empirical exploration.”

    For some reason he can’t seem to understand that the Earth system is constantly receiving energy. It is an equilibrium temperature that is rising with a constant influx of energy.

    They somehow think what Roy is saying is if you pour a cup of tea and wrap insulation around it, then the GHE means the water in the cup will warm up and not cool at a slower rate.

    That is their flawed thought process. What goes on with the GHE would be to put insulation around the kettle as heat is being constantly added. The water in the kettle will get hotter with insulation than without, not sure what the equilibrium temperature will be but it would depend upon the quality of the insulation.

    So many get it wrong. I rarely see the Postma followers ever give examples of a continuous heat source and adding some form of insulation and noting the results.

    Roy does all these experiments, they do none and call him dishonest. Postma is a real crap scientist but he is able to convince so many gullible people then they come on this blog with the same arguments but never understand what is being said.

    • mpainter says:

      Yeah, well forget about Postman, I never heard of him and couldn’t give a hoot.

      Back to pinning you down, Norman. Do you still insist that back radiation “absorbed by the surface” doesn’t warm the surface?

      • Norman says:

        mpainter

        I need to make sure my semantics are correct on these fine points as I think that is the reason for much misunderstanding on the basic issue.

        I understand how the process of GHE works but I might not be able to communicate it properly and may make mistakes in my presentation. This does not mean the GHE process is incorrect, it means my ability to communicate it clearly is where the flaw exists.

        Here is a attempt. Backradiation alone (ALONE is the key word) will not warm the Earth’s surface because the radiant energy leaving the surface is greater than the radiant energy of the backradiation absorbed by the surface.

        This is why David Appell is correct except he leaves out the energy leaving the surface and only mentions the energy entering the surface.

        Most skeptics of GHE note that it is absurd and against the laws of physics for backradiation to, by itself, warm the Earth’s surface and by itself it will not. That is the point I am bringing up.

        The next point is that the solar flux will only warm the Earth’s surface to an equilibrium temperature in that the temperature will rise until it is at a point where the outgoing energy is the same as the solar incoming flux. Solar flux alone will warm the surface to an equilibrium temperature which is completely dependent upon the actual solar flux.

        When both backradiation and solar flux are included the equilibrium temperature of a given solar flux will go up since the backradiation is suppressing the amount of radiant energy that can leave the surface (the heat, the net surface energy flow).

        Without backradiation the surface HEAT or NET ENERGY (either choice means the same thing) will be of a higher value than with a backradiation and this amount is dependent upon the amount of backradiation. The higher the value of the backradiation the less HEAT can leave the surface and the incoming solar flux will accumulate a higher energy into the surface.

        Still not sure if the communication is working. If that does not make sense to you I can continue trying until either you convince me I am wrong with good logical arguments and some evidence or you can understand correctly what I am trying to state. Thanks for the consideration and have a nice day.

        • mpainter says:

          Funny thing, Norman, my comments get shorter but yours get longer. And longer.

          The definition of the GHE in a nutshell: it warms the surface.

          Soooo, does the GHE warm the surface?

          • Ball4 says:

            “does the GHE warm the surface?”

            A. Earth atm. GHE allows the sun to warm the surface Ts – Te = ~ 288 – 255 = 33K, all measured data.

          • Norman says:

            mapainter

            Ok to keep it shorter. Does insulation around an oven warm the oven?

            That is the point of contention.

            Insulation around an oven with a heating element on will get warmer than the same oven with same heating element with no insulation around it. Does the insulation warm the oven? The oven is warmer with insulation but is the insulation actually warming it?

            Without a heating element the non-insulated and insulated oven will reach the same equilibrium temperature.

          • Norman says:

            mpainter

            If in your thought process, you believe insulation warms an oven or anything then I would conclude that likewise GHE is warming the surface.

            If you think like I do, insulation does not warm things, but things will get warmer with it, then you would conclude GHE does not warm the surface.

            I think it is a semantic problem, how people define concepts in their head.

            David maybe thinks insulation warms things as does Toneb. They would not be wrong, it is just how they view it so in their view GHE is warming the Earth’s surface. It is not the way I view it.

          • mpainter says:

            Norman, then your view is that back radiation absorbed by the surface acts as insulation and retards cooling of the surface. And that back radiation is absorbed by the surface together with insolation but only insolation warms. So in the example that I gave, the sea surface absorbs 1,400 W/sq m of radiant energy, but only 1,000 W actually warms; the other 400 is radiated from the surface with no effect on surface temperature.

            Does this fairly represent your views?

          • Norman says:

            mpainter

            Your posts are what causes me to post long involved ones that are too long for most people to read or care about. Does not seem short ones work though.

            The position of Toneb and David Appell, that GHE warms the surface will cause lots of confusion and the skeptics to claim that violates the 2nd Law of Thermodynamics.

            I understand what they are saying but unless you really have a good grasp of it then you will conclude incorrectly that a cold surface is warming a warmer one.

            I think no matter how I try to explain it, some will get confused by my explanation.

            Only one input can cause actual warming of the Earth’s surface and that is the solar insolation.

            Solar insolation will warm a cold surface on its own.

            GHE will not warm a surface on its own. Without solar insolation the surface will just keep cooling (slower than without GHE) until it is as cold as a surface without any GHE.

            The combination of solar insolation and GHE will lead to a higher surface temperature than with just insolation. With just GHE you get only cooling.

            The combination of solar insolation and GHE will warm the surface to a higher temperature than one without GHE.

            Just as insulation will raise the temperature of a heating element but it in and of itself is not warming the heating element.

            It is difficult to communicate this concept. I doubt this will help, and I am not sure anything I post will. Thanks for you time and considerations.

          • Joel Shore says:

            This argument is really just about semantic issues and it is rather painful to see it just go on and on.

            I think it is best if we use an example that is easier to understand. (It’s also an example that I think you libertarian-types…who question climate change because you don’t like the policy implications of the science…can appreciate.) Let’s say that you earn $100,000 per year. And, let’s say that the government imposes a wealth (not income) tax of 25% (and, for simplicity, assume you don’t spend any money). Then, it is not hard to show that the “steady-state solution” after several years would be that you have a wealth of $400,000 (because then the amount you pay to the government exactly balances the amount that you earn each year).

            Now let’s suppose that the government decides to give a 20% rebate on your taxes, i.e., return 20% of what you pay. Then, in effect the new tax rate is 20% and your new steady-state wealth would be $500,000. So, you are $100,000 richer.

            Now comes the question: Did the government make you richer? This is really a semantic debate. A lot of people would think it silly to say that the government made you richer just by letting you keep more of your hard-earned money. They would say that it is your income that is responsible for your wealth and the net flow of money is from you to the government. On the other hand, there is no doubt that you are richer because the government is now taking less of that money than before.

            This is as perfect an analogy as one can find for the situation we are talking about: Income is the energy received from the sun. The wealth is the Earth’s surface temperature. The wealth tax is the energy re-emitted and the rebate is the back-radiation. Money is a conserved quantity (at least in this example) just like energy and the wealth tax means that the amount of money you lose to the government is an increasing function of your wealth just like the amount of energy that the Earth radiates is an increasing function of the temperature.

          • mpainter says:

            Joel, you misread the exchange. There was no argument. Norman was merely articulating his view of the actuality of the GHE, back radiation, etc., under my encouragement and attempts to clarify.
            Read it again and don’t be pained.

          • Joel Shore says:

            mpainter now:
            “Joel, you misread the exchange. There was no argument. Norman was merely articulating his view of the actuality of the GHE, back radiation, etc., under my encouragement and attempts to clarify.”

            mpainter earlier:
            “If David is ‘partially right’ then you are partially wrong. Because you said back radiation does not warm the surface. David says it does. Someone is confused. Is it you or David. Or do you mean that David is all wrong, and you you were just being nice about it?”

            So, in fact, you are clearly confused (or else engaging in sophistry) regarding the use of words that are simply not well-defined. As I said, this is purely a semantic question. If you tell me your definition of what “warm” means as a transitive verb describing the actions of one body on another (in a way that it applies when the second body is exchanging energy not just with this body but also with a third body and I can tell you whether or not back-radiation warms the surface). Without such a clear definition, one simply can’t answer the question because the terms in the question have not been properly defined.

            This kind of semantic ambiguity is exactly what the forces of sophistry, like Joe Postma, are exploiting to confuse people like yourself about basic physics.

          • gbaikie says:

            “If you tell me your definition of what warm means as a transitive verb describing the actions of one body on another (in a way that it applies when the second body is exchanging energy not just with this body but also with a third body and I can tell you whether or not back-radiation warms the surface). Without such a clear definition, one simply cant answer the question because the terms in the question have not been properly defined.”

            In terms of gases, warm means increase average velocity of the gas molecules. In terms of liquids and solids, warms means to increase the vibrational energy of molecular structure of the solid or liquid.

          • mpainter says:

            Joel, you rant about the word “warm”. For our purposes here it is understood as an increase in kinetic energy. And I think that you are the only person who had difficulty grasping that. You will find that “warm” is used in that same sense by many of your heroes of the global w.a.r.m.
            …ing advocates.

          • Toneb says:

            Norman:

            “David maybe thinks insulation warms things as does Toneb. ”

            Yes and no.
            It is a matter of semantics.

            The warming is done via insulation.
            The energy (terrestrial LWIR) is re-received by the surface, having lost it the first place.
            And it is less than it lost.
            It is not more energy (just the back-radiated component) than it lost – therefore it is not warming it.

            10 – 5 =5
            5 + 4 =9
            9 is less than 10 (Not warming)

            Where it CAN warm the SKIN of the surface is due to thermal flux from depth, whereby a new equilibrium is achieved re energy out vs energy in (total below + above).
            This only happens at night when there is no absorbed TSI.

            Any clearer?

  49. peterg says:

    To directly measure the greenhouse effect, just point an infra-red thermometer at the sky. Depending on the amount of water vapor in the atmosphere, a temperature way above the universe background radiation will be read. I get -42 degrees C at the moment, but Sydney Australia is under a high pressure system which means not a lot of moisture around. (This idea is not original. I did think of it myself, but so have a lot of others…)

    So probably in for a relatively cool night. But way warmer than if the sky radiation was -270C or so.

    • gbaikie says:

      peterg says:
      September 5, 2016 at 3:30 AM
      “To directly measure the greenhouse effect, just point an infra-red thermometer at the sky.”
      Roy Spencer our host, also says this.

      I wonder if one can use the infra-red thermometer at higher elevation and by the measurement determine where the greenhouse effect is halved?

  50. mpainter says:

    Did you try the ketchup?

  51. mpainter says:

    Fine, Norman. You have difficulty expressing your views. To me, there is a paradox between “absorbed by the surface” and “does not warm”.
    Because___the back radiation varies according to the time of day (at any particular place).
    What about the clouds at night? Clouds passing overhead can add 20-50 W/sq m to back radiation…at night, with no insolation. By your view, this does not “warm the surface”. I believe that measurements indicate otherwise.

    • Ball4 says:

      mpainter – You make a good point here. Measurements are available.

      http://www.esrl.noaa.gov/gmd/grad/surfrad/dataplot.html

      Click on Goodwin Creek, it is showing your 1000 and 400 about now. Click on DWIR and air temperature. Plot data. Then you can click on previous day. Do that for a week or so back noting DWIR and T.

      Note sometimes DWIR spikes at night (passing cloud(s)) and air temperature does go up which is your “warm the surface” in action. (Solar DW also shows the passing clouds).

      You should also find DWIR spike ups at night when the air temperature misbehaves and goes down.

      There are some spectacular DWIR jumps too, I have to wonder if a bird alighted on the instrument.

      Measurements show both warming and cooling for increases in DWIR, moral of the story is other constraints need be imposed (no birds, no air column cooling breezes, etc.)

  52. Joel Shore says:

    Just to editorialize a little more: I like this analogy so much because it cuts through all the sophistry that people like P-stma and other who deny the greenhouse effect engage in. I.e., they never distinguish between “heating” in the sense of the net flow of energy being from the colder atmosphere to the warmer Earth (obviously a violation of the 2nd Law) and “heating” in the sense of causing the Earth to be warmer in steady-state than it would be if the net flow of energy away from the Earth were even larger (at a given temperature) than it is once back-radiation is taken into account.

    There is really no reason to be getting into these semantic conundrums because I personally don’t care if you want to say that the government made you richer or not. As long as we agree that
    (1) You are richer than you would be if the government did not give you the tax rebate.
    (2) The net flow of money is from you to the government.
    (3) It is only because you are earning your income that the government rebate causes you to become richer (rather than just to lose your wealth at a slower rate).
    …Then, we agree on the science and the rest is semantics.

    • Joel Shore says:

      Just one comment on the semantic issues: I think the problem arises because while heat as a noun has a precise definition in physics, “heat” or “warm” as a verb really does not. So, the question, “Does back-radiation warm the surface?” is simply not a well-defined question because the term “warm” is not defined. Norman has explained well the sense in which back-radiation does or doesn’t warm the surface. Without a precise definition of what “warm” as a transitive verb means (particularly in the case where the surface is receiving energy from more than one object), it simply makes no sense to ask the question of whether it warms it or not.

      • Norman says:

        Joel Shore

        Good point on the semantics.

        I did like your financial analogy but it probably will not make things any clearer.

        I think this type of debate has been going on quite some time

      • jerry l krause says:

        Hi Joel,

        Since the topic is semantics, I have a question: What is your definition, as a physicist or the word: equilibrium?

        Have a good day, Jerry

        • Joel Shore says:

          Jerry,

          I am not particularly interested in defining words that I haven’t used. I tend to avoid the term “equilibrium” myself, although I see others have used it a lot here (and I can’t say I never slip up myself), as I prefer “steady-state”. The reason is that the system is not in thermodynamic equilibrium.

          On the other hand, I may be too much of a stickler on this (coming from the statistical physics background that I do) as there are other types of equilibrium besides thermodynamic equilibrium (which is why giving one overarching definition may be difficult) and I have heard people apply “radiative equilibrium” in the context of atmospheric science.

          In fact, the Wikipedia page on radiative equilibrium notes: “Radiative equilibrium is one of the several requirements for thermodynamic equilibrium, but it can occur in the absence of thermodynamic equilibrium. There are various types of radiative equilibrium, which is itself a kind of dynamic equilibrium.”

    • mpainter says:

      Your tax analogy, Joel, holds a different significance to me. I see that terms and logic of science and physics do not serve to convey succinctly the GHE, or the process of back radiation, or the involved physical processes. That one should resort to such an analogy as yours underscores the uncertainty of the science. Plus, your analogy does not really seem apt: income taxes, rebates, etc., seem far removed from the issues of science that we strive to resolve.

      • Ball4 says:

        I agree mp, analogies always have holes. In this one the Fed could create the money out of thin air (money not a conserved quantity in an economy) and the taxpayer not be a bit wealthier as prices increase likewise. For example, you get a raise of 9% and apples increase from $1 to $1.09.

        • Joel Shore says:

          Ball4: You are introducing all sorts of real-world complications with money that, yes, would make the analogy inapplicable. ****So, the solution is to not introduce such complications.*** I don’t think it takes a great imagination to say: “Let us suppose that money is a conserved quantity, so ignore complications like the Fed creating it out of thin air, etc., the decreasing value of money due to inflation, …”

          Look, mpainter is stuck on some very basic stuff like how conserved quantities behave in extremely simple situations. If he can’t understand how it works in physics, it makes sense to try to help him understand it in a case where he may be more familiar and may not have the same sort of mental blocks he has to understanding it in the physical world when it is telling him things about the world that he ideologically cannot accept.

          • mpainter says:

            Mental blocks, Joel? When will you, and the rest of the zealots realize that the science is not settled?

          • Ball4 says:

            Disagree Joel, the analogies will never help me, mp or most if not all others. What will always help is the experiment Dr. Spencer did in the previous post & the (simple analogue that shouldn’t be pushed too far) analysis Dr. Spencer did in the top post. I will admit msm does try to use analogies (GHE!) for their readership but among those with some deeper and proper training in the field, a proper experiment will always work.

            It is unfortunate that mp doesn’t pay attention to proper experiments and asserts the opposite; wisely use that as a tool to learn why writers ignoring proper experiments do so at their own expense not at the expense of those who learn from the experience. Those folks are the richer.

            “You are introducing all sorts of real-world complications”

            Well, duh, yeah, that’s exactly why the analogy approach always has holes, better to use the real world in simple, proper experiments. This is why thermo. classes (et. al.) have that lab period attached. They don’t have analogy period.

      • Tim Folkerts says:

        Mpainter, I get the opposite message!

        The language of physics DOES succinctly explain the GHE — but only to those who actually understand the physics to begin with. For those who don’t understand the physics, then imperfect analogies may be helpful as a stepping stone toward better understanding.

        Analogies are ways for those with mastery of an idea try to convey that idea to someone without mastery using a setting that is potentially more familiar to he student. The analogy is a crutch for the student.

        • mpainter says:

          Thanks for your response, Joel.

          ” The language of physics DOES succinctly explain the GHE”
          ###

          Always a need to examine the assumptions behind the math. What say you Joel, is “back radiation absorbed by the surface” without warming the surface?

          And by “warming” I use that word in the usual sense.

        • jerry l krause says:

          Hi Tim,

          You wrote: “The language of physics DOES succinctly explain the GHE but only to those who actually understand the physics to begin with.”

          Joel Shore, another physicist, was discussing semantics. So, I asked him what his definition, as a physicist, of the word–equilibrium–was. So I am curious what your definition of this word, equilibrium, might be?

          Have a good day, Jerry

          • Tim Folkerts says:

            Jerry, there are various definitions depending on the circumstances. Since we are looking at thermodynamics here, then a definition of thermodynamic equilibrium would be appropriate. Something like “In thermodynamic equilibrium there are no net macroscopic flows of matter or of energy, either within a system or between systems. “[from Wikipedia]

            I should note that for the sun/earth/space situation under consideration, we should really be talking about “steady-state” (which allows for balanced flows in and out) rather than true “equilibrium”.

          • Ball4 says:

            Agree Tim steady state more meaningful than equilibrium for Earth system.

            Jerry asks a good question too, deserves an answer, equilibrium must be defined in each use as to what exactly the author means, good texts will do that, not leave it up to the reader to figure out. Since thermodynamics is all about equilibrium states though, some say it should be called thermostatics.

            Jerry – I looked in Bohren 1998 and found: necessary conditions for strict thermodynamic equilibrium are no change with time of macroscopic thermodynamic variables, no gradients of those variables, and no dependence of the macroscopic system on its history.

            At the molecular level of course change is rapid and incessant.

          • alphagruis says:

            The climatic system is from a thermodynamics point of view of course not at all a system that is in equilibrium. It’s even a formidable problem of non-equilibrium thermodynamics and more precisely a system that is far from thermodynamic equilibrium in the sense defined by Nobel Price Ilya Prigogine, that locally produces entropy and dissipates energy continuously, yet organizes matter at macroscopic level and spontaneously creates order as here in trades winds, hurricanes or living creatures.

            See the concept of dissipative structure: https://en.wikipedia.org/wiki/Dissipative_system

            Only local thermodynamic equilibrium may be invoked in tiny parts of such systems.

            As pointed out by Tim Folkerts some “steady state” situation may be established were (mean)
            energy fluxes are constant and balanced and it is this kind of “equilibrium” that is usually invoked by climatologists as opposed to bona fide “thermodynamic equilibrium”. The latter for instance would imply first of all a uniform temperature everywhere the same in the whole system.

          • jerry l krause says:

            Hi Tim, Joel, Ball4, and alphagruis,

            Thank you all for responding to my question (which I now paraphrase): What is your definition of ‘equilibrium’ as a physicist?

            Joel’s response: “I am not particularly interested in defining words that I havent used. I tend to avoid the term equilibrium myself, although I see others have used it a lot here (and I cant say I never slip up myself), as I prefer steady-state. The reason is that the system is not in thermodynamic equilibrium.” seems to summarize each of your responses.

            I asked this question because I am a chemist and I recently discovered how differently this word, which I consider a very fundamental word of science, has been taught in chemistry and physic textbooks.

            In Physical Chemistry 2nd Ed by Walter Moore, page 5, I read: “5. Equilibrium. The ordinary subjects for chemical experimentation are not individual particles of any sort but more complex systems, which may contain solids, liquids, and gases. A system is a part of the world isolated from the rest of the world by definite boundaries. The experiments that we perform on a system are said to measure its properties, these being the attributes that enable us to describe it with all requisite completeness. This complete description is said to define the state of the system.

            “The idea of predictability enters here; having once measured the properties of a system, we expect to be able to predict the behavior of a second system with the same set of properties from our knowledge of the behavior of the original. This is, in general, possible only when the system has attained a state called equilibrium. A system is said to have attained a state of equilibrium when it shows no further tendency to change its properties with time.”

            Tim wrote: “I should note that for the sun/earth/space situation under consideration, we should really be talking about steady-state (which allows for balanced flows in and out) rather than true equilibrium.”

            Now, it seems that the steady state condition to which you all refer is the necessary condition for equilibrium. Yet, it seems you all do not associate the steady state condition as being a system at equilibrium.

            Now, the reason for my question was when I went to a physic text book by Marshal and Pounder, page 74, I read: “According to Newton’s Second Law of Motion, if all the forces on a body balance each other so that there is no net or resultant force, the body has zero acceleration and is said to be in equilibrium.”

            Is this difference of definition of the word, equilibrium, the reason that each of you prefer steady state to equilibrium?

            For a moment I questioned what Tim’s “true “equilibrium”” was. But then I found that Ball4 had written: “I looked in Bohren 1998 and found: necessary conditions for strict thermodynamic equilibrium are no change with time of macroscopic thermodynamic variables, no gradients of those variables, and no dependence of the macroscopic system on its history.” This answered two questions for me. For I not only did not understand what Tim’s true equilibrium was, but I also did not understand what a thermodynamic equilibrium was.

            Am I correct if I understand a thermodynamic equilibrium to be the “equilibrium” which Morrow defined?

            Now, relative to equilibrium systems and hence, non-equilibrium systems, I ask: Is the earth’s atmospheric system ever an equilibrium system?

            Have a good day, Jerry

          • alphagruis says:

            Jerry,

            First one should not confuse mechanical and thermodynamic equilibrium. They are really very different things.

            A system in thermodynamic equilibrium is as defined in your Bohren reference: Uniform pressure, temperature, i. e. no gradients with entropy, internal energy etc that do not change with time. In particular there is no energy dissipation and production of entropy in such a system.

            So the earth atmospheric system is definitely never in thermodynamic equilibrium but it may be in a steady state of non equilibrium where for instance its internal energy remains constant but there is a vertical temperature gradient or lapse rate. As pointed out previously the very presence of this temperature gradient implies a system that is not in thermodynamic equilibrium. Moreover there is continuous dissipation and production of entropy i.e. the total entropy of the (system + surrounding) steadily increases.

            As a most remarkable example of far from thermodynamic equilibrium system is a living creature. In it’s grown-up state it may be in a steady state of non equilibrium (no change in weight, flow of energy in and out cancel, but continuous production of entropy exported to its environment. This is in sharp contrast with when it grows after birth, no steady state in this case, weight increases, energy in exceeds energy out, it’s entropy and internal energy change with time, but there is still continuous dissipation of energy and production of entropy exported to its environment, which is what characterizes systems out of thermodynamic equilibrium..

      • mpainter says:

        I meant Tim, not Joel, excuse please.

      • Joel Shore says:

        “That one should resort to such an analogy as yours underscores the uncertainty of the science.”

        What a bizarre statement. The fact that we have to resort to analogies simply illustrates the weaknesses in the understanding of those who we are talking to. It is not necessary for us to resort to such analogies when we are talking to other physicists or climate scientists.

        And, I explained to you exactly why the analogy is applicable: The scientific issues aren’t complicated. You are stuck on some very basic issues like how a conserved quantity behaves, how one gets a steady-state solution and how that solution behaves in some simple instances, … And, for these issues the analogy is in fact perfectly well-suited. I think the reason you don’t like it is that it gives you a result that you don’t want to believe so you are desperately clinging to the notion that somehow the analogy fails.

        • mpainter says:

          No, not a bizarre statement, a considered opinion. Abandoning scientific terms for stretched analogies seems to me symptomatic of the present state of climatology, as in the inflammatory press releases that accompany so much of climate science.

          As for the rest of your comment, you obviously regard the science as settled. Don’t expect the rest of us to agree. Plus, you cannot show that atmospheric CO2 is anything but beneficial for the biosphere.

        • Joel Shore says:

          Don’t blame us and a field of science for your ignorance and inability to understand basic science. As I noted, when I have discussions with scientifically-literate people then I don’t need to give them such analogies. In fact, my physics colleagues wonder why I would waste my time talking to people who can’t understand the greenhouse effect and deny this piece of basic physics. Sometimes I wonder too.

          You are entitled to your opinions about climate science. Just don’t expect them to care any wait amongst people literate in physics because they absolutely won’t.

          • Joel Shore says:

            Sorry…In my haste, I wrote “care any wait” instead of “Carry any weight”. Go figure.

          • mpainter says:

            Joel, I am always willing to discuss issues of science. But discussing far-fetched and inept analogies has no appeal to me.

            And still no science from you, only an extended rant against my “ignorance” for not swallowing your stretched analogies.

  53. Norman says:

    mpainter

    Here are links to your questions. Maybe seeing it will help you to understand what I am saying.

    http://www.esrl.noaa.gov/gmd/webdata/tmp/surfrad_57cf166c942ad.png

    It show a winter period with a very cloudy night. Temperatures do not drop and the UWIR and DWIR are very equal. The energy added to the surface by DWIR is removed at the same rate as the UWIR leaving the kinetic energy of the surface molecules unchanged during the all night exchange of energy.

  54. Norman says:

    There may be times (mostly winter months) were DWIR can warm the surface. This would be due to a temperature inversion where the atmosphere above is warmer than the ground below.

    This could be an example. Have to look at the local weather conditions because more is going on for specific locations than radiation. It is only one of the energy budgets. Local areas not only have to consider evaporation, convection and conduction but air masses moving in from other locations.

    I just picked this graph because it did show DWIR higher than UWIR and warming night time temperatures.

    http://www.esrl.noaa.gov/gmd/webdata/tmp/surfrad_57cf198728345.png

  55. Norman says:

    http://www.esrl.noaa.gov/gmd/webdata/tmp/surfrad_57cf1bed1cf7e.png

    Current Desert Rock. DWIR much below UWIR and the surface is cooling.

  56. Norman says:

    mpainter,

    For the most part on all the graphs I have looked at the DWIR does not warm the surface in the normal sense.

    By lowering the amount of radiant energy leaving, however, it will allow the constant solar input to increase the temperature of the surface leading to a warmer surface than one without DWIR.

    • Kristian says:

      Norman says, September 6, 2016 at 1:54 PM:

      By lowering the amount of radiant energy leaving, however, it will allow the constant solar input to increase the temperature of the surface leading to a warmer surface than one without DWIR.

      Again, for the nth time: In your scenario THERE IS NO LOWERING OF THE AMOUNT OF RADIANT ENERGY LEAVING THE SURFACE, Norman!!!! There is only more radiant energy ENTERING the surface. Warming it directly. This is not “insulation”. This is tantamount to extra “heating”. How can you not see this!?

      Again, the “back radiation” EXPLANATION is wrong. The insulation effect isn’t.

      • Norman says:

        Kristian

        I know you don’t believe those measurements in my links are real (even though they can calibrate the instruments to real temperatures in the lab to get correct readings. I take an IR gun a work and point the laser at different parts and it does quite a good job of giving the same temperature arrived at by other means) but they explain quite well what is going on with the GHE.

        I do see I did not post my thoughts correctly. I should have put the word NET in front of my radiation.

        My corrected sentence: “By lowering the amount of NET radiant energy leaving, however, it will allow the constant solar input to increase the temperature of the surface leading to a warmer surface than one without DWIR.”

        From your post: “here is only more radiant energy ENTERING the surface. Warming it directly. This is not insulation. This is tantamount to extra heating. How can you not see this!?

        Again, the back radiation EXPLANATION is wrong. The insulation effect isnt.”

        It is very obvious you are making these comments without even trying to take a look at the nice links I put in for mpainter to review. I realize I can’t communicate the ideas to him with words but thought visual images of the process would demonstrate the concept.

        • Norman says:

          Kristian

          http://www.esrl.noaa.gov/gmd/webdata/tmp/surfrad_57cf64c42d54c.png

          I am linking to a graph of UWIR, DWIR and net IR with air temperature.

          If you take a brief moment to look at the graph it will demonstrate clearly what I am saying. You ask “How can I not see this?” that the DWIR is warming directly.

          If you look at the graph you will see your thoughts are incorrect and on some strange tangent that makes little physical sense. The back radiation explanation is not wrong. You nee to invest in a good physics thermodynamics book and correct all the flaws in your understanding of how radiant energy works. All current physics books accept that all matter, with temperature, emits radiant energy (EMR). The amount of radiant energy a surface emits is based solely upon its temperature and emissivity. Radiant energy is either reflected, transmitted, or absorbed. It is real energy and will add kinetic energy to a surface if absorbed. The reason a hot surface with a colder one around does not warm is not because the cooler one stopped radiating, it is because the warmer one is emitting radiant energy away (real energy, real joules) at a faster rate than it is absorbing radiation from the cooler surface making the net heat flow from hot to cold.

          All the graphs show this. That you need to stick you head in a hole in the ground is not my problem and tell me I am wrong repeatedly when my thought process is based upon the current understanding of thermodynamics. I have posted links to textbooks to show the thought process. When I did I got tons of flak because it was printed by MIT.

          The ESRL graphs show exactly what the textbooks say should happen I do not know why I need to subscribe to your twisted and incorrect physics. Maybe you are the one who should study modern physics instead of claiming everyone in the world, all thermo scientists, some who post on this blog, are wrong and you are the only one in the Universe who figured it out correctly even though you have zero supporting evidence for you vast amount of posts. Link me to a scholarly paper that in any way supports the physics you hold firm to.

        • Kristian says:

          Norman says, September 6, 2016 at 6:51 PM:

          I know you dont believe those measurements in my links are real (even though they can calibrate the instruments to real temperatures in the lab to get correct readings.

          Well, the temperature measurements are real, and the radiant heat flux (‘net LW’) measurements are real. All other values, however, are only COMPUTED, Norman. So in the sense that those computations are ‘real’, then …

          You still see two separate, opposite arrows when you look at a thermal radiation field. Those arrows are mathematical constructs only … In reality there is but ONE movement of radiant energy through the thermal radiation field – the net, the probabilistic average, of ALL microscopic (quantum) movements through ALL points in its threedimensional space.

    • mpainter says:

      Norman,
      “By lowering the amount of radiation leaving..”

      ###

      Are you saying that “back radiation absorbed by the surface” _lowers_ the amount of radiation leaving? This is cooling, it appears.

      ========================

      Norman: “it will allow constant solar input to increase the temperature..”

      ###

      Where is solar input constant? Sun rises, surface warms; Sun sets, surface cools. Whether there be back radiation or none.

      • Norman says:

        mpainter

        Yes your statements are correct. But you only tell part of the reality.

        I have already stated solar insolation is the only input that actually heats a surface on its own. The combination of the solar insolation and DWIR will allow the surface to reach a higher temperature.

        The surface will cool but without back radiation it will cool to a much higher degree than with a surface with backradiation.

        And if you take even a brief time to look at the ESRL graphs I linked you too you will see strong evidence that the amount of backradiation has considerable effect on the cooling rate of the surface. Real world measured values, this would be considered the empirical proof of the concept that so many need to reject for unknown reasons.

        • Kristian says:

          Norman,

          This is so silly! You state, without reservation: “By lowering the amount of radiant energy leaving, however, it will allow the constant solar input to increase the temperature of the surface leading to a warmer surface than one without DWIR.”

          1) In your scenario, you are specifically NOT lowering the amount of radiant energy leaving, you are specifically INCREASING the amount of energy ENTERING. To create extra warming.

          Hence,

          2) It is specifically NOT the constant SOLAR input that further raises the temperature in your scenario. It is specifically – directly and all by itself – the extra input of energy from the “DWLWIR”.

          Just follow the energy, Norman:
          https://okulaer.files.wordpress.com/2014/10/drivhuseffekten.png

          In your scenario, there is NOTHING to obstruct any energy at any time from ESCAPING the surface, and the solar input is, as you say, constant. So the ONLY flow of energy in your scenario that can create the EXTRA warming, beyond the pure solar equilibrium, from 232 to 289 K, is the “DWLWIR” from the atmosphere.

          You say:

          I have already stated solar insolation is the only input that actually heats a surface on its own. The combination of the solar insolation and DWIR will allow the surface to reach a higher temperature.

          No, Norman. Beyond pure solar equilibrium (165 W/m^2 IN = 165 W/m^2 OUT, at an ideal 232 K), the Sun cannot contribute to any further increases in temperature. In your scenario. So the final increase in temperature, from 232 to 289 K, is solely caused by the “DWLWIR”. In your scenario:
          https://okulaer.files.wordpress.com/2014/10/drivhuseffekten.png

          The surface will cool but without back radiation it will cool to a much higher degree than with a surface with backradiation.

          No, in your scenario (the “back radiation” EXPLANATION), the “back radiation” specifically constitutes an extra INPUT of energy to the surface, not a reduction in the OUTPUT of energy from the surface. That’s extra “heating”, not extra “insulation”.

          You will just have to change the way you view and describe this process, Norman. Because as it stands, you are promoting nonsense physics …

          • mpainter says:

            “nonsense physics”.

            As in “back radiation absorbed by the surface” without warming the surface?

          • Norman says:

            Kristian,

            It seems nonsense to you because all your knowledge of physics seems to have been derived by you on your own using your own logic and common sense and not consulting textbook knowledge based upon many experiments and tested applications.

            I post links for you to observe and see for yourself. You reject them based upon your own internal understanding of how the measuring devices work even though in lab tests they seem to be able to measure the temperatures of objects just fine.

            If you will not look at any links it becomes pointless to keep debating with you. I offer empirical evidence which you claim my points are nonsense. You will not look at the evidence or reject it because it shows strong evidence you are incorrect in your current understanding so you have to make claims that it is not measuring what they scientists claim it does, somehow you seem to be the much wiser than they.

          • Norman says:

            Kristain

            I will reload and try again.

            Please just link to the graph and read my points.

            YOU Posted: “1) In your scenario, you are specifically NOT lowering the amount of radiant energy leaving, you are specifically INCREASING the amount of energy ENTERING. To create extra warming.”

            No sir that is not at all what is going on and the evidence demonstrates your point is senseless.

            http://www.esrl.noaa.gov/gmd/webdata/tmp/surfrad_57cf85e117aba.png

            The actual evidence clearly shows the DWIR is not greater than the UWIR and the surface is cooling, the rate of cooling is what is significant. The NET radiant energy loss from the surface with DWIR or “backradiation” is around minus 200 W/m^2. Without the DWIR the surface would be losing energy at the rate of around 500 W/m^2

            Watts/m^2 is equal to joules/sec-m^2. Joules are what detemine the internal energy of a body, if more of them leave the surface the body will cool faster. Without GHE the surface will cool at a much greater rate.

          • Norman says:

            Kristian

            I think I might be closing in on why you do not understand the GHE and continue to see the scientific world (including Dr. Spencer himself) as misguided in their current understanding.

            YOU STATE: “In your scenario, there is NOTHING to obstruct any energy at any time from ESCAPING the surface, and the solar input is, as you say, constant. So the ONLY flow of energy in your scenario that can create the EXTRA warming, beyond the pure solar equilibrium, from 232 to 289 K, is the DWLWIR from the atmosphere.”

            You think a constant input can only warm to a certain temperature for some unknown reason. Something in what you think is common sense. The truth is a constant input of energy will pile up. A constant flux of 165 W/m^2 means the solar input is adding 165 new joules of energy into the surface every second per square meter.

            If the energy is not lost at the same rate in the first second the square meter has 165 additional joules. The next second 330 joules. A constant input means you will just keep piling up energy. The temperature of a surface is unknown with only the incoming energy. 165 Watt/m^2 can raise the temperature several degrees, thousands of degrees as long as the energy can be added.

            The solar input can raise the Earth’s surface to equivalent of the Sun’s surface if the radiant energy is not allowed to exist.

            The cool temperature of an equilibrium surface with the 165 flux is because that is the temperature solar energy can raise a surface that can emit radiation at its maximum rate. If something prevents the outward flow the equilibrium temperature will go up and it can reach fantastic high temperatures depending upon how much the outgoing energy is prevented from leaving.

          • Kristian says:

            Norman says, September 6, 2016 at 9:11 PM:

            It seems nonsense to you because all your knowledge of physics seems to have been derived by you on your own using your own logic and common sense and not consulting textbook knowledge based upon many experiments and tested applications.

            No. It seems nonsense physics because it is nonsense physics. I am not using “my own homemade” physics to try and show you this, as you keep telling yourself. I am using simple, straightforward, regular physics. Regular thermodynamics, regular radiation physics, regular quantum physics.

            I’m sorry, Norman, but YOU’RE the confused one here. I know PERFECTLY well what you are trying to say. But it’s just muddled … You have a closed mind on this issue, that’s pretty obvious.

          • Kristian says:

            mpainter says, September 6, 2016 at 8:26 PM:

            “nonsense physics”.

            As in “back radiation absorbed by the surface” without warming the surface?

            Yes. Like energy ADDED to the surface somehow reducing the OUTGOING energy. Like the ADDED energy from the atmosphere somehow “allowing” the (assumed constant) SOLAR input to magically create further warming.

        • Norman says:

          mpainter

          I can’t force you to look at the links I put in my posts to you but rather than make statements as you did:

          nonsense physics.

          As in back radiation absorbed by the surface without warming the surface?”

          I have stated the concept is difficult for me to explain to you in words that you and I both accept and agree upon definition and can then logically deduce what is being said.

          That is why I chose visual evidence in the form of graphs of measured values.

          It clearly shows why back radiation absorbed by the surface (except in a few extreme cases of temperature inversions) does not warm the surface, primarily because the surface is emitting energy at a greater rate.

          • Norman says:

            mpainter

            I would take you define warming the same as I do. An increase in temperature over a previous value?

            I also believe you accept that the temperature of an object is the result of its internal energy, and type of material it is made out of.

            Do you likewise believe electromagnetic radiation possess energy within itself and that this energy, contained in electric and magnetic fields, can add or subtract internal kinetic energy from a body via surface interaction?

            Surface radiates away EMR and it loses energy and cools?
            Surface absorbs radiation and gains energy warming?

            Do you accept that is you add energy to an object but it is losing energy at faster rate than you are adding energy, it will cool? Simple example. You are trying to warm an iron bar with your hand (source of heat) that is immersed mostly in liquid nitrogen.

            Do you accept that if you are adding energy to an object faster than it is losing energy that it will warm?

            If you accept all these or understand them in a similar fashion to how I do then why do you think that with two radiant energy fluxes going on (incoming and outgoing) at the same time that one that is adding energy being of a lesser value than the process losing energy that the lesser value would indeed be able to warm the surface?

          • mpainter says:

            Norman, I have looked at your links, thank you. I see upwelling radiation correlated with air temperature, and back radiation only weakly correlated to air temperature. I see it rise when clouds pass overhead.

            Did you see these things?

          • mpainter says:

            Yes Norman, yes to all of that. There is one more yes: that 400 W/sq m radiation added to 1,000 W/ sq m radiation will sum to 1,400 W/sq m radiation. Yes.

          • Norman says:

            Kristian

            Again: Please look at the link and just for the sake of thought process consider the fluxes you see are real measured values of energy flows.

            http://www.esrl.noaa.gov/gmd/webdata/tmp/surfrad_57cf85e117aba.png

            YOU Claim I am wrong and try to convince yourself I am. Reality is on my side.

            YOU: “No, in your scenario (the back radiation EXPLANATION), the back radiation specifically constitutes an extra INPUT of energy to the surface, not a reduction in the OUTPUT of energy from the surface. Thats extra heating, not extra insulation.

            You will just have to change the way you view and describe this process, Norman. Because as it stands, you are promoting nonsense physics ”

            If you look at the link it clearly shows that back radiation is not an extra INPUT of energy. IN reality its overall effect is a reduction in the OUTPUT of energy from the surface and it acts quite similar to insulation even though the processes are different in nature the overall result is the same. Each, insulation and backradiation slow the ability of a surface to cool. Without an input of energy it would not matter. With an input of energy it makes a huge difference.

            https://okulaer.files.wordpress.com/2014/10/drivhuseffekten.png

            In the graph you link to the backradiation is not wamring the Earth by itself. The Earth surface is radiating at the rate of 398 W/m^2. The backradiation is 345 W/m^2. It can’t heat the surface by itself and the diagram does not show it does. Only your faulty and incorrect reasoning makes you think that is what the diagram shows.

            It does show with back radiation the Earth’s surface is only losing radiant energy at the rate of 53 Watt/m^2 because of the backradiation.

            If not for the backradiation the Earth’s surface would be losing energy at 398 W/m^2 from radiation alone. If you add the other loses of surface energy (112 W/m^2) you would lose 510 W/m^2 and with only 165 W/m^2 incoming solar your surface is going to cool very fast and keep cooling until it reaches the equilibrium temperature 165 W/m^2 can sustain. As the surface cools the radiant and other forms of energy transfer from the surface will go down until they equilibrium with the solar incoming flux.

  57. Norman says:

    mpainter

    Yes, I agree with you that 1000 watts/m^2 plus 400 watts/m^2 will equal 1400 Watt/m^2.

    Now what is it you want to determine with this conclusion? Where do you go with this?

    You made this point above: “So, Norman, lets see if I understand you correctly. Lets take the hypothetical case of tropical ocean with a clear sky insolation of 1,000 W/sq m and a back radiation of 400 W/sq m, both of these radiative fluxes absorbed by the surface for total of 1,400 W/sq m absorbed by the surface.
    So, if I understand correctly, the surface would not warm according to the total of 1,400 W, but only according to the 1,000 W of insolation. Is this correct?
    And the reason that the 400 W/sq m caused no warming is because the surface radiated that before it could warm anything. Right?”

    I think it is correct to add the two fluxes as you have. So you have 1400 W/m^2 being absorbed by the ocean surface. How much will the ocean surface warm? The answer to this would be based upon how long this flux was maintained and how much energy was being lost at the same time.

    With a 1000 W/m^2 flux and 400 DWIR for a total of 1400 W/m^2 the ocean will warm. But then when the solar insolation drops to zero and the DWIR is still 400 W/m^2 but the UWIR of the ocean surface is 460 W/m^2 the surface will cool via radiant loss as well as the other energy loss processes (evaporation and convection).

    So again is the DWIR warming the ocean surface? Without the solar insolation the water will not warm but only cool? Together they can warm the ocean more than the single solar flux.

    • mpainter says:

      Norman, you say “I think it is correct to add the two fluxes as you have. So you have 1400 W/m^2 being absorbed by the ocean surface. How much will the ocean surface warm? The answer to this would be based upon how long this flux was maintained and how much energy was being lost at the same time.”

      ####

      The ocean warms. And warms. And warms. Because water is transparent and the insolation is absorbed at depth. As deep as 100 meters. So from day to day the ocean warms because the heat is cumulative, that is, the oceans do not cool at night except partly. Each day adds more heat to the 100 meters. Every night only a fraction of that is lost. The heat and the temperature rise day after day.
      Until the ocean’s surface waters (the upper 300 meters) circulate above a certain latitude. Then they cool. And cool. And cool. They cool to 0 C.

      So what does this have to do with the energy budget diagrams?
      Answer: nothing.

      • Norman says:

        mpainter

        Do you have evidence of your point that water would just keep heating unless the ocean currents moved it North or it just your opinion of how the system works?

        Do you have local heat budget (not the global one) but an actual specific heat budget of tropical water to demonstrate your point?

        CERES might be able to help you provide evidence. Otherwise your post is an opinion just founded on your belief of how it should work.

        The global energy budgets are based upon actual measured values. What they do is smear real measured values and average them out among all the Earth’s square meters. This may not be a correct process but it is based upon actual measured values. I do not know that yours is such unless you can come up with some values. Some real numbers.

        • mpainter says:

          Evidence: its not a matter of evidence, Norman, it is a matter of textbook oceanography. Do you understand : *meridional overturning circulation*? If you do, you know that the tropical oceans warm from circa 4 C (upwelling) to 30 C.
          Study SST, tropical Pacific. Contrast ENSO regimes. It is there in the data. If you can’t think it through you are lost.
          And no, it is not “just my opinion”. It is oceanography. “Real numbers”.
          The oceans accumulate heat in the tropics (warming) and release in the high latitudes (cooling). Even the turniphead Trenberth has said so.

    • Kristian says:

      Norman says, September 6, 2016 at 10:18 PM:

      With a 1000 W/m^2 flux and 400 DWIR for a total of 1400 W/m^2 the ocean will warm.

      Ok. So will [1000+400=] 1400 W/m^2 warm the ocean *more* than just 1000 W/m^2? And if so, how/why?

      So again is the DWIR warming the ocean surface?

      Yes it is. Directly. Forcing the surface to cool more (increased “UWLWIR”). In your scenario.

      Without the solar insolation the water will not warm but only cool?

      Er, but the same is true if you remove the “DWLWIR”, Norman. The surface T_avg will go down, not up. Still, we KNOW that the solar flux is a direct HEAT input.

      IOW: In your scenario, the solar flux and the “DWLWIR” are considered thermodynamic equivalents, energy inputs to directly create warming – if they are removed, the surface will cool, if they are added back, the surface will warm.

      Meaning, they are both TREATED as radiant heat fluxes. Even though only the solar flux happens to be one. The “DWLWIR” is physically an integrated part of the surface heat LOSS …

      • Ball4 says:

        “they are both TREATED as radiant heat fluxes.”

        Mostly by Kristian which results in confusing comments/conclusions by using the heat term incorrectly to make twisty pretzel comments when actually scientists treat SW and LW as radiant energy fluxes each photon with energy hf so not thermodynamic equivalents.

        Neither red nor yellow arrow(s) in the top post are heat Kristian.

      • mpainter says:

        Vermin will seize upon the confusion extant in thermodynamics concerning the term “heat” and its ambiguous usages to sow further confusion. As a deliberate tactic. Wise persons will not allow themselves to be baited.

  58. Norman says:

    Kristian

    YOU: “Er, but the same is true if you remove the DWLWIR, Norman. The surface T_avg will go down, not up. Still, we KNOW that the solar flux is a direct HEAT input.”

    It will go down from what it is with DWIR but it will only go down to a new equilibrium temperature and remain there.

    With just DWIR and no solar input the temperature will just continue to drop until it reaches the new equilibrium temperature of input from the Earth’s interior and outer space and I don’t know what they number would be.

    The two fluxes are considerably different in what they are able to do.

    YOU: “Meaning, they are both TREATED as radiant heat fluxes. Even though only the solar flux happens to be one. The DWLWIR is physically an integrated part of the surface heat LOSS ”

    Ball4 already addressed your claim. DWIR is not treated as a heat flux (except in rare temperature inversions where the emitting atmosphere is warmer than the surface, in this condition DWIR is greater than UWIR and the surface can be warmed by DWIR making it a heat flux under these conditions).

    • mpainter says:

      Sorry, Norman, but you say “DWLWIR is not treated as a heat flux” and b.s. button on that. You have slipped a mental gear. Let’s see if you even figure it out.

      • Norman says:

        mpainter

        I did explain that if there is an inversion the DWLWIR could be considered a “heat flux”. In this case the warmer atmosphere will transfer energy to the colder surface, which is then a heat flux.

        If you believe I err it will probably how you define “heat flux”. It seems this is were lots of confusion takes place when opposing sides state their positions.

        In my definition a heat flux only takes place from warmer to cooler. It is a NET energy flow not an individual flow.

        Any individual flux will not be a “heat flux” since the concept is a NET flux of all radiant inputs. The solar insolation will not be a “heat flux” at an equilibrium state since it will not add warmth to the surface. It would be a heat flux if the NET energy from it was positive.

        The best way to approach it to avoid confusion is to listen to Ball4 on this one. Each flux can only be considered a radiant energy flux with direction (Up or Down). It has energy but is not counted as heat.

        When you add all the fluxes together the you find out if there is any “heat flux”. It the temperature is not changing you will have no “heat flux” but you will still have the individual fluxes.

        • mpainter says:

          See referred cartoon in post. Note Tsurf. Note caption: “energy flow”. DWLWIR is depicted as a constituent of “energy flow” which determines surface temperature. By your lights, this is incorrect. DWLWIR cannot be part of the “energy flow” at the surface.

        • mpainter says:

          Note how the cartoon treats DWLWIR as part of the “radiant heat flux” (Kristian’s term) but calls it “energy flow”. Norman, I urge you to think this cartoon through. The difficulty that you (and others) have in depiction of the surface effects of the GHE stems from your denial of the this cartoon.

          Please don’t use the tactic of squabbling over whether the term “heat” refers only to a measurable property of matter (kinetic energy) or whether it refers only to the transfer of energy (a process rather than a property, radiant energy being identical to radiant heat in this sense), lest you convict yourself of sophistry.

  59. Norman says:

    Kristian

    In my scenario and the rest of the scientific world.

    YOU: “IOW: In your scenario, the solar flux and the DWLWIR are considered thermodynamic equivalents, energy inputs to directly create warming if they are removed, the surface will cool, if they are added back, the surface will warm.”

    No they are no equivalents. And no the DWIR does not directly create warming. It will warm with solar input but not on its own.

    If you start with Earth in a state at 200 C (cold but still having an atmosphere…not collapsed). Without a solar flux the surface will keep cooling even with a DWIR so it is not the same. If you have just solar and no DWIR in this case the solar will warm the surface to the radiant equilibrium temperature and hold it there indefinitely.

    So no they are not the same and no one treats them the same.

    With DWIR the solar input will raise the surface temperature to a new higher equilibrium (warming it) because the DWIR lowers the amount of NET radiant energy leaving the surface and the temperature will increase until the Net surface radiant energy matches the incoming solar. If you stopped the other heat transfer mechanisms from the surface, the surface would get much hotter still but it would reach a new equilibrium and remain there.

    Not real hard for most to understand. Seems impossible for you though.

    • mpainter says:

      Norman, see Dr. Roy’s “cartoon” at the head of this post. What does that tell you? Obviously the cartoon represents DWLWIR as part of the surface radiation flux.

      The question remains : how is the DWLWIR absorbed by the surface without warming the surface?

      You, Norman, have been struggling through dozens of comments trying to answer that question. And you have been all over the place in your explanations. Your latest is to deny that DWLWIR is part of the heat flux. You need to show a little consistency in your science.

      • Kristian says:

        Norman is deeply confused on this issue, mpainter. It’s as simple as that. Can he get himself to admit it? Of course not …

      • Norman says:

        mpainter

        I really have not been all over the place, it is fairly consistent.

        I am using the currently accepted definitions of terms used in a scientific context.

        The cartoon above has atmosphere flux as a radiant flux not as a “HEAT FLUX” that is your false addition not stated above.

        This will not make it any clearer but this is how current definitions are understood (Tim Folkerts can correct me if I am wrong but I highly doubt I am).

        You have two hot plates each at 1000 C facing each other. There is zero HEAT flow or flux between the two.

        You have another set of plates one at 10 C the other at 0 C that face each other. There is a heat flow between these two and it is greater than the heat flux of the 1000 C plates.

        Both sets of plates also have radiant energy fluxes. The 1000 C plates each have a much higher radiant flux than the cooler plates, they just have zero HEAT FLUX as the term is defined. It is the NET ENERGY FLOW between objects (can be two, or more).

        So if you have a problem with the definition I can’t be blamed for you lack of understanding. Maybe it is up to you to update your terms.

        • mpainter says:

          Norman, the cartoon is wrong, then? The cartoon represents back radiation as “absorbed by the surface” as an “energy flow” that determines temperature, Tsurf.
          You say this is wrong.

          Your still have not described how back radiation warms the surface without warming it. Perhaps you are thinking about the temperature gradient from surface to air. Perhaps you are trying to express the notion that back radiation lessens that gradient, hence it warms by retarding the rate of cooling at T1, hence it rises to T2. Is this what you mean?

          • Norman says:

            mpainter

            In the cartoon. Yes backradiation is absorbed by the surface. I have never stated anything different. Does it warm the surface?

            That depends, mostly not.

            Once again I will try. Backradiation does not warm the surface in most cases. It is a lesser flux than the outgoing flux and will not keep up with the outgoing energy.

            Does that make sense to you? Probably not.

            NOTE PLEASE: IT is the COMBINATION of solar flux and DWIR that warm the surface not the DWIR alone. Can you understand that point?

            On the daily cycle of day and night. The solar flux will most of the time warms the surface (extreme clouds may prevent this process). Solar flux will warm the surface on its own (nothing to do with the global budget, these are separate topics and should not be mixed together or it brings a mighty torrent of confusion for many). Even with no backradiation, during the day solar flux will warm the surface. DWIR alone will not warm the surface, it only leads to cooling surface. Look at the links again they show it so clearly!

            Can you grasp the difference here? The combination of the two fluxes will lead to a warmer surface than the solar flux alone will achieve.

            How can something so simple be so baffling to you? Even with empirical evidence for you to look at.

          • mpainter says:

            Norman, nothing is as plain, clear as the diagram in the cartoon. Clearly it means that back radiation is warming the surface. Your view defies the laws of physics: that the absorbed back radiation does not increase the kinetic energy of the absorbing molecules.

          • Norman says:

            mpainter

            One problem is you do not read my posts. I think you quick scan them looking for some material. Evidence: In your post you make this claim: “Your view defies the laws of physics: that the absorbed back radiation does not increase the kinetic energy of the absorbing molecules.”

            I already addressed that in a post above to clarify for you. Saying my view defies the laws of physics when you do not know my view is a strange sort of conclusion.

            Here is what I wrote above. Will you read it? Most likely not. I will post it as evidence that you are making claims that are not mine.

            ME ABOVE ON THIS SAME THREAD: “A surface molecule absorbs a DWIR and starts to jiggle more which is picked up by the surrounding molecules and it would seem that the overall kinetic energy would go up so why doesnt it? That is because for this one molecule that absorbed an IR photon and increased its local kinetic energy there were two molecules elsewhere on the surface that emitting photons (same time) and lost more kinetic energy than the surface gained by the one absorbed photon. With this process going on trillions plus times per second you have a net loss of energy to the cooler atmosphere.”

          • Norman says:

            mpainter

            Your conclusions are most difficult to determine the logical pathways that lead to such conclusions.

            I your post you state as a conclusion: “Norman, nothing is as plain, clear as the diagram in the cartoon. Clearly it means that back radiation is warming the surface.”

            How do you logically arrive at this certain conclusion about the cartoon? There are no values to the fluxes.

            Roy himself states: “Here are the basic energy fluxes included in the model. The illustration is just schematic.”

            It is only showing direction of flows and not magnitude. Based upon this information how do you conclude the cartoon “Clearly it means that the back radiation is warming the surface”?

          • mpainter says:

            Norman, study the Stefan-Boltzmann principle. That is the key. If you can grasp that, it should help clear up your confusion.

            A few comments on S-B: this concerns hypothetical “black bodies”. The earth’s surface is not a perfect black body, by no means, but the land surface absorbs and radiates radiant energy in a fashion that approximates hypothetical black body behavior,i.e., that it radiates according to the radiant energy that it absorbs.

            So study S-B and I hope that helps.

        • mpainter says:

          Norman, you “So if you have a problem with the definition I cant be blamed for you lack of understanding. Maybe it is up to you to update your terms.”

          ###

          I have no such problems. Nor do I have any problem apprehending the significance of the cartoon of the above post.

          Whether that cartoon correctly depicts the radiant flux (energy flow, in the cartoon) is a question aside from what the cartoon intends to show.

  60. Kristian says:

    Norman says, September 7, 2016 at 7:38 AM:

    In my scenario and the rest of the scientific world.

    No, Norman. This is where you’re confusion arises. You read about the “back radiation explanation of the GHE”, and then you refer to some radiation physics texts saying there are two streams of radiation between two objects involved in a radiant heat transfer. From this you then draw the conclusion that THEREFORE the “back radiation explanation of the GHE” is correct and agrees with regular physics.

    When in fact it clearly DOESN’T.

    The difference between those radiation physics texts and the “back radiation explanation of the GHE” is this:

    i) In standard radiative transfer physics, the scientists, even when strictly adhering to the bidirectional transfer principle (“Prevost’s theory of exchanges”), are always careful to ensure that their physical description of the radiative transfer up for discussion complies with thermodynamic laws, based on the simple empirical knowledge that, in reality, the two streams in question are in fact inseparable, they are physically integrated into one ‘net transfer’ at all times. You would NEVER, in standard radiative transfer physics, pretend or suggest that the two streams somehow come with distinct, mutually independent thermodynamic powers. MATHEMATICALLY you are of course free to work with them as individual entities, but you can NOT split them up, place them on either side of the equal sign and expect to gain any PHYSICALLY meaningful insights as a result. Because you know that by doing so, all you would accomplish is utter confusion as to what causes what inside your radiative transfer: Temperature is ALWAYS the cause of a radiance, a radiance is NEVER the cause of a temperature. ‘Thermal radiation’ is caused by temperature, it is not a cause of temperature. Only ‘net transfers’ are, that is, “radiant heat fluxes”.

    ii) The “back radiation explanation of the GHE” violates all of these principles … It is NOT a product of standard physics, Norman. It is an isolate, an aberration. A misconstrued version, an illegitimate child of, standard physics. A product of a blatant MISUSE and/or of a MISINTERPRETATION of standard physics. You cannot and should not disjoin the two streams and place them on either side of the energy budget of a heated object.

    • Kristian says:

      Exemplified in Earth’s surface energy budget:

      i) Standard radiative transfer physics, in compliance with regular thermodynamic laws:

      Net transfer IN (Q_in) = net transfer OUT (Q_out) ->

      Net solar (ASR) = net LW (OLR) + non-radiative losses ->

      165 W/m^2 = [398-345=] 53 W/m^2 + [24+88=] 112 W/m^2 = 165 W/m^2

      ii) “Back radiation explanation of the GHE”:

      Net solar (ASR) + DWLWIR – non-radiative losses = UWLWIR ->

      165 W/m^2 + 345 W/m^2 – 112 W/m^2 = 398 W/m^2

      i) would be the standard physics approach, ii) the homemade approach of “Climate Science^TM” …

    • Tim Folkerts says:

      “Temperature is ALWAYS the cause of a radiance”
      So … if there are two objects with two different temperatures, each will cause a radiance. The two radiances will travel different directions, with different intensities, with different distributions of energies/wavelengths/frequencies. Even when one stream crosses/passes/opposes the other, each stream has its own unique properties determined by the temperature of the object that caused that radiance.

      In other words, by your own logic, the two streams are distinct.

      • Kristian says:

        Tim Folkerts says, September 7, 2016 at 10:33 AM:

        (…) if there are two objects with two different temperatures, each will cause a radiance.

        Yes, and if those two objects are situated in space, those two radiances will, for all intents and purposes, be equal to the radiant HEAT LOSS of each object.

        The two radiances will travel different directions, with different intensities, with different distributions of energies/wavelengths/frequencies.

        Not really. On a microscopic level, individual photons will travel in different directions. On a macroscopic level, the radiance – or rather, the radiant heat flux; the net movement of radiant energy – will travel in the direction of lower radiation intensity and/or energy density.

        Even when one stream crosses/passes/opposes the other, each stream has its own unique properties determined by the temperature of the object that caused that radiance.

        Read the above. This is not what happens. A radiance is only realised as an actual macroscopic forward movement of radiant energy as long as it’s opposed by a lesser radiative potential (intensity/density).

        In other words, by your own logic, the two streams are distinct.

        In other words, you mix up the microscopic (chaotic, quantum) and macroscopic (ordered, thermodynamic) realms.

        Again, I am not saying individual photons do not and cannot fly from cold objects to hot. I’m saying that a radiant macroscopic power density flux does not spontaneously move from a cold place to a hot.

        The energy present and the energy density/intensity distribution within a photon cloud does not IN ITSELF constitute a macroscopic (physically detectable) MOVEMENT (transfer) of energy. Statistically/probabilistically, only the net or average of all photon intensities and path directions through all points in threedimensional space constitute a macroscopic movement, an actual transfer of energy, through the radiation field. Such a movement requires two radiative potentials of unequal strength on either side of the radiation field. One more intense than the other. Hotter than the other. This is what we call a “radiant heat transfer”.

        • Norman says:

          Kristian

          Energy is transferred, heat is not. They are different concepts. You can’t separate photon flux and understand two distinct energy flows and you are unable to separate energy and heat. You are locked in this view and no one will change your view.

          You want others to see it your way but they do not and will not as it is not the correct view.

          You remind me of a boy who was raised by his family to think of oranges as apples. So his whole life, when he saw what others see as an orange he would see an apple. All good an well within the family, they ask for an orange and he gives them what I would call an apple.

          When he gets out to the larger world and everyone else is opposite he fights to try and convince him. Most boys will change to adapt to the rest of the world. Stubborn boys raised on one notion believe the rest are wrong and only their definitions are correct.

          • Kristian says:

            Norman says, September 7, 2016 at 1:22 PM:

            You remind me of a boy (…)

            You remind me of a boy, too, Norman. So let’s leave it at that, shall we? There is no point continuing this utterly futile ‘discussion’ of ours. You will never change. You will never step out of your bubble.

            You think the DWLWIR helps raise the surface temp, but it still doesn’t warm the surface. You also think that by ADDING energy to the surface, you’re actually REDUCING the energy LOSS from the surface, and that by adding extra energy to the surface from the ATMOSPHERE, then the SOLAR input, which is constant and is NOT obstructed from escaping the surface again, is what magically creates further warming.

            A child would see how all this is not only illogical and against all common sense, but also just plain stupid.

            How can you defend such nonsense …!?

          • Norman says:

            Kristian

            Now you have to be pulling my leg! You actually posted this statement? “then the SOLAR input, which is constant and is NOT obstructed from escaping the surface again, is what magically creates further warming.”

            What is not reflected is completely absorbed by the surface, it is not escaping it is no longer there to leave, it went to increasing the kinetic energy of the surface molecules.

            Kristian: “You also think that by ADDING energy to the surface, youre actually REDUCING the energy LOSS from the surface, ”

            Not what I think but what can actually be observed in reality not abstract speculation.

            I sent a few links to mpainter down below. I was hoping you would look at them to demonstrate the direction you are on it a wrong one and try and help you find the correct science path.

            My goal on this blog is to keep the science as honest and pure as possible. I reject a lot of claims by the extremist views of David Appell but yours are just as antiscience as his. More fluffed up by belief than reality.

            Would you ever consider downloading this thermodynamics textbook to your computer and reading some of it when you have time.
            http://web.mit.edu/lienhard/www/ahtt.html

      • Tim Folkerts says:

        Kristian, ultimately the only message I hear in your recent posts is “Yes, I know that individual photons exist, but it is incorrect physics to treat them like they exist”.

        • Kristian says:

          Tim Folkerts says, September 7, 2016 at 1:18 PM:

          Kristian, ultimately the only message I hear in your recent posts is “Yes, I know that individual photons exist, but it is incorrect physics to treat them like they exist”.

          Uhm, you know that’s not what I’m saying, Tim. So stop misrepresenting me. And stop playing stupid. It doesn’t become you. I would much rather see you engage in a proper analysis of the different perspectives on how these processes work.

          After all, they’re all just model descriptions of reality. We are describing the very same process. We SEE the same thing, but we choose to explain and define what we see in different ways.

          I really don’t get the blinkered, hidebound stubbornness of you people … What’s so terrible about admitting that it’s not some perceived “DWLWIR flux” (which is only an apparent radiative temperature EFFECT) from the cool atmosphere to the warm surface that makes the surface even warmer, but rather the TEMPERATURE of that atmosphere being much closer to that of the surface itself than space …?

          • Tim Folkerts says:

            Kristian,

            Frankly I am not sure what you are saying! Yes, it is all “just models” — but you are the one insisting that the more detailed, more complete model can’t possibly be right, while the generic, averaged-out model is the only right perspective.

            Whats so terrible about admitting that its not some perceived DWLWIR flux (which is only an apparent radiative temperature EFFECT) from the cool atmosphere to the warm surface that makes the surface even warmer, but rather the TEMPERATURE of that atmosphere being much closer to that of the surface itself than space ?”

            Several things are wrong. First of all, DWLWIR is not an “apparent effect” but rather a real, measurable energy flow (in the form of IR photons).

            Moving on, there could be lots of different sorts of atmospheres all at the same temperature that would have drastically different effects on the surface temperature. So *temperature* per se is not the deciding factor. You would see dramatically different impacts on surface temperature from 20 C atmospheres of (100% N2) vs (99% N2, 1%CO2) vs (98% N2, 1%CO2, 1% H2O).

            On the other hand, any atmosphere that supplies some specific amount of DWLWIR will have the same impact of surface tempearatures. 200W/m^2 of DWLWIR from pretty much any atmosphere will have basically the same impact of surface temperatures.

            So it is indeed the RADIATION, not the TEMPERATURE that matters!

          • Norman says:

            Tim Folkerts

            After posting comments to both mpainter and Kristian it is so pleasant to read the thought process of someone who actually took advanced science classes. Thanks it is refreshing!

          • mpainter says:

            Hi, Tim, time for a reappraisal of the physics, right? Time to constrain the GCM’s with observations, don’t you think?

            What’s this that I hear about the TOA CO2 spectrum as originating mostly in the stratosphere? Did you know that?

          • Kristian says:

            Tim Folkerts says, September 7, 2016 at 5:14 PM:

            Frankly I am not sure what you are saying! Yes, it is all “just models” but you are the one insisting that the more detailed, more complete model cant possibly be right, while the generic, averaged-out model is the only right perspective.

            Er, what are you trying to say? It is indeed the more detailed, more complete description of the radiation field that says that the ‘net movement’ of radiant energy through a radiation field ISN’T the net of TWO opposite macroscopic movements, like two arrows on a piece of paper, but the average of ALL microscopic movements. The one saying it’s like two arrows, is a simplistic mathematical model only. Are you telling me you don’t know this?

            “What’s so terrible about admitting that it’s not some perceived DWLWIR flux (which is only an apparent radiative temperature EFFECT) from the cool atmosphere to the warm surface that makes the surface even warmer, but rather the TEMPERATURE of that atmosphere being much closer to that of the surface itself than space?”

            Several things are wrong. First of all, DWLWIR is not an “apparent effect” but rather a real, measurable energy flow (in the form of IR photons).

            You know this is incorrect, Tim. You KNOW that the DWLWIR from a cool atmosphere to a warmer surface (“back radiation”) isn’t something we can measure, but something we will have to compute. We measure (as in ‘physically detect’) the temperature of the surface and the ‘net LW’ (the radiant heat flux) up. The two component ‘fluxes’ of a ‘net flux’ are just ASSUMED to be there. Mathematically. But they cannot ever be separately detected/observed. You know this, so why do you pretend you don’t?

            Moving on, there could be lots of different sorts of atmospheres all at the same temperature that would have drastically different effects on the surface temperature. So *temperature* per se is not the deciding factor.

            Quite an assertion there, Tim.

            You know that I’ve pointed out how the temperature isn’t the full story. The temperature GRADIENT is one thing. No, the MASS is the real cause. The air TEMPERATURE is merely one (albeit a very important one) property with which a massive atmosphere affects the surface temperature of a planet. On a planet like Venus, for instance, the bulk air density/pressure gradient from the surface up is such that you need to go tens of kilometres up the atmospheric column before you reach levels of air ‘thin’ enough so as to allow sufficient amounts of energy to escape to space via radiation. Still, it happens at basically the same pressure/density levels as on Earth (although the thick tropopause cloud layer on Venus complicates the direct comparison somewhat). Same on Titan.

            You would see dramatically different impacts on surface temperature from 20 C atmospheres of (100% N2) vs (99% N2, 1%CO2) vs (98% N2, 1%CO2, 1% H2O).

            *Sigh*

            Where is that 100% N2 atmosphere? And where is that 20 C atmosphere containing 99% N2 and 1% CO2 affecting (I guess, warming) the surface temp less than one containing 98% N2, 1% CO2 and 1% H2O? I would think the surface would rather be warmer in the former case, from a higher albedo and greater atmospheric absorp tion of incoming solar (by the H2O) in the latter.

            This is just you ASSUMING stuff, Tim. It’s got nothing to do with the real world. Titan has an atmosphere containing somewhere between 96 and 98% N2. It affects the surface temp in exactly the same manner as the Martian and the Venusian atmosphere do (both containing 95-96% CO2). The things that matter to the differences between their specific surface T_avg are only solar input, lapse rates and atmospheric mass.

            So it is indeed the RADIATION, not the TEMPERATURE that matters!

            No, read this explanation of why it’s temp rather than DWLWIR:
            http://www.drroyspencer.com/2016/08/simple-time-dependent-model-of-the-atmospheric-greenhouse-effect/#comment-223543

            You’re seeing this issue from the wrong perspective. You’re looking at the apparent 345 W/m^2 of DWLWIR and from this point you start out by asking “What if those W/m^2 weren’t there?” But at that point, the warming of the atmosphere has already happened. The DWLWIR is 345 W/m^2 rather than 0 W/m^2 because the relevant atmospheric layers have gained an effective temperature of 279 rather than 2.7 K.

            OF COURSE you would think, just like Norman here, that it is the radiation that matters. But you’re basically seeing a mere EFFECT of temperature as the CAUSE of that temperature.

          • Ball4 says:

            Kristian writes “We measure (as in physically detect) the temperature of the surface”

            Kristian should know even HCN mercury thermometers on the surface compute the air temperature just like a radiometer computes the DW and UW LW. Temperature is not physically detected either from the avg. KE of the molecules; Kristian has a lot to learn.

          • Tim Folkerts says:

            I think we have reached an impasse. Your focus is too narrow and literal.

            * yes, pyrgeometers only measure the net flow of IR, but there are other instrumen out there.
            * yes, a one-way flow model works wonderfully in many settings, but it has limitations.
            *yes, a simple macroscopic model can provide answers to a wide range of thermodynamic problems, but acknowledging atoms and photons and statistical mechanics provides deeper, broader understanding.
            *yes, there is only one atmosphere, but if your science can’t make predictions about new situations, then it is not really science.

          • Ball4 says:

            Kristian writes: “The things that matter to the differences between their specific surface T_avg are only solar input, lapse rates and atmospheric mass.”

            Then Kristian ought to be able to compute Earth T_avg. 288K from only “solar input, lapse rates and atmospheric mass.”

            Show us.

            We already know (energy in – energy out) increases T_avg. to steady state computing to ~288K as shown in top post atm. 0.8 emissivity analysis. Or other Tavg. computed from atm. emissivity (opacity) 0.0 to 1.0.

          • Kristian says:

            Tim Folkerts says, September 8, 2016 at 6:52 AM:

            Your focus is too narrow and literal.

            Who could ask for a more classic example of warmist projection?

            * yes, pyrgeometers only measure the net flow of IR, but there are other instrumen out there.

            Instruments that directly detect each separate ‘component flow’ in a radiant heat transfer? Really? Do tell!

            * yes, a one-way flow model works wonderfully in many settings, but it has limitations.

            Maybe. But at least it has the all-important strength that it never allows a cold object to spontaneously raise directly the U and T of a warmer object by transferring a flux of energy to it.

            *yes, a simple macroscopic model can provide answers to a wide range of thermodynamic problems, but acknowledging atoms and photons and statistical mechanics provides deeper, broader understanding.

            It IS specifically acknowledging atoms and photons and statistical mechanics, Tim. That’s exactly WHY it’s a better and more precise description of reality than the two-way flow model. Try to pay attention.

            If you average out all individual photon movements and intensities throughout a radiation field, you get ONE bulk movement of radiant energy, Tim. NOT two opposing ones. From hot to cold only. The two-way flow model is a mathematical simplification of reality, to make calculations easier.

            *yes, there is only one atmosphere, but if your science cant make predictions about new situations, then it is not really science.

            So, is your idea about atmospheric IR activity setting the surface temperature rather than atmospheric mass able to predict how the tropopause both on Titan, Earth (~0.5-1% IR-active atm) and Venus (97% IR-active atm) is situated around the 100-200 mb level, plus that Mars doesn’t have a “radiative GHE” as defined at all, despite having an atmosphere made up of 95-96% CO2?

            I’m afraid my ‘science’ is more able to predict and explain these observations than yours, Tim …

          • Ball4 says:

            “But at least it has the all-important strength that it never allows a cold object to spontaneously raise directly the U and T of a warmer object by transferring a flux of energy to it.”

            Not never Kristian, that has been demonstrated* possible experimentally in accord with 2LOT as universe entropy increased which is a reason one way fails. Another reason one way also fails is Planck law showing all objects emit/absorb at all frequencies, all temperatures.

            *A colder object can spontaneously make a warmer object warmer still Kristian, as Dr. Spencer’s test shows in previous post. Both the cardboard and ice irradiate the hot plate. “Experiment Results Show a Cool Object Can Make a Warm Object Warmer Still”

          • Norman says:

            Kristian

            Your physics is like D*o-u*g*C*o*t*t*o*n. Not based upon anything but your own opinions and conclusions. Ungrounded thought process that lacks the rigor of science. Make believe.

            You seem like that all you need do is say it is so and if it is.

            Here you post your own flawed opinion. “If you average out all individual photon movements and intensities throughout a radiation field, you get ONE bulk movement of radiant energy, Tim. NOT two opposing ones. From hot to cold only. The two-way flow model is a mathematical simplification of reality, to make calculations easier.

            Two-way flux is not because of a mathematical simplification of reality. It is a logical rational conclusion based upon the atomic theory. If a hot surface is emitting so much radiant energy it is most logical to assume that if its temperature remains the same it will emit the same amount of radiant energy regardless of the surrounding radiant energy sources. No mechanism found for another hot surface to suppress the emission of radiation from a surface. Your ideas are screwy at best and with you would really think about how poorly reasoned they are.

          • I’m still waiting for Kristian or any other self-proclaimed expert in atmospheric thermodynamics to produce a simple time-dependent model that produces anything like the real atmosphere’s vertical temperature profile from an arbitrary temperature starting point. Otherwise, it’s all just hand waving, and I am bored with the usual suspects’ “technical term salads”.

          • Kristian says:

            Roy W. Spencer, Ph. D. says, September 8, 2016 at 3:25 PM:

            I’m still waiting for Kristian or any other self-proclaimed expert in atmospheric thermodynamics to produce a simple time-dependent model that produces anything like the real atmosphere’s vertical temperature profile from an arbitrary temperature starting point.

            Hehe. And I’m waiting for Roy Spencer’s derivation of Earth’s surface T_avg without already knowing it first.

          • Ball4 says:

            Dr. Spencer already did so in the top post spreadsheet with all measured input Kristian. The model doesn’t know the answer to start. It even gets the 255K right which it doesn’t know either to start.

            Now it is Kristian’s turn to do so from: solar input, lapse rates and atmospheric mass showing us his calculation or spreadsheet can produce “anything like the real atmosphere’s vertical temperature profile from an arbitrary temperature starting point.”

          • Tim Folkerts says:

            Kristian says: “The two component fluxes of a net flux are just ASSUMED to be there. Mathematically. But they cannot ever be separately detected/observed. “

            Before derailing us with lots of extra questions, let’s agree on some basics — like “do photons exist?” Your statements (like the one above) clearly show that you consider photons to be merely ‘mathematical assumptions’ that “cannot ever be separately detected/observed”. You are going against all of physics for the past 100 years, so you better have pretty strong support for this position you are taking.

            And until we clear that up, there is really no point even getting into further discussions.

          • Kristian says:

            Tim Folkerts says, September 9, 2016 at 6:46 AM:

            Kristian says: “The two component fluxes of a net flux are just ASSUMED to be there. Mathematically. But they cannot ever be separately detected/observed.”

            Before derailing us with lots of extra questions, let’s agree on some basics like “do photons exist?” Your statements (like the one above) clearly show that you consider photons to be merely ‘mathematical assumptions’ that “cannot ever be separately detected/observed”. You are going against all of physics for the past 100 years, so you better have pretty strong support for this position you are taking.

            And until we clear that up, there is really no point even getting into further discussions.

            You’re not that strong on this microscopic vs. macroscopic realm thing, are you Tim? The inherent disorder of the quantum world and the inherent order of the world of thermodynamics. Chaos vs. consistent patterns.

            This inability (or unwillingness?) to comprehend the simple fact that a ‘photon’ and a ‘radiant power density flux’ are two different things on a fundamental level apparently makes everything I write completely unintelligible to you.

            For instance, you say: “Your statements (like the one above) clearly show that you consider photons to be merely ‘mathematical assumptions’ that “cannot ever be separately detected/observed”.”

            No, Tim. I’m not saying that a ‘photon’ is merely a mathematical assumption that cannot ever be separately detected/observed. I’m POINTING OUT that the notion of two distinct and oppositely directed ‘radiant power density fluxes’ operating inside ONE radiant heat transfer is merely a mathematical assumption (and/or simplification of reality) which cannot ever be empirically verified, because what you always end up detecting, no matter what you do, is the radiant HEAT only, the ‘net radiation’, the statistical/probabilistic – UNIDIRECTIONAL – average of ALL photon movements and intensities inside the entire radiation field.

            I’ve been telling you this now for quite some time, Tim. Why do you continue to shy away from discussing (or even addressing) it?

          • Ball4 says:

            “I’m POINTING OUT that the notion of two distinct and oppositely directed ‘radiant power density fluxes’ operating inside ONE radiant heat transfer is merely a mathematical assumption..”

            Those fluxes are each made up of photons Kristian. Since photons exist both the radiant power density fluxes exist. Tim is correct.

            Those fluxes are not merely mathematical constructions, the net flux (arithmetic) points which way the KE flows in the objects irradiated (an important finding in many designs like car engines), heat flow is a measure of their KE flow. The last Dr. Spencer post experiment should convince Kristian.

            Kristian should also look up an experiment ref.d by Planck and see how this physics was established at the very beginning (Tim’s 100+ years ago). Will be instructive. And, OMG, maybe, hopefully, reduce the length of comment streams.

            Sure, neither photon stream can be separated to do work by 2LOT, I think that is all Kristian is really meaning to convey. The 2 photon streams can be superposed because they are independent which Kristian doesn’t yet grasp.

          • Ball4 says: September 9, 2016 at 11:49 AM

            Kristian:(Im POINTING OUT that the notion of two distinct and oppositely directed radiant power density fluxes operating inside ONE radiant heat transfer is merely a mathematical assumption..)

            And a mathematically impossible assumption at that!!

            “Those fluxes are each made up of photons Kristian. Since photons exist both the radiant power density fluxes exist. Tim is correct.”

            Trick, Some fools call one cycle of a 0.5 micron wave packet a 2 ev photon as that is the Lorentz energy of one cycle of 0.5 micron absorbed flux. How many cycles of such flux before an electron has a 50% probability of emission from Nickel at 20 C? A 1 micron flux has zero probability no-matter how many cycles, so far! What is it that you are calling a photon?

            “Those fluxes are not merely mathematical constructions, the net flux (arithmetic) points which way the KE flows in the objects irradiated (an important finding in many designs like car engines), heat flow is a measure of their KE flow. should convince Kristian.”

            There is only one flux, as per the definition of flux! The last Dr. Spencer post experiment is a clear demonstration of that singular physical process!

            “Kristian should also look up an experiment ref.d by Planck and see how this physics was established at the very beginning (Tims 100+ years ago). Will be instructive. And, OMG, maybe, hopefully, reduce the length of comment streams.”

            In what experiment did Dr Planck ever refer to flux or a physical transfer of power? Planck was very, very careful to only refer to “specific intensity”, which can only be identified a spectral field strength, normalized to the term ‘spectral radiance’! Note the (cm x sr) in the denominator of y axis graphs of all Planck’s equations.

            “The 2 photon streams can be superposed because they are independent which Kristian doesnt yet grasp.”

            Such cannot be superposed as Maxwell’s equations forbid such construct! The generation of flux or the “magnetic flux density” B never physically happens with such combination of (E x H) Poynting vectors at the same frequency! Your religious nonsense is never scientifically creditable!

          • Ball4 says:

            Will, consider the incoherent photons inside an opaque cavity being let out of a small hole as BB radiation field as Planck et.al. did in experiments in a lab, the reflected (and refracted) waves do satisfy the partial differential equations of the electromagnetic field i.e. the Maxwell equations. So you are completely wrong about that.

            “In what experiment did Dr Planck ever refer to flux or a physical transfer of power?”

            The ones he references in “The Theory of Heat Radiation” so Will is wrong about that too. But Will IS right about Planck using cm as “The Theory of Heat Radiation” is written in the CGS system.

            “There is only one flux, as per the definition of flux!”

            A photon is a quantum energy flux, multiple photons, multiple fluxes especially inside the cavity, a bath of photons. One photon flux up and one photon flux down is two fluxes Will, they are independent. Buy a clue!

          • Ball4 says: September 9, 2016 at 7:02 PM

            “Will, consider the incoherent photons inside an opaque cavity being let out of a small hole as BB radiation field as Planck et.al. did in experiments in a lab, the reflected (and refracted) waves do satisfy the partial differential equations of the electromagnetic field i.e. the Maxwell equations. So you are completely wrong about that.”

            Trick,
            I asked but you refuse to identify what you may mean by the word photon. How many cycles of EM flux to emit an electron from room temperature Nickel? At what frequency?

            wj(In what experiment did Dr Planck ever refer to flux or a physical transfer of power?)

            “The ones he references in The Theory of Heat Radiation so Will is wrong about that too. But Will IS right about Planck using cm as The Theory of Heat Radiation is written in the CGS system.”

            References where? The per cm refers to “wavenumber” not to the cgs system. In the same manner the per steradian refers to a static field strength vector, never to any flux crossing the aperture area. If there were no opposing ‘radiance’ the actual exit flux across that aperture per cm would have been PI times that much as the aperture would represent a Lambertian surface and the S-B equation would hold for those wavelengths. Buy a clue!

            wj(There is only one flux, as per the definition of flux!)

            “A photon is a quantum energy flux, multiple photons, multiple fluxes especially inside the cavity, a bath of photons.”

            How many cycles of flux for your imaginary ‘photon’ please?
            You claim some spontaneous flux inside a closed cavity of a single temperature. Please present some viable conjecture as to why such flux shall ever be generated? You seem to believe in perpetual motion.

          • Ball4 says:

            Will: “References where?”

            Buy a clue and get a copy of “The Theory of Heat Radiation” Will, see footnotes p. 199. Will would know this if Will had demonstrated actually reading the whole thing.

            The per cm refers to wavenumber in the CGS system Planck used.

            “How many cycles of flux for your imaginary photon please?”

            Depends on its frequency Will, your photon energy depends on frequency too, h*f, buy a clue Will quote Planck.

            “You claim some spontaneous flux inside a closed cavity of a single temperature. Please present some viable conjecture as to why such flux shall ever be generated?”

            Buy a clue Will, read Planck, quote his words, no conjecture, read the testing in footnotes p. 199 that demonstrated by experiment spontaneous flux inside a closed cavity of a single temperature.

            There is no perpetual motion Will, that is only your religious belief. Buy a clue Will, read Planck.

        • Tim Folkerts says: September 9, 2016 at 6:46 AM

          Kristian says: The two component fluxes of a net flux are just ASSUMED to be there. Mathematically. But they cannot ever be separately detected/observed.

          “Before derailing us with lots of extra questions, lets agree on some basics like do photons exist? Your statements (like the one above) clearly show that you consider photons to be merely mathematical assumptions that cannot ever be separately detected/observed. You are going against all of physics for the past 100 years, so you better have pretty strong support for this position you are taking.”

          Where oh where are your so called detected/observed thermal EM photons being emitted in a direction of higher radiance? You do not have even one viable conjecture as to how such spontaneous emission may occur. It is you Tim that has no scientific support for your nonsense.

          • Ball4 says:

            Will asks, “Where oh where are your so called detected/observed thermal EM photons being emitted in a direction of higher radiance?”

            A. In Dr. Spencer’s experiment, last post.

            If the radiance was from hot plate 1 way, the cardboard would have had no effect on thermometer reading. The data show an effect. Will is completely wrong, buy a copy of “Theory of Heat Radiation” Will and get a clue.

          • Ball4 says: September 9, 2016 at 8:53 PM

            Will asks, Where oh where are your so called detected/observed thermal EM photons being emitted in a direction of higher radiance?

            “A. In Dr. Spencers experiment, last post.”

            Trick, Nowhere is that ever evident!

            “If the radiance was from hot plate 1 way, the cardboard would have had no effect on thermometer reading. The data show an effect. Will is completely wrong, buy a copy of Theory of Heat Radiation Will and get a clue.”

            Again, and again, The hot plate has radiative potential HP, cardboard radiative potential CB, Ice has radiative potential IC. Got that! Not a flux, but just a potential for flux (an EM field strength)!
            The only flux ever spontaneously generated must be proportional to the difference in opposing potentials nothing more, nothing less, and absolutely never bi-directional spontaneous thermal radiative flux.
            HP-IC must have Flux P (power from heat-lamp) must be equal to HP – CD. If CD is higher than IC than with CD in place HP must increase so as the difference remains as Flux P. No opposing flux is ever needed nor is it ever generated…

          • Ball4 says:

            Wrong Will, Dr. Spencer’s thermometer indicated an increase in temperature measuring an increase in hit plate KE not hotplate PE from your “potential” flux off the ice, a very silly notion, photons are EMR per Maxwell’s equations not potentials, buy a clue Will pick up a copy of “The Theory of Heat Radiation”.

          • Ball4 says: September 10, 2016 at 7:54 AM

            Wrong Will, Dr. Spencers thermometer indicated an increase in temperature measuring an increase in hit plate KE not hotplate PE from your potential flux off the ice,

            Trick, Do not say ‘wrong’, point out what you think is even one error pwease!
            Dr. Spencer’s lamps At higher temperature than all else are the only supply of additional KE in the whole demo, the only!The lamps sustain a variable level of KE (sensible heat) in the plate.
            That same plate must dispatch that same ‘extra power’ to achieve a sustainable/stable temperature. The plate can ‘only’ do that; if exists a lower ‘radiance’ surround that has the capability of accepting such EM flux from the energized plate. As the ice ‘radiance’ is replaced by the cardboard ‘radiance’, the plate ‘radiance’, must increase in order to sustain that required difference in thermal EM radiative potential (delta radiance). I.E. the plate temperature must increase to sustain the required difference in potential (radiance). Radiance is never a potential energy, it is a flux (power) potential. Please learn the difference. Please also try to learn the difference twixt potential for flux, and flux itself. Please point out even one technical error in you can!

          • Ball4 says:

            Will, if that is your best effort to admit Dr. Spencer’s experiment and top post show the 2stream physics then not worthwhile pointing out some detail technical errors.

          • Ball4 says: September 10, 2016 at 8:06 PM

            “Will, if that is your best effort to admit Dr. Spencers experiment and top post show the 2stream physics then not worthwhile pointing out some detail technical errors.”

            buy a clue Will pick up a copy of The Theory of Heat Radiation.

            I have so read every word of Max Plancks 1914 novel, including auf Deutsch, with assistance. There is absolutely nothing in that document that is anywhere close to what you claim!

          • Ball4 says: September 10, 2016 at 8:06 PM

            “Will, if that is your best effort to admit Dr. Spencers experiment and top post show the 2stream physics then not worthwhile pointing out some detail technical errors.”

            Complete response “without silly Spencer’s rulez” now at:
            http://joannenova.com.au/2016/09/weekend-unthreaded-133/#comment-1836179

          • Ball4 says:

            Will saya “I have so read every word of Max Plancks 1914 novel..”

            Will, you demonstrate no actual evidence of having done so. And it wasn’t a novel. Quote Dr. Planck’s words as evidence.

          • Ball4 says: September 11, 2016 at 7:36 AM

            Will saya (I have so read every word of Max Plancks 1914 novel..)

            “Will, you demonstrate no actual evidence of having done so. And it wasnt a novel. Quote Dr. Plancks words as evidence.”

            From Planck! “Fuck you Gertrude auf deutsch!”

            Perhaps not a novel as it is does not flow with time. Planck’s 1914 novel clearly depicts the Max lifelong struggle with the concept of “quantum”, (discreet rather than a continuum). Are irrational numbers discreet? In what way? BTW ‘Photons’ are not involved. Such fantasy came way way later!

        • Ball4 says: September 11, 2016 at 10:02 AM

          Will: References where?

          “Buy a clue and get a copy of The Theory of Heat Radiation Will, see footnotes p. 199. Will would know this if Will had demonstrated actually reading the whole thing.”

          The per cm refers to wavenumber in the CGS system Planck used.
          wj (How many cycles of flux for your imaginary photon please?)

          ‘Depends on its frequency Will, your photon energy depends on frequency too, h*f, buy a clue Will quote Planck.’

          Is one cycle a ‘photon’. i.e. one ev for 1 micron EMR? Yes? No?
          Does one cycle of flux constitute your insane version of some fantasy ‘photon’? Yes? No?

          wj (You claim some spontaneous flux inside a closed cavity of a single temperature. Please present some viable conjecture as to why such flux shall ever be generated?)

          “Buy a clue Will, read Planck, quote his words, no conjecture, read the testing in footnotes p. 199 that demonstrated by experiment spontaneous flux inside a closed cavity of a single temperature.”

          There was no “demonstration” only conjecture! Did you even read to page 200 where Planck writes of multiple sources of radiance! You seem only a troll coached by tidbits from goons at SKS! Do you have your viable conjecture of flux inside a closed cavity or not? if you have such what is your version of generated flux. Planck had none!

          “There is no perpetual motion Will, that is only your religious belief. Buy a clue Will, read Planck.”

          Yes no perpetual motion except for your fantasy of imaginary flux.
          Even Planck had great worry!!
          http://onlinelibrary.wiley.com/doi/10.1002/andp.201100712/pdf
          Why your insistence in appearing as a Pfucking Pfool?

          • Ball4 says:

            Will is technically incorrect per Dr. Spencers experiments. There is no perpetual motion Will despite your insistence, buy a clue and get a copy of Plancks The Theory of Heat Radiation to quote from directly.

      • Ball4 says:

        Kristian makes progress here. Just a few days ago Kristian wrote the DWIR photons “flying” were only postulated: “the postulated back radiation flux from the atmosphere”.

        Now Kristian has come around to “individual photons will travel in different directions….I am not saying individual photons do not and cannot fly from cold objects to hot”

        So now the cold atm. object in reality does “fly” photons to the hot surface object. Somewhere Dr. Spencer is smiling. With this comment, Kristian now agrees with all the top post arrows.

        And yes Kristian, net macro KE flows from surface to atm. during the micro photon exchange increasing universe entropy as heat is a measure of constituent KE in an object.

    • Ball4 says:

      Kristian twists yet another pretzel, “In standard radiative transfer physics…radiant heat fluxes”

      Those texts get it right unlike naive Kristian. A specific one, Yves Le Grand wrote a standard radiative energy transfer text “Light, Colour, and Vision”, 1957, John Wiley & Sons, p.4: “…radiant heat is a meaningless term.” and adds, “to say that the sun, for instance, radiates heat is nave”.

      All the texts show you can disjoin (Kristian term) the two streams and place them on either side (or right over each other) of the energy budget of a heated object just like the top post diagram since the incoherent photons do not interact. Find us a text that writes otherwise Kristian. The only blatant misuse, misinterpretation is by Kristian.

      No violation Kristian. Two opposing visible or near IR plane waves (photons if you will) same frequency and amplitude arrive at a plane disk on each side, the total power received by the detector is the sum of the powers of the two waves acting separately. See a good text for the math proof.

      This property of waves (EMR) is not shared by two streams of automobiles. If they cross each other, collisions irreversibly change the automobiles.

  61. Steve Richards says:

    Perhaps I can offer my description.

    1) The earth surface receives solar radiation and warms.
    2) The surface emits IR upwards.
    3) GHGs in the atmosphere partially intercept this radiation stream.
    4) These GHGs increase in temperature slightly.
    5) These GHGs radiate up and down (now slightly more due to its temperature rise.
    6) The lapse rate exists from surface temp reducing down to 2~3K at TOA
    7) The t^4 – t^4 formula shows that the surface is faced with the slightly higher temp of GHG due to its absorbed IR.
    8) If there were no GHGs, the temperature of the atmosphere would be slightly lower which will lower the surface temperature.
    9) More GHGs increase the atmospheric temperature and the surface temp will follow.
    10) The concept of energy flowing down from GHGs of a lower temperature is a nonsense.
    11) This is my current view, potentially modifiable due to experiments coming shortly.

  62. Ball4 says:

    Steve – Is that 1-11 the same as the top post Fig. 1 cartoon?

    • Steve Richards says:

      No. I need to understand what happens to a hotter body when exposed to IR from a cooler body in a vacuum.

      • Ball4 says:

        Steve – the too post arrows intentionally do not show any flux for convection, conduction so represent radiative energy transfer in vacuum conditions.

      • gbaikie says:

        “No. I need to understand what happens to a hotter body when exposed to IR from a cooler body in a vacuum.”

        If the cooler body was an ideal blackbody- nothing happens.
        But we don’t have any ideal blackbodies, which means all known existing bodies will reflect and re-radiate radiation rather than absorb all the radiation. The opposite of perfect or ideal blackbody is something which reflects or re-radiates all radiation which intercept it.
        And parabolic mirror will take light hitting it and reflect as directed light.
        So, suppose you had a perfect parabolic mirror at Earth distance from the sun and the mirror was a large as the sun.
        Go the mid point distance of Earth and the Sun. The reflected sunlight from the perfect parabolic mirror, with provide the same amount of sunlight as the sun from distance of 1.5 earth distance-
        1.5 AU. And of course from that distance the sun is 1/2 the distance of Earth. The suns will be the same apparent size- and reflected sun will same amount of flux as though from 1.5 AU distance- like Mars- about 600 watts per square meters- though the reflected sun will look bigger [much bigger]. If instead of parabolic mirror you used a flat mirror, the apparent size of the sun would just like looking at it from 1.5 AU- though it would be weaker sunlight as compared to parabolic mirror.
        Anyways this mirror would cause the sun to not cool a much, and sun would have to emit more radiate energy. Roughly the sun would expand a bit to do so. Or it seems to me the sun would get bigger rather than hotter- though if bigger essentially is hotter- Earth gets a few more watts of sunlight.

        Or reflecting radiant energy is good way to keep an object cool in sunlight in vacuum. Or acts as insulation- house insulation is
        mostly about insulating again convectional heat loss- which only issue in an atmosphere [or within a pressurized vessel in space].
        Reflecting light is also a way of magnifying sunlight. Or with enough flat mirrors focused on spot one make something as hot as
        the sun.

      • Steve Richards says: September 10, 2016 at 3:36 AM

        “No. I need to understand what happens to a hotter body when exposed to IR from a cooler body in a vacuum”

        That depends on the source of power of your proposed “IR flux”!!
        If that power source ‘creates’ a ‘radiance’ (power potential) greater than that of your “hotter body”, at some or all frequencies, than EM radiant flux may be generated proportional; to that difference in ‘radiance’, with the sign of that difference indicating the direction of the singular flux, at that frequency.
        If only “sensible heat” of mass with flux power potential a monotonic function of temperature is available, than all generated thermal EM flux will be spontaneous, always proportional to the delta ‘radiance’ at each frequency, and in each direction. Such unidirectional spontaneous flux will always be in the direction of lower absolute flux.
        This was the conclusion of 97% of the early 20th century scientists’ really, really, attempting to figure out just what is going on! This is called ‘search’ not re-search, as is the current plagiarism (with attribution). Such conclusion may someday be falsified in the true scientific manner, but nothing so far comes close!
        All what now goes for ‘science’, claims all mass with temperature most radiate, with absolutely no evidence of their religious fantasy.

        • Steve Richards says:

          Ok, thanks for the responses.

          If I were to tighten the question a bit more,

          Two identical resistances, 50mm apart within a vacuum.

          Resistor A has 5V@1A flowing causing it to rise in temperature to 100C.

          Resistor B has slightly less V and A causing to rise to 80C.

          Both resistors off, contraption cools to room temp.

          Resistor A energised and arms to 100c, resistor B is warmed by radiation to 50C.

          Resistor B now energised WITH SUFFICIENT current for resistor B to reach 80C.

          What happens to the temperature of resistor A?

          Also it was mentioned that the degree of blackbody perfection of the two resistors may change the results, can anyone say why?

          • Ball4 says:

            Steve – An easy experiment. Do it. Let us know results like Dr. Spencer did. Past tests have shown no vacuum is needed as you can duplicate results at 1bar.

            “What happens to the temperature of resistor A? Also it was mentioned that the degree of blackbody perfection of the two resistors may change the results, can anyone say why?”

            Consider resistor A shiny as a mirror and then black as coal. Aligned perpendicular then parallel to B. Steady state T depends on both the irradiance T and direction (and polarization) and the incident object composition matter for steady state T. Dr. Spencer showed that in the last post by experiment.

          • gbaikie says:

            Generally, semi conductor conducts electricity better the higher temperature they are [metals are opposite to this].

            “Resistor A energised and arms to 100c, resistor B is warmed by radiation to 50C.”
            So saying resistor B is radiantly heated by A, and not warmed or energised by electrical power like B is?
            And I assume direct current and in a series?

            “Resistor B now energised WITH SUFFICIENT current for resistor B to reach 80C.”
            Now having electrical power which is less only causing same
            Resistor to warmed to 80 C [because it’s in a series with a identical Resistor- I assume]
            “What happens to the temperature of resistor A?”
            It could lower in temperature as higher temperature semi conductor become less resistant when their temperature rises.
            But since metals do the opposite one can design and make it to the opposite.

            “Also it was mentioned that the degree of blackbody perfection of the two resistors may change the results, can anyone say why?”

            Well first you say you in vacuum and you say you have room temperature- leading to a conclusion the one is in a box which has vacuum or a box is a vacuum. In either case the resistors are not really in a vacuum- in terms of radiant energy. But also I suppose that doesn’t matter much.
            In any case the resistors can warm each other, if metals the warmer they get the more power they use, and the opposite if semi conductors- and one usually use metals with wiring/connectors.

  63. phi says:

    With conduction, there are also two opposite flow of energy.
    I suggest to fans of back radiation and forcing that they try to design a boiler with the help of back conduction and isolation forcing. The result could be interesting.

    • phi says:

      There is also something that is often ignored : the use of back radiations and the notion of forcing in climatology are not at all related to some special properties of radiative transfer (that would differentiate them from conduction transfers) but to a strange premise placed on convection. This premise says that convection and radiative phenomena are independent, and that convection alone sets the temperature gradient. The surprising corollary is that convection does not participate in global energy transfer but act in a loop closed by backradiations.

      • Bart says:

        Yes, I think that is the key weakness. You can look at it from the point of the reductio – assume convection is so powerful that temperature equilibrates instantaneously and uniformly throughout the atmosphere. There is no lapse rate in that case, and the surface temperature reduces to the same as in the non-atmosphere case.

        So, there has to be a point of diminishing returns, and an inflection that reverses the sign of GHE impact at some point. Given the propensity of natural systems to strive toward their physically imposed limits, I suspect we are at or very near that inflection point, and are effectively insensitive to further GHG forcing in the present climate state.

      • Joel Shore says:

        phi: There is nothing particularly strange about it. It is simply an assumption that the timescales for convection are so much faster than radiation that they can be essentially considered as instantaneous. These sorts of separations of timescales arguments are used throughout physics and are not particularly controversial.

        Bart: I don’t understand your point. It makes no sense to assume that convection does something that it simply doesn’t do. The whole point about convection is it is a very fast and efficient process when it operates but it can only drive the lapse rate down to the adiabatic lapse rate and no further because an atmosphere with a less steep lapse rate is not unstable to convection.

        I agree that if we lived in a universe where the atmosphere was unstable to convection down to an isothermal state then it would completely wipe out the radiative greenhouse effect. But, that’s not the Universe we live in.

        • alphagruis says:

          These sorts of separations of timescales arguments are used throughout physics and are not particularly controversial.

          Yes of course, as well as related lengthscales. For instance it is what one does when one theorizes in a simple thermal conduction experiment. There is a temperature gradient across the sample and one makes the (usually valid) assumption that there is a well defined temperature locally but not globally i.e. collisions establish local but not global thermodynamic equilibrium.
          Similarly in the so-called adiabatic approximation in molecular or solid state physics one assumes that electrons move so much faster than nuclei or ions that one may solve the Schroedinger equation for a given atomic configuration and use the result as a function of configuration as an input for the calculation of vibration modes.

        • Bart says:

          “The whole point about convection is it is a very fast and efficient process when it operates but it can only drive the lapse rate down to the adiabatic lapse rate and no further because an atmosphere with a less steep lapse rate is not unstable to convection.”

          Again, the lapse rate is not the whole story. The formula only gives you a rate, not absolute temperature, and it is only valid up to the ERL.

      • alphagruis says:

        First, radiation propagation in atmosphere is not by a mechanism that compares to thermal conduction where one may safely assume local thermodynamic equilibrium. Thats not the case with radiation transfer here, radiation in not in local equilibrium with air except perhaps in lower troposphere and center of the CO2 15 micrometer line. So there is nothing like a local Fourier law in radiation transfer in atmosphere as in stellar physics for instance, where the photon mean free path is short enough to ensure local thermodynamic equilibrium.
        Second convection and radiation transfer are not independent since deep convection is precisely triggered by GHG hindered radiation transfer. Thats why the folks studying these things use the concept of radiative-convective equilibrium.
        http://irina.eas.gatech.edu/ATOC5560_2002/Lec26.pdf
        And that convection sets the vertical gradient is a simple consequence of the way it works, as pointed out by Joel Shore. Nothing strange or surprising, the wet adiabatic lapse rate being an attractor of the system that is a system far from thermodynamic equilibrium and as such exhibits its own organization principles, setting lapse rate, forming trade winds, hurricanes, El Ninos etc.
        Finally wet convection does of course not only participate but dominates in global heat transfer to the top of the atmosphere.

        • phi says:

          “Finally wet convection does of course not only participate but dominates in global heat transfer to the top of the atmosphere.”

          Not with the dominant theory. That’s what lies beneath her skirts:
          http://oi61.tinypic.com/e6u2pk.jpg

          • alphagruis says:

            No, that is just the result of the confusion that lies beneath your skirt not dominant theory.

            Moreover please don’t label your “diagram” as the one of Trenberth. Trenberth in his diagram just shows energy fluxes at interfaces and of course no fancy loops that are plain nonsense. Maybe you might think very hard about this, finally.

          • phi says:

            alphagruis,
            This is indeed the dominant theory and I challenge you to prove otherwise.

            “Trenberth in his diagram just shows energy fluxes at interfaces…”

            This is a poor choice because indicated only for net flows assimilable to heat flow. Energy flows can not drown in the earth except in the form of heat. If it is purely energy flows, you have to complete them.

            Well, I do not see more than the flabby notion of a moist adiabatic attractor which affects only the pronounced low pressure area.

          • alphagruis says:

            This is indeed the dominant theory and I challenge you to prove otherwise.

            No , it isn’t.

            And it’s up to you to prove that your nonsense loops have anything to do with the so-called “dominant theory” and Trenberth diagram. For instance you must definitely not understand properly what convection means and does to associate a loop with relevant heat transfer.

            Actually there are no literate physicists who disagree with what you call “dominant theory” and there is nothing like an “alternative theory” or precisely an alternative explanation of the atmospheric greenhouse effect. “Theory” by the way is much too overblown in context, we are not at all talking about something like competing quantum gravity and string theories…

          • phi says:

            alphagruis,

            “…your nonsense loops…”

            But they are not mine !!!

            If you stick to integrate back radiation you need to recycle them and you can not do otherwise than with loops. Nobody denies this. We can discuss the distribution of flows (my schema minimizes the loop of the greenhouse effect) but not deny the logical consequences of a poor concept.

          • alphagruis says:

            But they are not mine !!!

            Of course they are.

            Nothing to recycle, nothing to deny, just your poor understanding of basic concepts in physics.

            A simple suggestion: Consider reading a good textbook that explains from scratch how two opposite photon fluxes are involved in reaching equilibrium between thermal radiation in a box held at temperature T and the walls of that box i.e. the very basic and fundamental concept of thermal and blackbody radiation.

            Feynman lectures are a good start: http://www.feynmanlectures.caltech.edu/I_41.html

          • phi says:

            alphagruis,
            I wonder where are going back radiation if they are not involved in a loop.
            A prerequisite to scientific thinking is logic.
            Bon vent.

          • alphagruis says:

            Bon vent

            Merci, j’en aurai bientt besoin au sens propre du terme.

          • alphagruis says: September 9, 2016 at 1:26 AM

            “A simple suggestion: Consider reading a good textbook that explains from scratch how two opposite photon fluxes are involved in reaching equilibrium between thermal radiation in a box held at temperature T and the walls of that box i.e. the very basic and fundamental concept of thermal and blackbody radiation.”

            You claim some ‘opposing photon flux’. Such is a physical impossible! See: http://www.drroyspencer.com/2016/08/simple-time-dependent-model-of-the-atmospheric-greenhouse-effect/#comment-224455

            “Feynman lectures are a good start: http://www.feynmanlectures.caltech.edu/I_41.html

            Now you claim that some lecture on “Brownian motion” somehow relates to the generation of an EM Poynting flux! Why?

  64. Norman says:

    mpainter

    This will not help you understand my points. It is a waste of time on my part. You might click on the links but you will not attempt to understand the message they are telling you.

    It is okay though, I learn more by it. I wish Kristian would at least look at the data and consider it. Dream big I guess.

    http://www.esrl.noaa.gov/gmd/webdata/tmp/surfrad_57d099bdbd3bb.png

    This graph is just of all the radiant fluxes together and the air temperature. You can see there are numerous radiant inputs. Each is significant in determining surface temp.

  65. Norman says:

    mpainter

    http://www.esrl.noaa.gov/gmd/webdata/tmp/surfrad_57d09a6ad5c8b.png

    This is all the fluxes combined vs temperature. You can see when the NET flux is negative surface temperatures drop (cooling). When NET flux is positive you an see surface temperatures rise (warming).

  66. Norman says:

    mpainter

    http://www.esrl.noaa.gov/gmd/webdata/tmp/surfrad_57d09b1c7f7f0.png

    Just surface IR emission. I give these to show you actual numbers.

  67. Norman says:

    mpainter

    http://www.esrl.noaa.gov/gmd/webdata/tmp/surfrad_57d09bc94fe3b.png

    Combined Surface emission minus backradiation. It is negative the entire 24 hour cycle.

  68. Norman says:

    mpainter

    http://www.esrl.noaa.gov/gmd/webdata/tmp/surfrad_57d0a151a409f.png

    Solar net. The downwelling solar minus the reflected solar that is not absorbed.

  69. Norman says:

    mpainter

    Now the big one. The one that totally supports what I have spent many words trying frivolously to explain to you. If you look at the links you may understand (Dream big, hope for more).

    http://www.esrl.noaa.gov/gmd/webdata/tmp/surfrad_57d0a5f24d0ed.png

    Once again the backradiation. If you look at the number value during the day it is around 335 W/m^2.

    http://www.esrl.noaa.gov/gmd/webdata/tmp/surfrad_57d0a202f36ea.png

    This link is of the Total NET radiation. It is all the fluxes with backradiation included. Note it reaches around 440 W/m^2 or so during the middle of day.

    Dream big! What does this mean? Remove the 335 W/m^2 from the Daytime total NET flux and you have around 105 W/m^2 heating the surface instead of 440 W/m^2.

    All the graphs clearly and without doubt show backradiation alone does not warm the surface but actually is too little of a flux and the surface cools. This final post shows beyond reasonable doubt that it is the combination of the two fluxes that achieves the higher equilibrium temperature of the Earth’s surface proving that the GHE is real, valid physics, does not violate the 2nd Law of Thermodynamics and is currently working in the real world to maintain higher equilibrium surface temperature.

  70. Norman says:

    mpainter

    http://www.esrl.noaa.gov/gmd/webdata/tmp/surfrad_57d0a7cea15bb.png

    This one clearly and most positively shows that backradiation is not warming the surface even though the surface is absorbing it.

    Look at the temperature part of the graph during the night with no solar flux involved and explain to me why you think backradiation is warming the surface when the evidence shows it does not.

    The net radiation loss is negative. How does this warm the surface?

    • Kristian says:

      Norman says, September 7, 2016 at 5:53 PM:

      The net radiation loss is negative. How does this warm the surface?

      *Sigh*

      As has been explained to you now probably several tens of times, like this:
      https://okulaer.files.wordpress.com/2014/10/drivhuseffekten.png

      This EXPLANATION of the effect is physically nonsensical, Norman. NOT the effect itself. The “back radiation” EXPLANATION of it. According to it, there is no other energy flow that can raise the surface temp from 232 to 289 K but the DWLWIR. That makes the DWLWIR a radiant heat flux number two, right next to the solar flux.

      • Norman says:

        Kristian

        I have also countered your explanations many times and provide real world data. You escape from it is then to claim the instruments are not measuring a real downwelling flux of radiant energy. I think your escape is a bogus one. You come up with your own ideas and when challenged with facts have to claim the facts are wrong.

        The FACTS (which you can’t accept) the measured reality, the same device that can accurately determine temperatures in labs of materials during calibration (Both Roy and Ball4 have sent you articles on these instruments which you just conveniently ignore because it does not fit into your beliefs…rather than examine the flaws in your beliefs you reject the science of instrument measured values and just go right on ahead with your unsupported claims).

        One thing I have in my favor is measured evidence. So far you have zero support of any of your conclusions. I sent you a link to a energy transfer textbook to read which I doubt you ever will.

        The measured values show DWLWIR does not alone heat the surface. The surface cools with this flux alone. It is a real flux created by a warm atmosphere that generates IR via GHG molecules. It can’t warm the surface because it is less than the UWLWIR. The graphs to measured values clearly show all what I post.

        Your only escape is to reject the evidence that supports my claim and shows yours to be wrong. So provide evidence for your claims that DWLWIR is a radiant heat flux. The graph you provide does not suggest this at all!!

        THE UWIR in your graph is 398!! The DWIR is 345! The number 345 is less than 398! IT is not adding heat to the surface!! It does warm the surface. It adds energy, not heat! Your thought process has flaws that are not fixable by rational thought.

        • Norman says:

          Kristian

          Roy Spencer spent time and effort on the previous thread to demonstrate to people of your thought process how backradiation can change equilibrium temperatures. You still reject all evidence presented to you. Real world measured values (you have to reject the instruments used to measure this and won’t accept any form of measurement). Roy does an experiment which blows your assertions out of the water but it has no effect you just keep going straight ahead.

          I do not know what science you follow. Reject all empirical data that counters what you strongly believe and call the reality nonsense as if that somehow will make your claims stronger.

          I have already explained how solar influx can raise the temperature if the DWIR acts as a radiant reduction from UWIR you don’t like that explanation even though it is logical.

          As Roy has stated numerous times. It is not the energy going in that determines a surface temperature, it is the energy going in and out that determine this.

          A light bulb filament at only 10 watts power will achieve a surface temperature over 2000 C.

          http://physics.stackexchange.com/questions/176918/how-to-calculate-temperature-of-an-incandescent-bulb-filament

          The very small surface area of the filament does not let out enough energy at room temp so the temperature keeps rising until the the surface can emit 10 watts which is at temps over 2000 C

          Real world fully supports the GHE, data fits, Roy’s experiments prove it. And then you think I am wrong?

          • Norman says:

            mpainter

            You: “Input does not sum with output. It sums with residual heat. And that is the laws of physics. DWLWIR and upwelling IR are not vectors, you cannot sum them as vectors. Net is screwball AGW science.”

            Which laws of physics would that be? And Net is screwball.

            This is your opinion of course as you have not actually looked at the physics but make the claim it is physics.

            For a little while I was swayed by the PSI group and this thought process and considered. After reading Roy’s Blog and learning some physics I reject all the crap being peddled on this blog and want to restore science.

            I may not change you but when you and Kristian peddle false unverified opinions as if they were real physics and you both were in the “know” and everyone else is on the “take”. It appeals to uneducated Conspiracy people who do not research or investigate on their own.

            Here is real physics. This will show up in many textbooks, it is well understood and accepted and used throughout heat transfer engineering.

            REAL PHYSICS (not peddled opinions):
            http://web.mit.edu/16.unified/www/FALL/thermodynamics/notes/node137.html

        • mpainter says:

          Net is theoretical construct, not observed.
          Observed values are the measured values.
          Input does not sum with output. It sums with residual heat. And that is the laws of physics. DWLWIR and upwelling IR are not vectors, you cannot sum them as vectors. “Net” is screwball AGW science.

          Repeat, DWLWIR sums with residual heat of insolation. Upwelling results, the measured value, not the theoretical net.

          • Ball4 says:

            “DWLWIR and upwelling IR are not vectors, you cannot sum them as vectors.”

            The radiometer views the whole hemisphere above it mp, so the photons measured come from all vectors above. The red arrows in top post should be cones as scientists in the field know, well except for mpainter. And science (1LOT) does add ALL the energy in minus ALL the energy out of a control volume to get the net change in energy inside – just as in the spread sheet top post and all experiments to date confirm 1LOT.

      • Ball4 says:

        “That makes the DWLWIR a radiant heat flux number two, right next to the solar flux.”

        That really is a net negative (KE & radiant) energy flux (in Kristian term’s a bizarre negative heat flux what ever that means), since the LW DW-UW net is negative as shown in the ESRL plots. KE flows from surface to atm. Kristian not the other away around as you write. And your chart bizarrely neglects a yellow arrow for the solar SW atm. absorbed in the 345 LW atm. emitted.

        Kristian has a lot of science to learn and improve. This should be taken as an opportunity for Kristian not an adversity.

    • mpainter says:

      Norman, only the upwelling IR is obtained by measurement. The “net” is this measurement less the DWLWIR. “Net” is a theoretical construct.
      The reality is the measured upwelling IR.

      Upwelling IR depends on Tsurf, Tsurf depends on DWLWIR (plus residual heat of insolation)

      See if you can figure out the rest, Norman. I hereby resign.

      • Norman says:

        mpainter

        Net is two measured values added together not a theoretical construct. All the fluxes are measured values. The Nets are just simple math on the measured values.

        One instrument is facing downward measuring UWIR the other faces up and will measure the DWIR. The addition of these two measurements is the net.

      • mpainter says:

        Overriding principle:
        Radiant energy is absorbed as kinetic energy. Let’s see how far you get with overriding principles, Norman, or if you cling to your pseudo-vector mathematics.

        • Norman says:

          mpainter

          I will stick with the empirical evidence and the textbook explanations of heat transfer. I really do not know from what background you come from with your points. Do you have higher level physics studies or does most your information come from the Internet and your own opinions?

          There are some who actually took thermodynamics courses at University levels and post here and they do post ideas like what I post. Textbooks also go along with this reasoning.

          I have not seen the word “pseudo” used since g*e*r*a*n was banned by Roy.

          It is not clinging to anything. I like science integrity. When people go off and try to use their own reasoning then act as if it is the reality without grounding themselves back into a textbook on the topic, I feel the need to comment.

        • mpainter says:

          Norman, the empirical evidence is the measurement, not the pseudo-vector mathematics. You are thoroughly confused.

          My textbook says that “radiation absorbed by the surface” is absorbed as kinetic energy. What does yours say?

          And does your textbook treat isotropic radiative flux as vector sums?

          • Norman says:

            mpainter

            Here is a book for you to read.

            https://books.google.com/books?id=0dqMyFOUqxoC&pg=PA45&lpg=PA45&dq=radiation+absorbed+at+surface+becomes+kinetic+energy&source=bl&ots=z3iV9-s4Qc&sig=x_g_PXml_XbXZAuE1V-5De9GFhQ&hl=en&sa=X&ved=0ahUKEwiz8OWCi4HPAhVFbR4KHWxMDrcQ6AEILDAC#v=onepage&q=radiation%20absorbed%20at%20surface%20becomes%20kinetic%20energy&f=false

            This one has view factors for radiation and explains it quite well.
            The Earth/Atmosphere act as a sphere inside a sphere which has a view factor of 1. All the Earth’s radiation will hit the atmopshere.

            http://www.kostic.niu.edu/352/_352-posted/Heat_4e_Chap13-Radiation_HT_lecture-PDF.pdf

            Also shows how NET radiation works, since you do not know how this process works it would help you to read through it for greater understanding.

            1st law of thermodynamics would have to treat isotropic radiative flux as vector sums when the field of view is 1. All energy has to be accounted for.

            I think your physics book was written by Kristian. Do you have a link to this magic physics book that provides you with such informed knowledge that all other physics professionals are wrong and don’t even know it.

            I have linked you to many sources of physics information you just give me a single line quote.

          • mpainter says:

            Norman, I assume that you take the position that radiation absorbed by the surface is NOT absorbed as kinetic energy.

            If you do not take that position, you need to say so. Because others are reading this. You see.

          • Norman says:

            mpainter

            I already answered this twice now. Why do you keep bringing the same point up over and over?

            But because you ask here is the third post of what I have already said. Maybe this time you might read it or not.

            Again: “ME ABOVE ON THIS SAME THREAD: A surface molecule absorbs a DWIR and starts to jiggle more which is picked up by the surrounding molecules and it would seem that the overall kinetic energy would go up so why doesnt it? That is because for this one molecule that absorbed an IR photon and increased its local kinetic energy there were two molecules elsewhere on the surface that emitting photons (same time) and lost more kinetic energy than the surface gained by the one absorbed photon. With this process going on trillions plus times per second you have a net loss of energy to the cooler atmosphere.

          • Norman says:

            mpainter

            NOTE: Rather than spending so much time posting maybe you should look at the links to thermodynamics I posted and read the material and learn what the science is saying. All your questions are contained within.

            I can’t help it you have some really poor ideas about how radiation works. Not sure where you got these ideas. Was it this web site?

            https://okulaer.wordpress.com/

            It has lots of information and maybe it convinced you. Now I would hope you go to some real science sources and spend time reading and learning the material.

          • mpainter says:

            Ok, Norman, I think that I understand now: the photon jiggles the absorbing molecule a little bit, but not enough to warm it. How good of you to take the time to explain thanxgoodbye.

          • Norman says:

            mpainter

            I think at least I have found out why you do not understand my point.

            I am not sure you understand what a temperature of an object is or a surface.

            It is the average kinetic energy of the surface or object.

            So in your mind if you add kinetic energy to one part of a surface that means the average kinetic energy of the entire surface is going up.

            Since you don’t consider the temperature to be average kinetic energy it makes explaining things nearly impossible.

            You might read up on it (since it is average kinetic energy) that some molecules on the surface are vibrating much more rapidly in their fixed positions than other surface molecules and this whole process moves around the surface continuously. That is why a surface emits a spectrum of radiation and not just at one wavelength. The surface has many different molecules with many different states of kinetic energy. I hope you read more physics in the future so you can understand these things and posters do not have to waste time trying to explain them to you.

          • Norman says:

            mpainter

            Science and technology continue to move forward. The are developing Quantum Thermometers to see variations of heat in objects that normal thermometers would not.

            If you look at the image of cell you can see that it has temperature variations but a standard thermometer would only give you the average temperature of a cell or many cells.

            https://physics.aps.org/articles/v8/52

            At least this may help you understand that surfaces are not one uniform layer of kinetic energy so adding energy to one part automatically adds kinetic energy to the whole surface.

            If you have competing processes where one is adding some energy and the other is removing energy, the only way you could tell if the kinetic energy, overall, is going up or down is to know the combination of the processes.

            If more energy is added to the surface than removed the kinetic energy of the whole surface goes up (but you will still colder and warmer regions on that surface…it is the overall that a thermometer will indicate).

            Hope all this helps in your quest for Truth. The “Truth is out there” we just have to be open and willing to look for it.

          • gbaikie says:

            — Norman says:
            September 9, 2016 at 4:49 AM

            mpainter

            I think at least I have found out why you do not understand my point.

            I am not sure you understand what a temperature of an object is or a surface.–

            It depends what you talking about.

            An object cooling can have cooler surface. An object being warmed
            can have a warmer surface.
            A surface can have specific meaning- ie, gases don’t have a surface.
            Only solids and liquids and plamsa [anything with structural connection] can have a surface.
            But it’s common for people to refer to the surface temperature
            as the air temperature close to the surface.
            Or air does not have or is a surface, surface air temperature is
            air near the surface. Or a store at the corner, can be called the corner store.

            The ocean is warmed by sunlight below it’s surface. The ocean is warmest at the surface because warmer water is less dense, and rises to the surface.
            In comparison a land surface is warmed at it’s surface, and heated surface conducts heat below the surface. The higher the temperature of the land surface in relation to interior [below the ground/surface] the more heat can be conducted in a time period.
            The heated land surface also heats the air above it, and likewise
            the hotter the surface the more heat can transferred to the air.

            With ocean a higher surface temperature results in more evaporation. Evaporation is adding gases to atmosphere and water vapor has lower density as compared to air. This results in surface air temperature being close to the temperature of the surface of the ocean, whereas with land surface there can be quite
            large differences between the surface of the land and the air near the surface.

          • gbaikie says:

            I would add that there is two major surfaces on Earth, the land surface and the ocean surface.
            In terms of global average surface air temperature one can ignore the temperature of the land surface.
            Or the ocean surface temperature dominantly controls the global average air temperature.

            So to model Earth one should understand what the temperature of a planet completely covered with oceans would be.
            And at Earth distance such planet completely cover with ocean and has 1 atm [of any gas] will have higher average global temperature
            than Earth- or Earth’s land area generally lower the global average air temperature. There few reasons- not one reason for this- but one reason is air temperature above ocean is the same temperature as ocean surface temperature.

          • mpainter says:

            Norman, study Stefan-Boltzmann, maybe that will help you.

          • Norman says:

            mpainter

            Maybe if you would be a little more specific in exactly what point you would like me to study. I would be more than happy to learn but not sure what direction you think I need help in, clarification certainly would not hurt. Thanks.

  71. Tattare says:

    Solar radiation of 1368W passes down the atmosphere. At the surface a fraction remains. The amount is found by dividing it by the volume of a sphere, 4/3pi*r^3, and we get what energy a cubic meter of air contains when irradiated.
    In the same way that TSI is divided by four to get effective temperature, we divide by 1.333333…. to get energy-content of a spherical volume with pi*r cancelled out.
    =1026W/m^3

    Now we let this continue to transfer through the surface to see what a cubic meter of solid surface matter receives.
    =769,5W/m^3.

    Now we know how much energy that heats up the massive surface. And we get a familiar number there.

    To get the fluxdensity emitted by the surface we need to divide it into 2m^2 of emitted energy, since the energy of 1 heated m^2 is emitted by two m^2.

    Holy crap. That gives 384W/m^2.

    There is no energy missing in solar radiation to heat up the surface. We can stop claiming that icecold gas heats the surface, we can use only the hot sun. Phew!

    Everything is back to normal again. No icecold gases heating warm surfaces. That is good, because the greenhouse effect was the only thing that claimed such bad physics.

    I think my simple model is simpler than yours, and it get the right answer in a straightforward manner without rubberball-radiation that contradicts maxwell-boltzmann distribution. The m-b distribution tells us that temperature is a result of density of energystates at different levels. Radiation from a cold gas to a warm surface would dilute the higher states of excitation in the warm surface, the effect would be lower temperature.

    More co2 means more dilution. If we want to account for atmospheric radiation. It-is-not-about-the-number-of-photons.

  72. Tattare says:

    Another point. Everyone is aware that the stefan-boltzmann law is a form of E=m*c^2?

    E is P and m is A. P/A=σT^4

    σT^4 is the energy equal to the photons c^2.

    To add an atmosphere or small amounts of co2 is equal to increasing m or A.

    See what happens when you add mass or Area to the equation. If the energy increases, as the warmist puts its faith in, the speed of light must increase.

    Einstein simplified P/A=σT^4 into E=m*c^2 to get the energy in the smallest quantities. But they both address the same thing, energy in relation to matter.

    Your claim is that an atmosphere or small fractions of it, when added to the solid surface or fractions increase, will increase the energy in the system by increasing the mass. When energy is divided over an increased mass and that energy increases, Einstein starts turning over in his grave.

    Very soon he is spinning at a high speed and you should dig him up and connect a generator to his corpse, then you will get even more free energy added to that you claim comes from icecold gas.

    Please dont say that increasing the amount of icecold porous matter will increase the density of energy in an open system in contact with the ultimate heatsink of 3K space. It will be very embarrassing in the future to have made such claims.

  73. Norman says:

    mpainter or Kristian

    How do either of you explain the results of Roy Spencer’s experiment on the previous post?

    Why does putting cardboard on top of ice result in the surface temperature of the heated plate from warming up and reaching new equilibrium temperatures?

    His backradiation explanation works perfectly to explain it. Wonder what yours are.

  74. Norman says:

    mpainter and Kristian

    Why do you think that a 10 watt power supply can increase one surface to over 2000 C and another -158 C?

    The first is a tungsten filament in a light bulb. The other is a one meter square surface.

    Why does surface area matter so much?

    It is what Kristian can’t accept. Slow the rate at which radiant energy leaves and the temperature rises. It is not just the incoming that matters but also the outgoing.

    Radiation emission is a surface phenomena. Decrease the area with the same input energy and the temperature must rise until it emits the same amount of energy it receives. For an 8 mm^2 tungsten filament surface receiving 10 watts the temperature must rise to over 2000 C to get rid of 10 watts through the very small surface area.

    Backradiation overall effect is to reduce the NET radiation emitted by the surface with the same incoming energy. The result is a temperature rise just as reducing surface area also increases surface temperature under a constant energy input, the joules just keep piling up until they can escape through high temperatures.

    • Kristian says:

      In all this time we have come no further, Norman.

      You say, September 8, 2016 at 9:29 PM:

      mpainter and Kristian

      Why do you think that a 10 watt power supply can increase one surface to over 2000 C and another -158 C?

      The first is a tungsten filament in a light bulb. The other is a one meter square surface.

      Why does surface area matter so much?

      It is what Kristian can’t accept. Slow the rate at which radiant energy leaves and the temperature rises.

      (My boldface.)

      You STILL think I have this mental block preventing me from understanding your position, Norman. I don’t. I understand your position perfectly well. That’s not the problem here. All this time you have done nothing but building straw men to tear down. And you’re doing it again here now.

      The mental block is all yours. At least try to see how there’s no logic in what you’re saying. You’re indeed describing a very real effect, a real phenomenon, Norman. We all agree on that. It’s called “insulation”.

      All I’m trying to tell you is this: YOU’RE DESCRIBING IT IN A WAY THAT MAKES ABSOLUTELY NO PHYSICAL OR LOGICAL SENSE.

      Simple analogy:
      We have a room that at any point in time holds one hundred people. There’s an open door at each end of the room. Every minute, ten new people arrive in through the first door, but at the same time, ten people also leave the room through the second door, making sure we maintain a dynamic balance between input and output of people, and thus always keep the number of people inside the room the same: 100.

      Then, at some point, we start letting two people per minute enter the room through the second door as well. What happens? Well, there are still ten people entering the room through the first door every minute, and there are also still ten people exiting the room through the second door during the same period. There are no restrictions imposed saying the ten people leaving each minute now all of a sudden aren’t allowed to. No, the only change is that now two extra people are entering through the same door per minute as the ten are leaving.

      We all know what will happen to the number of people inside the room in this situation. It will increase.

      But what CAUSES the INCREASE? I’m not talking about the dynamic balance between input and output that caused the original number. I’m only talking about what causes this number, this baseline, to start INCREASE.

      Both before and after the ‘change’, ten people per minute freely enter the room through the first door, and both before and after the ‘change’, ten people per minute exit the room through the first door.

      What causes the number of people inside the room to increase, Norman?

      I quote you verbatim: “Slow the rate at which radiant energy leaves and the temperature rises.”

      You are NOT slowing the rate at which radiant energy leaves the surface, Norman. NOT in how you specifically describe the process behind the temperature rise. You are directly speeding up the overall rate at which radiant energy enters and is absorbed by the surface. By adding the DWLWIR to the solar input. And that’s how you get extra warming. That’s how you increase the number of people inside the room.

      Again, we describe the very same effect, the very same physical phenomenon, and we even get the same end result – reduce Q_out while keeping Q_in unchanged, and you get warming.

      The bone of contention is simply: How is Q_out reduced?

      I say: By increasing the temperature of the heated object’s thermal surroundings, in our case, the atmosphere. THEREBY increasing the apparent radiance of the atmosphere, the DWLWIR, reducing the ‘net LW’ (“the radiant heat loss”) AND the other heat losses from the surface. (There are also other ‘massive’ effects reducing surface heat loss, like surface air pressure and the pressure/density gradient of the atmospheric column.)

      You say: By increasing the DWLWIR.

      • Tim Folkerts says:

        Kristian, perhaps you are not doing a good job of explaining your position. What I hear you saying in your analogy is NOT what I hear you saying other places.

        Specifically, by saying the “two flow model” is only ‘mathematical’, you are sending the message that you can’t have 2 people coming in the back door to get a net flow 8 out. Instead, the message you have communicated is that the reality is that 8 are leaving and none are returning. You have communicated that “10 leaving and 2 returning” is a false model that cannot actually exist.

        So the debate as I been hearing it is (using your analogy)
        NORMAN: 10 leave, but 2 others come in (two flows)
        KRISTIAN: only 8 leave (one flow)

        • Tim Folkerts says:

          And further, after a new steady-state is achieved, debate as I been hearing it is (using your analogy)
          NORMAN: 12 leave, but 2 others come in (two flows)
          KRISTIAN: 10 leave (one flow)

          • Ball4 says:
            September 10, 2016 at 5:18 AM

            Will says (That is the only flux, with the sign indicating direction!)

            “No Will, you are proven completely wrong by Dr. Spencers experiment in which the cardboard had a proven effect in the data. Both the cardboard and ice irradiated the hot plate with photons. If what you write were true inserting the cardboard would have had no effect. Buy a clue Will and pick up a copy of The Theory of Heat Radiation and find out exactly why you are completely wrong.”

            Dr. Spencer starts with the ungodly fantasy that all mass with temperature must radiate EMR. Nothing can be further from the physical near this planet Earth.
            Trick, Do not say wrong, point out what you think is even one error pwease!
            Dr. Spencers lamps At higher temperature than all else are the only supply of additional KE in the whole demo, the only!The lamps sustain a variable level of KE (sensible heat) in the plate.
            That same plate must dispatch that same extra power to achieve a sustainable/stable temperature. The plate can only do that; if exists a lower radiance surround that has the capability of accepting such EM flux from the energized plate. As the ice radiance is replaced by the cardboard radiance, the plate radiance, must increase in order to sustain that required difference in thermal EM radiative potential (delta radiance). I.E. the plate temperature must increase to sustain the required difference in potential (radiance). Radiance is never a potential energy, it is a flux (power) potential. Please learn the difference. Please also try to learn the difference twixt potential for flux, and flux itself. Please point out even one technical error in you can!

          • Kristian says: September 10, 2016 at 8:12 PM

            “Youre not the only person in the world who knows a little bit about physics. If you read this, you know where Im coming from. Radiation is not two opposing arrows on a piece of paper. Its slightly more complex than that:”

            Indeed! Thermal EMR with wide bandwidth, near zero coherence interval,powered only by Boltzmann (kTb) statistics. Seems to require more than understanding. Such requires continual, repeated, pawing through the junk box and finding, ‘that doesn’t fit quite right either’!
            Once you can recognize Lorentz invariance peeking from within Maxwell’s 22 equations but find that missing from the 4 or 6 of John Pointing, you can try something else from the junk box.

            “You might think you know what Im saying, Joel, but you obviously dont ”

            Kristian, Your expressions are from a different POV than my own. Yours I can disagree with, but I cannot ignore such!

          • Joel Shore says: September 11, 2016 at 6:34 AM

            Will says:(Such can easily explain the Gravitationally induced thermostatic lapse rate of Earths atmosphere, we think! This (of course) may be falsified at anytime. So far the totally incompetent academic physicists cannot even figure out what that term may possibly mean.)

            “(1) If you actually read a textbooks in atmospheric science,”

            Why would anyone bother to read one of your referenced “textbooks”
            Any Catechism has more religious understanding. Any Mad magazine has more science!

            “you would know that the dry adiabatic lapse rate is understood as indeed being related to the gravitational acceleration and the specific heat.”

            There is never anything possibly ‘adiabatic’ about any ‘well mixed’ troposphere! The coincidence that a non condensing maximum stable Earth thermal lapse rate being near -g/Cp likely will never be solved. All is religious fantasy. Changing atmosphere to 1 x 10^18 kg SF6, would be unlikely to change Earth’s surface pressure or thermal lapse rate! All you ever have is religious nonsense.

            “There is no debate on this. [The only confusion that sometimes arises is that some people think this is the equilibrium temperature in the atmosphere whereas the adiabatic lapse rate really represents a stability limit for the lapse rate. I.e., lapse rates steeper than this tend to spark convection, which drives down the lapse rate, whereas lapse rates less steep are stable (which is why the stratosphere does not have the sort of lapse rate one sees in the troposphere).]”

            Blah da da, blah da da, blah da da! Go somewhere and buy a clue!

            “(2) Despite your musings, there are not two different types of physicists who believe two different things. I have worked in both industry and academia and physicists in industry dont go around thinking that physicists in academia have different laws of physics than they do.”

            There are those that ‘can’, they all ‘do’! There are those that ‘cannot’, they all teach!

        • Kristian says:

          Tim Folkerts says, September 9, 2016 at 8:32 AM:

          Specifically, by saying the “two flow model” is only ‘mathematical’, you are sending the message that you can’t have 2 people coming in the back door to get a net flow 8 out.

          *Sigh*

          Read what Norman writes, Tim. He says that by ADDING more energy to a surface, you reduce the energy ESCAPING it. Do you find this a logical statement?

          Start by telling Norman how this process should be described, not me. And then we could perhaps start talking. Because this topic – when you finally get down to it – is really all about definition.

          • Tim Folkerts says:

            Kristian said earlier:
            “I quote you [Norman] verbatim: Slow the rate at which radiant energy leaves and the temperature rises. ” …

            And then later wrote.
            “Read what Norman writes, Tim. He says that by ADDING more energy to a surface, you reduce the energy ESCAPING it. “

            You may have quoted verbatim, but you didn’t seem to catch the context. Specifically, shortly after the first sentence you quoted, Norman said “Backradiation overall effect is to reduce the NET radiation emitted by the surface with the same incoming energy. “

            It seems pretty clear that Norman’s *intent* has always been that reducing the *NET* outgoing radiation (relative to some previous steady-state condition) leads to warming. He just says it using a 2-flow model (and occasionally doesn’t explicitly state the 2-flow model or the word “net”).

          • Norman says:

            Kristian

            Not exactly what I am saying. It is how you are interpreting it though.

            It is most obvious you do not understand how the GHE works and need really detailed linking thoughts to understand what is going on. Tim understands it.

            YOU: “Read what Norman writes, Tim. He says that by ADDING more energy to a surface, you reduce the energy ESCAPING it. Do you find this a logical statement?”

            You have to understand where DWIR comes from in the first place. It is created by the surface energy flows (all combined in the real world, just radiation in Roy’s simple model). Energy from the surface (radiation, latent heat, conduction and convection) are what warm the atmosphere and generate the DWIR. This is why you treat IR as a Net energy. You do not do it with solar because solar is its own energy.

            Does that make sense Kristian? It is a necessary step to build up the logical connection of what I post that you do not interpret correctly.

            The atmosphere absorbs 77 W/m^2 of solar flux so maybe about 39 W/m^2 of DWIR could come from solar energy absorbed in the atmosphere (since if it were the only source of atmospheric warming you would get a 38.5 Flux up and a 38.5 flux down to Earth).

            Most of the 100’s of W/m^2 of DWIR comes from the surface warming the atmosphere.

            Of the 398 W/m^2 that is leaving the Earth’s surface, a large amount of this is absorbed and converted to warming energy of the Earth. Of the 398 being emitted only 240 goes out to space, it means 158 W/m^2 are absorbed by the atmosphere and 345 are returned to the surface so the surface net loss is 53 W/m^2. It is what allows the solar input to pile up and raise the surface to a higher equilibrium temperature.

            The reason the DWIR reduces the amount leaving is because it is part of the UWIR, not a distinct or separate energy system.

            I doubt it will help but if you respond I will try to keep clarifying the situation.

          • Kristian says:

            Tim Folkerts says, September 9, 2016 at 10:56 AM:

            You may have quoted verbatim, but you didnt seem to catch the context. Specifically, shortly after the first sentence you quoted, Norman said “Backradiation overall effect is to reduce the NET radiation emitted by the surface with the same incoming energy.”

            It seems pretty clear that Norman’s *intent* has always been that reducing the *NET* outgoing radiation (relative to some previous steady-state condition) leads to warming. He just says it using a 2-flow model (and occasionally doesn’t explicitly state the 2-flow model or the word “net”).

            Then you haven’t paid much attention to what Norman has been saying. If there is one confused individual on this thread (in addition to Ball4, of course), it is Norman.

            Here’s what he originally stated:
            “By lowering the amount of radiant energy leaving [the surface], however, [the DWIR] will allow the constant solar input to increase the temperature of the surface leading to a warmer surface than without DWIR”

            When the absurdity of this statement was pointed out to him, he modified it thusly:
            “By lowering the amount of NET radiant energy leaving, however, it will allow the constant solar input to increase the temperature of the surface leading to a warmer surface than without DWIR”

            But he’s still not getting the point. The way he’s describing this process, there is no “NET radiant energy” leaving the surface. The only radiant energy leaving the surface according to his description is the UWLWIR. The DWLWIR, the other component of the net is rather entering the surface. And when the DWLWIR increases, that does NOT entail a reduction in the radiant energy leaving the surface, and it does not constitute a reduction in the NET radiant energy leaving the surface either. It’s not reducing any energy flows of any kind, in any direction. It is an increased energy INPUT, causing the energy OUTPUT to increase as well (+T -> +UWLWIR).

            Look what Norman says even in the quote you provide above:
            “Backradiation overall effect is to reduce the NET radiation emitted by the surface with the same incoming energy.”

            According to the two-way transfer model, the surface DOESN’T emit net radiation at all. It ONLY emits UWLWIR. And so, if you increase the DWLWIR you haven’t reduced the energy emitted by the surface at all. You have simply increased the energy INPUT to the surface.

            Directly – upon absorp tion – making the surface warmer. It doesn’t do this in the current state. Neither does the solar input. Because we’re in a steady state (a dynamic equilibrium) by now. But the DWLWIR (just as the solar flux) sure did raise the T_avg of the surface up to the point where this steady state was attained. According to the “back radiation” explanation of the “GHE”, based on a misinterpretation of the two-way transfer model.

            If everyone could just stick to the original, standard setup of the Stefan-Boltzmann equation …:
            q/A = σ (T_sfc^4 – T_atm^4) (unidirectional transfer)

            … rather than the mathematically customised version of “Climate Science^TM” …:
            q/A = σ T_sfc^4 – σ T_atm^4 (bidirectional transfer)

            … then all would be just fine. No confusion about causality. Then everyone would see at once that the rate of radiant heat loss from the surface simply changes as the atmospheric temperature changes, on a general level.

            And then the rate of bulk radiant energy movement (the macroscopic/thermodynamic flux), the actual transfer of energy moving from the warmer surface, through the radiation field, to the cooler atmosphere, could indeed be said to have been reduced. Because this movement naturally and spontaneously follows a downward gradient of radiation intensity and/or energy density. The ENERGY is always there, the net MOVEMENT of the energy isn’t. And that makes all the difference between a transfer and a non-transfer.

            Macroscopically, there is only the ONE net movement of radiant energy. From hot to cold.

            This perspective shouldn’t be that hard to grasp.

          • Ball4 says:

            “q/A = σ (T_sfc^4 – T_atm^4) (unidirectional transfer)”

            That is bidirectional energy transfer Kristian, photons from the surface irradiate the atm., photons from the atm. irradiate the sfc. Kristian is simply confused calling it unidirectional. Dr. Spencer’s last post test:

            q/A = σ (T_hotplate^4 T_ice^4) (bidirectional energy transfer by virtue of a temperature difference, two independent photon streams up & down)

            Then:

            q’/A = σ (T_hotplate^4 T_cardboard^4) (bidirectional energy transfer by virtue of a temperature difference, two independent photon streams up & down)
            )

            q .NE. q’ as shown in the data. The hot plate irradiates the ice, the ice irradiates the hot plate as does the cardboard. In the data.

            The heat is a measure of the net bidirectional energy transferred in the KE of the hot plate as calculated by the thermometer (and ice, cardboard if they had thermometers).

          • Ball4 says:

            Better?

            “q/A = σ (T_sfc^4 – T_atm^4) (unidirectional transfer)”

            That is bidirectional energy transfer Kristian, photons from the surface irradiate the atm., photons from the atm. irradiate the sfc. Kristian is simply confused calling it unidirectional. Dr. Spencer’s last post test:

            q/A = σ (T_hotplate^4 – T_ice^4) (bidirectional energy transfer by virtue of a temperature difference, two independent photon streams up & down)

            Then:

            q’/A = σ (T_hotplate^4 – T_cardboard^4) (bidirectional energy transfer by virtue of a temperature difference, two independent photon streams up & down)

            q .NE. q’ as shown in the data. The hot plate irradiates the ice, the ice irradiates the hot plate as does the cardboard. In the data.

            The heat is a measure of the net bidirectional energy transferred in the KE of the hot plate as calculated by the thermometer (and ice, cardboard if they had thermometers).

          • Tim Folkerts says:

            Sorry, Kristian, I have to side with Norman here about “net radiant energy leaving”. No physicist would ever try to make the distinction you are trying to make.

            Consider accounting as an analogy. You can increase “net income” by decreasing the “outgo” (expenses). No one considers this terminology in the least bit odd. The term “net income” includes flows of money in and flows of money out. Net income does NOT mean only summing actual money coming in from various sources.

            Its the same in physics. In this case, “leaving” is merely an adjective to tell people which direction is to be considered positive:
            (net radiant energy leaving) = – (net radiant energy arriving)
            Either expression would include flows in both directions, just with an opposite sign convention.

            Another example is in the 1st law of thermodynamics, which some write as Delta(U) = Q + W and others write as Delta(U) = Q – W. In the first case, “W” = “net work done ON the system” and in the send case, “W” = “net work done BY the system”. Both versions of “W” would depend on the work ON the system and work BY the system; they would just have opposite signs everywhere.

          • gbaikie says:

            –You have to understand where DWIR comes from in the first place. It is created by the surface energy flows (all combined in the real world, just radiation in Roys simple model). Energy from the surface (radiation, latent heat, conduction and convection) are what warm the atmosphere and generate the DWIR. This is why you treat IR as a Net energy. You do not do it with solar because solar is its own energy.–

            Have very tall [and large] cylinder on the Moon.
            Measure surface temperature inside and at bottom of cylinder.
            The add air into cylinder. The cylinder has open top.
            If an molecule is warm, it’s velocity could be 500 m/s.
            And the cylinder could high enough so 500 m/s does not escape lunar gravity and escape thru the top of cylinder. Or if high enough the top of cylinder doesn’t need a roof/ceiling/lid- it can be open.

            Put in enough gas so one has 1 psi of air pressure at the surface.
            And it would be 1 psi regardless of the temperature of gas. But
            the hotter the gas the higher the majority of gas is. Or if gas is say 150 K, most gas will be closer to the surface- like within 10 meters of the surface. Whereas 400 K gas maybe within 1 or 5 km of the surface. Or has to be very tall cylinder.
            So with very tall cylinder, add 10 psi of air [giving a air pressure similar to earth]. with 10 psi pressure at surface, the density of air at surface will be largely effected by temperature
            of gas. Or if gas is cool enough it could be same density are
            Earth surface air density- 1.2 kg per cubic meter.
            Because the Moon has low gravity, this would be cold air- wild guess less than 0 C.
            Also with 1/6th the gravity one needs 6 time more air mass above
            the surface inside cylinder. So to get 10 psi on the Moon requires more atmosphere as compare to with world with Earth’s gravity.
            Or depending on how you want to look at this, this 10 psi air at less than 0 C could be viewed as having more energy per 1 meter square vertical column from surface to space.
            Or to make cylinder hold higher temperatures it’s really, really
            tall [in terms of anything made by humans].
            So have really tall cylinder and have sunlight warm the air to 100 C. That very energetic and hot air, will not allow the surface
            of the moon to become hotter than lunar surface without a cylinder above it.
            And it does not matter what gases you add.
            Now without any gases in cylinder it’s possible the surface is warmer than rest of lunar surface- the cylinder reflects and increases amount of sunlight reaching surface. That why you check it’s temperature before putting gases in it. But other than that
            the gases aren’t going to increase the surface temperature- though the addition of gases will certainly will increase the average surface temperature.

          • Joel Shore says:

            Kristian says: ” rather than the mathematically customised version of Climate Science^TM :
            q/A = σ T_sfc^4 σ T_atm^4 (bidirectional transfer)”

            If you would actually read physics textbooks, you would find that the view you attribute to climate science is the view that physicists also subscribe to, which sort of explains why those of us who are physicists are always telling you that you are wrong, alas to no avail.

          • Joel Shore says:

            “Because this movement naturally and spontaneously follows a downward gradient of radiation intensity and/or energy density.”

            Since you are implicitly making analogies to other things, like a gradient in pressure, I am curious to know if you understand them any better. If I have two containers of gas separated by a permeable membrane and one is at a higher pressure and one at a lower pressure and I now poke a hole in the membrane, then I think we both agree the NET flow of gas molecules will be from the higher pressure container to the lower pressure container. Now here’s the question: Do you believe this to be because there is a unidirectional flow of gas molecules from the higher pressure container to the lower pressure container or do you believe that there are flows in both directions but a larger flow of molecules from the high to lower pressure and a smaller flow of molecules from the low to higher pressure?

          • Ball4 says: September 9, 2016 at 12:56 PM

            “Better?”
            No

            Kristian:(q/A = σ (T_sfc^4 T_atm^4) (unidirectional transfer))

            “That is bidirectional energy transfer Kristian, photons from the surface irradiate the atm., photons from the atm. irradiate the sfc. Kristian is simply confused calling it unidirectional.”

            Trick,
            There is absolutely no spontanious bidirectional thermal EM energy transfer. Such cannot exist ever! See: http://www.drroyspencer.com/2016/08/simple-time-dependent-model-of-the-atmospheric-greenhouse-effect/#comment-224455

          • Ball4 says:

            I agree Joel, Kristian writes this as I understand his thinking as some sort of a heat eqn. (one way) when it is really a radiant energy 2way transfer eqn.:

            “q/A = σ (T_sfc^4 – T_atm^4) (unidirectional transfer)”

            Which is really for bidirectional radiant energy transfer. Works for inversion too. Strictly, this is an idealization given that monodirectional irradiances do not exist; even a laser beam has a finite angular spread.

            Also, many feel too constrained to just up and down idealization given the photon bath existing at the surface but it serves a basic purpose to start – as shown in the top post idealization and the test Dr. Spencer shows in last post. It can be pushed too far, restraint is needed.

            A consequence of 2stream is scattering can therefore occur in only these two directions: a photon directed downward can be scattered only downward or upward, and similarly for a photon directed upward. This being tough for many to swallow simply debate endlessly, as demonstrated by Kristian. Idealizations are for starting to understand not ending.

          • Ball4 says:

            Will, you are completely wrong, buy a clue, better buy a copy of “The Theory of Heat Radiation.”. See:

            http://www.drroyspencer.com/2016/08/simple-time-dependent-model-of-the-atmospheric-greenhouse-effect/#comment-224464

          • Joel Shore says: September 9, 2016 at 7:05 PM

            Kristian says: rather than the mathematically customised version of Climate Science^TM :
            q/A = σ T_sfc^4 σ T_atm^4 (bidirectional transfer)

            No another JS lie: Kristain actually writes:

            If everyone could just stick to the original, standard setup of the Stefan-Boltzmann equation :
            q/A = σ (T_sfc^4 T_atm^4) (unidirectional transfer)

            rather than the mathematically customised version of Climate Science^TM :
            q/A = σ T_sfc^4 σ T_atm^4 (bidirectional transfer)

            “If you would actually read physics textbooks, you would find that the view you attribute to climate science is the view that physicists also subscribe to, which sort of explains why those of us who are physicists are always telling you that you are wrong, alas to no avail.”

            The infamous Joel Shore arrives!!!
            Why oh why would anyone want to read a corrupt Berkly ‘post normal’ catechism of pseudoscience? The original Stefan-Boltzmann equation is available as a true scientific masterpeice, never ever falsified!… While the broken Joel Shore climatism version has never even once been observed; simply because such cannot be possible.

            Ahhh “why those of us who are physicists are always telling you that you are wrong”. The academic cry! You must trust us,we know! WE ARE SKYINTISTS, you rat boy know nothing! see:
            http://www.drroyspencer.com/2016/08/simple-time-dependent-model-of-the-atmospheric-greenhouse-effect/#comment-224455

          • gbaikie says:

            “the higher pressure container to the lower pressure container. Now heres the question: Do you believe this to be because there is a unidirectional flow of gas molecules from the higher pressure container to the lower pressure container or do you believe that there are flows in both directions but a larger flow of molecules from the high to lower pressure and a smaller flow of molecules from the low to higher pressure?”

            If had 20 psi tire filled with helium and fill tire to 25 psi
            by addinfg compressed air from 120 psi pressure tank, it would surprise me that helium could travel back to compressor tank.
            Despite knowing that helium molecules average velocity is much faster than air.

          • Ball4 says: September 9, 2016 at 7:30 PM

            “I agree Joel, Kristian writes this as I understand his thinking as some sort of a heat eqn. (one way) when it is really a radiant energy 2way transfer eqn.:

            q/A = σ (T_sfc^4 T_atm^4) (unidirectional transfer)

            What monumental BS! That is the only flux, with the sign indicating direction!

            “Which is really for bidirectional radiant energy transfer. Works for inversion too. Strictly, this is an idealization given that monodirectional irradiances do not exist; even a laser beam has a finite angular spread.”

            False! lasers with constant physical beam size are difficult but not impossible, just expensive. Generally a beam focused at a particular distance is an acceptable compromise. That S-B equation is only a maximum total flux between two infinate Lambertial surfaces of zero curvature. The S-B equation includes the effective radiating solid angle of PI steradians for such Lambertian surfaces.
            True!! mono-directional irradiances do not exist from any source of finite solid angle area! But again irradiance is still not necessarily a flux. Irradiance is precisely, and only the UN-normalized field strength at some remote location from the source. It is the integral of radiance over the total solid angle of the source.
            If and only if there is no opposing (field strength (radiance) will irradiance result in that value of flux in W/m^2. Integrate that over your local area and you get the one way power physically transfered. If that thermal EM field strength is precisely opposed in every direction throughout the area, direction and frequency; no flux in any direction or frequency shall be generated.

          • Kristian says:

            Tim Folkerts says, September 9, 2016 at 1:59 PM:

            Sorry, Kristian, I have to side with Norman here about “net radiant energy leaving”. No physicist would ever try to make the distinction you are trying to make.

            Consider accounting as an analogy. You can increase “net income” by decreasing the “outgo” (expenses). No one considers this terminology in the least bit odd. The term “net income” includes flows of money in and flows of money out. Net income does NOT mean only summing actual money coming in from various sources.

            Its the same in physics. In this case, “leaving” is merely an adjective to tell people which direction is to be considered positive:
            (net radiant energy leaving) = (net radiant energy arriving)
            Either expression would include flows in both directions, just with an opposite sign convention.

            Another example is in the 1st law of thermodynamics, which some write as Delta(U) = Q + W and others write as Delta(U) = Q W. In the first case, “W” = “net work done ON the system” and in the send case, “W” = “net work done BY the system”. Both versions of “W” would depend on the work ON the system and work BY the system; they would just have opposite signs everywhere.

            Thanks for that paternal lecture, Tim. I really didn’t know anything about any of those things! Now, can we get back to the real issue …?

            In the “back radiation” explanation of the “GHE”, the outgoing ‘surface net LW’ term [Q_lw] – the only thermodynamically relevant radiative term besides the incoming ‘net SW’ [Q_sw] from the Sun – is essentially drained of its inherent physical meaning and significance, broken up and dispersed by simple mathematical tricks. Bestowing instead individual thermodynamic powers on its conceptual components:

            Q_sw(in) + DWLWIR – Q_c+e(out) = UWLWIR

            To find the UWLWIR on the righthand side, the final rate of outgoing radiant energy from the surface, assumed to be a direct resultant of the final surface temperature (σT^4), all you need to know is 1) the incoming solar heat flux [Q_sw(in)], 2) the DWLWIR, the final rate of incoming radiant energy from the atmosphere to the surface, and 3) the final outgoing conductive-eaporative heat flux [Q_c+e(out)].

            So if you know that the Q_sw(in) is 165 W/m^2, the DWLWIR is 345 W/m^2, and the Q_c+e(out) is 112 W/m^2, then you can easily derive Earth’s steady-state surface T_avg:

            165 + 345 – 112 = 398 W/m^2 -> 289 K

            Ingenious, isn’t it?

            In this simple explanation of Earth’s surface T_avg, the ‘surface net LW’ [Q_lw(out)] is basically a non-entity, a complete irrelevance, if anything, a mere EFFECT of the change in DWLWIR, just like the surface T_avg and the UWLWIR.

            In this explanation, it is specifically NOT by reducing its ‘net LW’ that you increase the surface U and thereby its T. You do it rather by increasing the energy input from the atmosphere, the DWLWIR. This directly causes TWO effects: i) the surface U and T goes up, and ii) the surface ‘net LW’ (UWLWIR-DWLWIR) goes down.

            In this explanation, the decrease in ‘net LW’ [Q_lw(out)] is reduced to a direct EFFECT of the increase in DWLWIR, right alongside the rise in surface U and T.

            The difference is simple: Rather than also letting two extra people per minute IN through the same door where ten people leave every minute, we place sort of a reverse bouncer at the door, refusing two of the ten people normally leaving exit.

            This is equivalent to what I described above: Less radiant energy (people) per unit of time is allowed to move away from the heated surface (the room), because there is a reduced gradient of radiative intensity and/or energy density facing it (the reverse bouncer). Again, the radiant energy itself (the ‘photon cloud’) is always there, everywhere, but a CHANGE in the average intensity in one or more directions through the radiation field is required for there to be a change in the average (net) MOVEMENT of radiant energy relative to the heated surface.

            http://www.rpi.edu/dept/phys/Courses/PHYS4420/BlackBodyThermo.pdf

            “The system to which the thermodynamics is being applied is certainly a strange one when compared to typical problems encountered in say, engineering thermodynamics. This system consists of electromagnetic radiation in thermodynamic equilibrium inside a closed, completely evacuated cavity of arbitrary shape with volume V and temperature T. Volume and temperature represent the two independent and measureable parameters in terms of which all thermodynamic variables may be expressed. Since equilibrium is assumed, one may define the radiation temperature as that of the walls. The system is an isothermal enclosure, and every point has the property that the intensity is independent of position. Furthermore, the radiation is isotropic and unpolarized.

            A typical approach taken by most modern physics texts is to treat the radiation as a series of standing waves. The normal-mode density and energy per mode are calculated, and this leads to the Planck law. An alternative viewpoint is taken in most statistical mechanics texts where one considers the system to consist of a photon gas that obeys Einstein-Bose statistics.

            (- – -)

            There are two somewhat different historical approaches to blackbody radiation. The first, due to Planck in 1900, considered the system as an assembly of harmonic oscillators with quantized energies of (n+1/2)ℏω (although Planck did not include the zero-point energy). The second viewpoint originated in Bose in 1924 and then Einstein in 1925, which considered the photon distribution over the energy levels. The two interpretations are actually the same; for example, in Planck’s method, an oscillator of energy (n+1/2)ℏω in the eigenstate n is equivalent to n photons in the energy level ℏω.

            A microscopic observer would find experiments to be rather dull at any point immersed in the blackbody radiation field. Since the field is isotropic and homogeneous, the luminosity would be independent of direction and he would be unaware of the cavity size in any direction. Furthermore, no polarization effects would be detected. If the temperature were varied, then he would measure changes in intensity and energy distribution (corresponding to a color change).”

            The TEMPERATURE of the atmosphere (plus its density/pressure/weight) is what limits the (total) surface heat loss at any particular surface T_avg, not its radiation. Its radiation is an EFFECT of temperature, not a CAUSE of it. If the effective temperature difference is small, then the apparent radiation difference is small too. But it’s not the radiation difference that causes the temperature difference. It’s the other way around …

          • Kristian says:

            Joel Shore says, September 9, 2016 at 7:05 PM:

            If you would actually read physics textbooks, you would find that the view you attribute to climate science is the view that physicists also subscribe to, which sort of explains why those of us who are physicists are always telling you that you are wrong, alas to no avail.

            And naturally, along comes arrogant little Joel. Yes, in order to work with these terms mathematically, you are of course free to shuffle them about. Like I said. But that doesn’t mean that these alternative versions of the original formula are somehow describing alternative versions of reality. Reality is what dictated the original formula, after all. Physicists – with some real-world insight and perspective, that is, who have been able to look beyond the tip of their nose when amassing their knowledge – know this. “Climate scientists” apparently don’t. Because they apparently think that it’s ok PHYSICALLY to let mere conceptual components of radiant heat fluxes increase the U and T of thermodynamic systems, directly and all by themselves, simply because it seems ok MATHEMATICALLY:
            https://okulaer.files.wordpress.com/2014/10/drivhuseffekten.png

          • Ball4 says:

            Will says “That is the only flux, with the sign indicating direction!”

            No Will, you are proven completely wrong by Dr. Spencer’s experiment in which the cardboard had a proven effect in the data. Both the cardboard and ice irradiated the hot plate with photons. If what you write were true inserting the cardboard would have had no effect. Buy a clue Will and pick up a copy of “The Theory of Heat Radiation” and find out exactly why you are completely wrong.

          • Ball4 says:

            Kristian incorrectly writes: “its ok PHYSICALLY to let mere conceptual components of radiant heat fluxes increase the U and T of thermodynamic systems, directly and all by themselves, simply because it seems ok MATHEMATICALLY:”

            It is ok mathematically AND physically. Dr. Spencer’s test proved radiant energy fluxes are not merely mathematical Kristian, they are physical too:

            q/A = σ (T_hotplate^4 – T_ice^4) (bidirectional energy transfer by virtue of a temperature difference, two independent photon streams ice up & hot plate down)

            as shown in his data, the added heat melting the ice was a measure of the KE exchanged, the temperature of the hot plate changing with the cardboard insertion showed as KE change on the thermometer.

            The ice & cardboard irradiated the hot plate as shown in the data.

          • Joel Shore says:

            Kristian says: “And naturally, along comes arrogant little Joel….”

            You have a strange definition of arrogance. Apparently, it is not arrogant if you fancy yourself an expert in physics and what physicists understand about thermodynamics even though you have no background in this area and particular have never taken a course or apparently read anything about statistical physics, the foundation upon which our modern understanding of thermodynamics rests. (And have not talked to any who have this background who actually agree with your point-of-view!)

            However, it is arrogant if you have actually committed the time to studying the relevant fields of physics, earning a PhD in physics, with research in statistical physics and you have the gall to point out the basic errors and lack of knowledge that one of the “not arrogant” people is demonstrating.

          • Ball4 says: September 10, 2016 at 5:28 AM

            “Kristian incorrectly writes: (its ok PHYSICALLY to let mere conceptual components of radiant heat fluxes increase the U and T of thermodynamic systems, directly and all by themselves, simply because it seems ok MATHEMATICALLY:)

            “It is ok mathematically AND physically. Dr. Spencers test proved radiant energy fluxes are not merely mathematical Kristian, they are physical too:”

            Again, only if you believe the religious fantasy of spontaneous thermal EMR flux in opposing direction!! such as:

            “q/A = σ (T_hotplate^4 T_ice^4) (bidirectional energy transfer by virtue of a temperature difference, two independent photon streams ice up & hot plate down)”

            What two photon streams? you pull this crap from the terlet!!

            “as shown in his data, the added heat melting the ice was a measure of the KE exchanged, the temperature of the hot plate changing with the cardboard insertion showed as KE change on the thermometer.”

            How does that ever indicate opposing thermal flux.

            “The ice & cardboard irradiated the hot plate as shown in the data.”

            Of course and yes! you fail to recognize that the term irradiate is merely an UN-normalized “radiance” or power transfer potential. Irradiance never, ever, refers to any flux. This is quite independent of how Wm STOAT Connolley changed the description of irradiance at Wikipedia!!

          • Ball4 says:

            “What two photon streams?”

            The one from the hot plate and the one from the ice.

          • Kristian says:

            Joel,

            You’re an arrogant little bugger (I wanted to write ‘twat’). You might not think so yourself, but you are. It seems you can’t help yourself.

            You’re not the only person in the world who knows a little bit about physics. If you read this, you know where I’m coming from. Radiation is not two opposing arrows on a piece of paper. It’s slightly more complex than that:

            http://www.astro.wisc.edu/~townsend/resource/teaching/astro-310-F08/09-radiation-field.pdf
            http://www.astroscu.unam.mx/~alan/atmosferas/chapter-1.pdf
            https://www.science.mcmaster.ca/medphys/images/files/courses/775/ch3.pdf

            You might think you know what I’m saying, Joel, but you obviously don’t …

          • Ball4 says:

            “If you read this, you know where Im coming from.”

            Not really Kristian. Except for one use of gamma, the ratio of specific heats, not one of those pieces uses the term “heat” so there is no need for Kristian to use it either when discussing radiation. Used properly, heat is simply the measure of KE in an object.

          • Ball4 says: September 10, 2016 at 8:40 PM

            (If you read this, you know where Im coming from.)

            Not really Kristian. Except for one use of gamma, the ratio of specific heats, not one of those pieces uses the term heat so there is no need for Kristian to use it either when discussing radiation.”

            Trick,
            Please go somewhere, anywhere, get your shoes tied, face wiped, diaper changed, with asswipe by someone nice!
            The term ‘Gamma’ is the gas chemistry term for ratio Cp/Cv yielding a value of 7/5, 1.4 for Earth’s atmosphere. Such reviles nothing of this atmosphere. That same value 7/5, 1.4 is called ‘isentropic exponent’ kappa in all engineering. Such can easily explain the Gravitationally induced thermostatic lapse rate of Earth’s atmosphere, we think! This (of course) may be ‘falsified’ at anytime.
            So far the totally incompetent academic physicists cannot even figure out what that term may possibly mean.

            “Used properly, heat is simply the measure of KE in an object.”

            Grand BS! So far temperature seems to be indicative of both sensible heat (some accumulation of power) plus gas mass pressure (a different accumulation of power) while never including the accumulation of coherent momentum power (Mv^2/2) which seems not represented by temperature.
            Why oh why do you attempt to promote your religious nonsense?

          • Joel Shore says:

            Will says: “Such can easily explain the Gravitationally induced thermostatic lapse rate of Earths atmosphere, we think! This (of course) may be falsified at anytime.
            So far the totally incompetent academic physicists cannot even figure out what that term may possibly mean.”

            (1) If you actually read a textbooks in atmospheric science, you would know that the dry adiabatic lapse rate is understood as indeed being related to the gravitational acceleration and the specific heat. There is no debate on this. [The only confusion that sometimes arises is that some people think this is the equilibrium temperature in the atmosphere whereas the adiabatic lapse rate really represents a stability limit for the lapse rate. I.e., lapse rates steeper than this tend to spark convection, which drives down the lapse rate, whereas lapse rates less steep are stable (which is why the stratosphere does not have the sort of lapse rate one sees in the troposphere).]

            (2) Despite your musings, there are not two different types of physicists who believe two different things. I have worked in both industry and academia and physicists in industry don’t go around thinking that physicists in academia have different laws of physics than they do.

          • Joel Shore says:

            How ironic! I am not the one who thinks this…You are. I think that a lot of people know physics, including here Tim Folkerts, Ball4, Roy Spencer, to name just a few. I also think that those who write the physics textbooks and the great physicists of the late 19th and early 20th centuries who codified statistical physics know physics.

            You are the one who thinks you are so wise that you can understand physics without even familiarizing yourself with this stuff. It must be impressive to be so brilliant that you can have a better understanding of thermodynamics than those who have studied the statistical physics that underlies it.

          • Joel Shore says:

            Sorry…My last post was responding to this quote that I accidently cut off—

            Kristian says: “Youre not the only person in the world who knows a little bit about physics.”

          • Joel Shore says:

            Fine links, Kristian, but none of them give any evidence whatsoever to support your view. They simply don’t deal with the question of two bodies (or a body and its surroundings) interacting radiatively. Whereas, I have provided you with textbook references that directly support our point of view…essentially any elementary physics textbook that writes down the Stefan-Boltzmann Equation applied to a body and its surroundings. For example, in University Physics by Young & Freedman (the textbook we happen to use in the introductory physics course), they explain the equation as being the amount of radiation that a body emits minus the amount it absorbs from its surroundings. (I tried to quote the book directly but something in the spam filter for this site doesn’t like it.)